Stroke, Stroke NCLEX, HESI- Brain Attack (Stroke), Brain Attack / Stroke, Chapter 45 Care of Critically ILL Patients with Neurologic Problems, Stroke nclex problems, HEC IGGY 45/TBI, stroke, brain attack

Ace your homework & exams now with Quizwiz!

Question 27 Type: FIB A patient has been admitted with stroke-like symptoms. The nurse would report a serum potassium level of less than _____mEq/L as below the desired level.

Correct Answer: 3.5 Rationale : The goal for potassium is between 3.5 and 5.3 mEq/L.

The stroke patient is prescribed docusate (Colace) once a day in the morning. What is the purpose of this drug specific to this patient? a. Laxative to prevent constipation b. Soften the patient's stool c. Increase fluid content of stool d. Prevent increased ICP

D

Which Glasgow coma scale (GCS) data set indicates the most severe injury and loss of consciousness? a. GCS of 13 with loss of consciousness of 15 minutes b. GCS of 9 with loss of consciousness of 30 minutes c. GCS of 12 with loss of consciousness of 3 hours d. GCS of 8 with loss of consciousness of 6.5 hours

D

The nurse has completed teaching a patient about carotid artery angioplasty with stenting (CAS). Which statement by the patient indicates understanding of the purpose of the procedure? a. "The stent opens the blockage enough to establish blood flow" b. "The stent occludes the abnormal artery to prevent bleeding." c. "The stent bypasses the blockage." d. "The stent catches any clot debris."

A

Which description best defines a basilar skull fracture? a. A simple, clean break in the skull b. A direct opening to brain tissue c. Fragments of bone are in brain tissue d. Cerebrospinal fluid leaks from nose or ears

D

Which statement is true about increased ICP in a surgical patient? a. It is a minor postoperative complication b. Diuretics such as furosemide may be given to decrease it. c. Cerebral edema usually subsides within 72 hours d. If not contraindicated, the head of the bed should be placed at 30 degrees

D

A reduced head elevation of less than 25 degrees is better for what type of strokes?

reduced head elevation of less than 25 degrees can improve perfusion pressure to damaged brain in ischemic conditions like most strokes

Bed elevation and turning technique for someone with TBI

Log roll the pt during turning to avoid extreme hip flexión, and keep HOB elevated atleast 30 degrees

What are the major causes of impaired intracranial regulation?

-cerebral edema -increased intracranial pressure -brain tumors

What is decompressive craniectomy?

-remove portion of skill to lower ICP

Manifestations of left brain damage

Aphasia, inability to remember words

Absent gag reflex, check which CN?

CN IX

Impaired tongue movement, check which CN?

CN XII

Permissive hypertension

If you drop blood pressure then more green tissue (ischemic) turns red (infarcted) (only treat BP if >200)

What is a craniotomy?

Remove lesion, repair damaged area, drain blood and so on.

Which intervention should the nurse implement to prevent joint deformities?

A. Place Nancy in a prone position for 15 minutes at least 4 times a day This helps to promote hyperextension of the hip joints, which helps prevent knee and hip flexion contractures.

Describe unilateral body neglect syndrome.

The patient is unaware of the existence of his or her left or paralyzed side.

Blow or jolt to head may result in what?

Traumatic brain injury

lacunar stroke

a stroke resulting from occlusion of a small penetrating artery with development of a cavity in the place of the infarcted brain tissue

33. A nurse checks the oxygen in the circulating volume for adequate concentration to support the brain's need of _____% of the oxygen supply of the body.

ANS: 20 The brain requires 20% of the available oxygen to function and to avoid hypoxic damage.

stroke

death of brain cells that occurs when there is ischemia (inadequate blood flow) to a part of the brain or hemorrhage into the brain

Assess the patients ability to chew, which reflect the function of cranial nerve?

(CN) V.

In addition, note any facial paralysis or paresis with which nerve?

(CN) VII

What are the main difference between left and right hemisphere strokes?

**Left** -aphasia -agraphia -alexia -possible memory deficit -reading problems -deficits in right visual field -slowness -cautiousness -depression **right** -impaired sense of humor -disorientation of time, place, person -inability to recognize faces -loss of depth perception -impulsiveness -euphoria -overestimation of abilities -constant smiling

What is a acceleration-deceleration injury?

**acceleration**— caused by an external force contacting the head, suddenly placing head in motion. **deceleration**— occurs when the moving head is suddenly stopped or hits stationary object. **particularly affected are the basal nuclei and the hypothalamus**

Populations at greatest risk for poor intracranial regulation

-older adults (from degenerative pathologic conditions and injuries (falls)) -adolescents and young adults (from traumatic injury)

TBI: factors that contribute to high mortality are...

-Falls causing subdural hematomas (closed head injuries), especially chronic subdural hematomas -Poorly tolerated systemic stress, which is increased by admission to a high-stimuli environment -Medical complications, such as hypotension, hypertension, and cardiac problems -Decreased protective mechanisms, which make patients susceptible to infections (especially pneumonia) -Decreased immunologic competence, which is further diminished by brain injury

What are the causes of cerebral edema?

-Mass lesions (brain abscess, tumor, hematoma, hemorrhage) -vascular insult (anoxic and ischemic episodes, cerebral infarction, venous sinus thrombosis) -head injuries and brain surgery (contusion, hemorrhage, posttraumatic brain swelling) -cerebral infection (meningitis and encephalitis) -toxic or metabolic encephalopathic conditions (lead or arsenic intoxication, hepatic encephalopathy and uremia)

What would a neurologic assessment for a stroke consist of?

-cognitive changes -motor changes -sensory changes -cranial nerve assessment -CV assessment

Describe common features of a patient with unilateral neglect

-common in pts with R side stroke -risk for falls is high -increased risk for clot in legs **swallowing assessment done**

Expected physiologic function

-cranial vault/skull -blood brain barrier -meninges -autoregulation -cerebrospinal fluid -hyperventilation -age related differences

What are the most common reasons for TBI to occur?

-falls in older adults -car accident -alcohol and drugs

What are some complications of stroke?

-hydrocephalus -vasospasms -rebleeding or rupture (7-10 days, chance of rebleeding is 20%)

What are the categories of impaired intracranial regulation?

-impaired perfusion -compromised neurotransmission -glucose regulation -pathology

What are some common tests and tools for assessment of intracranial regulation?

-mental status testing -GCS scale -cranial nerve testing -intracranial pressure testing -measurement of cerebral perfusion pressure -National institute of health stroke scale (NIHSS)

What are some diagnostic tests for ICP

-neuroimaging (MRI, CT) -skull radiograph -electroencephalogram -brain biopsy -lumbar puncture

What are some interventions to lower ICP?

-positioning -activity management -airway management -hyperventilation -bowel management

Cushing's triad consists of what?

-severe hypertension -widened pulse pressure -bradycardia

Common modifiable risk factors for developing a stroke

-smoking -substance abuse -obesity -sedentary lifestyle -oral contraceptive use -heavy alcohol use -use of phenylpropanolamine (PPA), found in antihistamine drugs

4 major causes of hemorrhagic stroke

1)deep hypertensive intracerebral hemorrhages, 2)ruptured saccular aneurysms, 3)arteriovenous malformation,4)spontaneous lobar hemorrhages

Secondary prevention of stroke (3)

1. Anticoagulants 2. Antiplatelets 3. Surgery

Arterial sources of stroke (3)

1. Intracranial vascular disease 2. Carotid vascular disease 3. Aortic arch

Secondary prevention: Risk factor modification (4)

1. smoking cessation 2. diabetes control 3. aggressive cholesterol lowering 4. hypertension control

7. What should a nurse ensure as a priority for a patient immediately after a CVA? a. Preservation of motor function b. Airway maintenance c. Adequate hydration d. Control of elimination

ANS: B Adequate oxygenation prevents hypoxemia, which can extend and worsen effects of the CVA.

How often should you be checking on a pt that you suspect has a epidural bleeding?

5-10 minutes for changes in neurologic status. **LOC from a epidural and subdural hematoma is a neurosurgical emergency**

What age group has the high incidence of brain injury?

65-75

The neurologist diagnosis I'd ask you make a left sided brain attack, stroke. The neurologist determines that Mrs. Jackson is not a candidate for tissue plasminogen activator, tPA. Enoxaparin 1 mg/kg subcutaneously every 12 hours is prescribed. Mrs. Jackson weighs 145 pounds. How many milligrams of Enoxaparin Will the nurse administer in each dose?

66 mg 145/2.2 = 65.9kg x 1mg/kg = approx. 66 mg

A patient displays signs of increased ICP, confusion, slurred speech, and unilateral weakness in the upper extremity. Which diagnostic test for this patient does the nurse question? a. Lumbar puncture (LP) b. Computed tomography (CT) c. Positron emission tomography (PET) d. Magnetic resonance imaging (MRI)

A

A patient has sustained a major head injury and the nurse is assessing the patient's neurologic status every 2 hours. What early sign of increased ICP does the nurse monitor for? a. Change in the LOC b. Cheyne-Stokes respirations c. Severe hypertension with widened pulse pressure (Cushing's reflex) d. Dilated and nonreactive pupils

A

A patient sustained a stroke that affected the right hemisphere of the brain. The patient has visual spatial deficits and deficits of proprioception. After assessing the safety of the patient's home, the home health nurse identifies which environmental feature that represents a potential safety problem for this patient? a. The handrail that borders the bathtub is on f the left-hand side. b. The patient's favorite chair faces the front door of the house. c. The patient's bedside table is on the right-hand side of the bed. d. Family has relocated the patient to a ground-floor bedroom.

A

A patient who had a craniotomy develops the postoperative complication of syndrome of inappropriate antidiuretic hormone(SIADH). The patient's sodium level is 126 mEq/L and the serum osmolality is decreased. In light of this development, which physician order does the nurse question? a. Encourage oral fluids b. Normal saline IV at 150Ml/hr c. Strict intake and output d. Daily weights

A

A patient with a right cerebral stroke may have safety issues related to which factor? a. Poor impulse control b. Alexia and agraphia c. Loss of language and analytical skills d. Slow and cautious behavior

A

The nurse is caring for a patient admitted with the medical diagnosis of probably epidural hematoma and decreased level of consciousness. During the shift, the patient becomes lucid and is alert and talking. The family reports this is her baseline mental status. What is the nurse's next action? a. Stay with the patient and have the charge nurse alert the physician because this is an ominous sign for the patient. b. Document the patient's exact behaviors, compare to previous nursing entries, and continue the neurologic assessments every 2 hours. c. Point out to the family that the dangerous period has passed, but encourage them to leave so the patient does not become overly fatigued. d. Monitor the patient for the next 48 hours to 2 weeks because a subacute condition may be slowly developing.

A

The nurse is caring for a patient at risk for increased ICP. Which sign is most likely to be the first indication of increased ICP? a. Decline of level of consciousness b. Increase in systolic blood pressure c. Change in pupil size and response d. Abnormal posturing of extremities

A

The nurse is caring for a patient receiving medication therapy to prevent recurrence of stroke. Which medication is pharmacologically appropriate for this purpose? a. Enteric-coated aspirin (Ecotrin) b. Gabapentin (Neurontin) c. Recombinant tissue plasminogen activator (Retavase) d. Bevacizumab (Avastin)

A

The nurse is caring for an intubated patient with increased ICP. If the patient needs to be suctioned, which nursing action does the nurse take to avoid further aggravating the increased ICP? a. Manually hyperventilate with 100% oxygen before passing the catheter b. Maintain strict sterile technique when performing endotracheal suctioning c. Perform oral suctioning frequently, but do not perform endotracheal suctioning d. Obtain an order for an arterial blood gas before suctioning the patient

A

Which statement is true about respiratory problems in a patient with a major head injury? a. Atelectasis and pneumonia can be prevented by proper pulmonary hygiene b. Suctioning should be avoided because of the increase in ICP c. Neurologic pulmonary edema occurs frequently d. The patient should avoid breathing deeply because of increased ICP

A

Which statement is true for a patient with a basilar skull fracture? a. There is potential for hemorrhage caused by damage to the internal carotid artery. b. There is an increased risk for loss of functioning abilities such as toileting c. There is an increased risk for cytotoxic or cellular edema with loss of consciousness. d. There is potential for decorticate or decerebrate posturing with loss of motor function.

A

Which type of hematoma occurs between the skull and the dura? a. Epidural hematoma b. Subdural hematoma c. Intracranial hemorrhage d. Contusion

A

Glasgow Coma Scale (GCS) score: what is considered clinically important?

A change of 2 points is considered clinically important;

The nurse continues to monitor Nancy's condition closely. Which finding would require immediate intervention by the nurse?

A. Nancy's cardiac output is less than 4 L/min The normal range for cardiac output to ensure cerebral blood flow and oxygen delivery is 4 to 8 L/min.

Which written documentation should the nurse put in the client's record? A. PT reported that client became dizzy and was lowered back to the bed with the assistance of a gait belt B. PT notified the primary nurse that the client could not ambulate at this time because of dizziness C. Client had difficulty ambulating from the bed to the chair when accompanied by the PT, variance report completed D. Client experienced orthostatic hypotension when getting out of bed

A. PT reported that client became dizzy and was lowered back to the bed with the assistance of a gait belt - This documentation provides the factual data of the events that occurred.

Gail tells the nurse she is going to go outside to smoke a cigarette and will only be gone for a few minutes. Which statement is warranted in this situation?

A. "I should let you know that smoking is a strong risk factor for a brain attack." The nurse should teach Gail that smoking is a modifiable risk factor that could prevent her from having a stroke. Smoking increases the risk for hypertension, which is a risk factor for a stroke.

Which additional clinical manifestations should the RN expect to find if these symptoms Mrs. Jackson has have been caused by a brain attack (stroke)? (Select all that apply)

A. Carotid bruit B. Elevated BP E. Drooling

The nurse is discharging a patient admitted with a transient ischemic attack (TIA). For which medications might the nurse expect to provide discharge instructions (select all that apply)? A. Clopidogrel (Plavix) B. Enoxaparin (Lovenox) C. Dipyridamole (Persantine) D. Enteric-coated aspirin (Ecotrin) E. Tissue plasminogen activator (tPA)

A. Clopidogrel (Plavix) C. Dipyridamole (Persantine) D. Enteric-coated aspirin (Ecotrin) Aspirin is the most frequently used antiplatelet agent. Other drugs to prevent clot formation include clopidogrel (Plavix), dipyridamole (Persantine), ticlopidine (Ticlid), combined dipyridamole and aspirin (Aggrenox), and anticoagulant drugs, such as oral warfarin (Coumadin). Tissue plasminogen activator is a fibrinolytic medication used to treat ischemic stroke not prevent TIAs or strokes.

A female patient who had a stroke 24 hours ago has expressive aphasia. The nurse identifies the nursing diagnosis of impaired verbal communication. An appropriate nursing intervention to help the patient communicate is to a. ask questions that the patient can answer with "yes" or "no." b. develop a list of words that the patient can read and practice reciting. c. have the patient practice her facial and tongue exercises with a mirror. d. prevent embarrassing the patient by answering for her if she does not respond.

A. Communication will be facilitated and less frustrating to the patient when questions that require a "yes" or "no" response are used. When the language areas of the brain are injured, the patient might not be able to read or recite words, which will frustrate the patient without improving communication. Expressive aphasia is caused by damage to the language areas of the brain, not by the areas that control the motor aspects of speech. The nurse should allow time for the patient to respond.

The ED nurse is completing the admission assessment. Nancy is alert but struggles to answer questions. When she attempts to talk, she slurs her speech and appears very frightened. Which additional clinical manifestations should the nurse expect to find if Nancy's symptoms have been caused by a brain attack (stroke)? A. Difficulty swallowing B. Decreased bowel sounds C. A carotid bruit D. Elevated blood pressure E. Hyperreflexic deep tendon reflexes

A. Difficulty swallowing - Difficulty swallowing can accompany a brain attack, placing the client at risk for aspiration. C. A carotid bruit - The carotid artery (artery to the brain) is narrowed in clients with a brain attack (stroke). A bruit is an abnormal sound heard on auscultation resulting from interference with normal blood flow. D. Elevated blood pressure - When a client has a brain attack (stroke), the blood pressure will often respond by going up. Increased BP is a sign of increased intracranial pressure.

The RN notes that Mrs. Jackson is no longer able to meet her nutritional needs and has lost 10 lbs.A gastrostomy tube is prescribed so that intermittent tube feedings can be administered. Which intervention should the RN implement first?

A. Elevate the head of the bed to a semi-fowler's position during the feeding. Prevents aspiration

The neurologist writes a diagnosis of "Suspected brain attack" and prescribes a non contrast computed tomography (CT) scan STAT. Which nursing intervention should the nurse implement when preparing Nancy and her daughter for this procedure? A. Explain to the daughter that her mother will have to remain still throughout the CT scan B. Determine if the client has any allergies to iodine C. Provide an explanation of relaxation exercises prior to the procedure D. Premedicate the client to decrease pain prior to having the procedure

A. Explain to the daughter that her mother will have to remain still throughout the CT scan. -Because head motion will distort the images, Nancy will have to remain still throughout the procedure. Since Nancy has decreased LOC, she may require head support to accomplish this.

Which condition is considered a modifiable risk factor for a brain attack?

A. High cholesterol levels B. Obesity C. HTN D. Hx of A.fib

Which modifiable risk factors for stroke would be most important for the nurse to include when planning a community education program? A. Hypertension B. Hyerlipidemia C. Alcohol consumption D. Oral contraceptive use

A. Hypertension Hypertension is the single most important modifiable risk factor, but it is still often undetected and inadequately treated. The public is often more aware of hyperlipidemia and oral contraceptive use as risk factors for stroke. Alcohol is also a modifiable risk factor.

A patient with carotid atherosclerosis asks the nurse to describe a carotid endarterectomy. Which response by the nurse is accurate? a. "The obstructing plaque is surgically removed from an artery in the neck." b. "The diseased portion of the artery in the brain is replaced with a synthetic graft." c. "A wire is threaded through an artery in the leg to the clots in the carotid artery and the clots are removed." d. "A catheter with a deflated balloon is positioned at the narrow area, and the balloon is inflated to flatten the plaque."

A. In a carotid endarterectomy, the carotid artery is incised and the plaque is removed. The response beginning, "The diseased portion of the artery in the brain is replaced" describes an arterial graft procedure. The answer beginning, "A catheter with a deflated balloon is positioned at the narrow area" describes an angioplasty. The final response beginning, "A wire is threaded through the artery" describes the mechanical embolus removal in cerebral ischemia (MERCI) procedure.

Computed tomography (CT) of a 68-year-old male patient's head reveals that he has experienced a hemorrhagic stroke. What is the priority nursing intervention in the emergency department? A. Maintenance of patient's airway. B. Positioning to promote cerebral perfusion. C. Control of fluid and electrolyte imbalances. D. Administration of tissue plasminogen activator (tPA)

A. Maintenance of patient's airway. Maintenance of a patent airway is the priority in the acute care of a patient with a hemorrhagic stroke. It supersedes the importance of fluid and electrolyte imbalance and positioning. tPA is contraindicated in hemorrhagic stroke.

Though Nancy's SaO2 potassium level, and telemetry readings are within normal limits for her age, her cardiac output is low. Which nursing interventions would be priority at this time? A. Monitor capillary refill every 2-4 hours B. Monitor level of consciousness C. Monitor vital signs every shift D. Strict intake and output E. Contact physician

A. Monitor capillary refill every 2-4 hours - Decreased cardiac output would affect tissue perfusion, reflected in a capillary refill of greater than 3 seconds. B. Monitor level of consciousness - With a decreased cardiac output, cerebral perfusion will be affected. This can be reflected in a further decreased level of consciousness. D. Strict intake and output - The kidneys use 25% of cardiac output, so when cardiac output is decreased, the kidneys may start failing. Close monitoring is essential. E. Contact physician - The physician needs to be notified regarding decreased cardiac output to decide whether to initiate IV fluids if hypovolemia is an issue and to determine other medical interventions.

Though Nancy's SaO2 potassium level, and telemetry readings are within normal limits for her age, her cardiac output is low. Which nursing interventions would be priority at this time?

A. Monitor level of consciousness With a decreased cardiac output, cerebral perfusion will be affected. This can be reflected in a further decreased level of consciousness. C. Strict intake and output The kidneys use 25% of cardiac output, so when cardiac output is decreased, the kidneys may start failing. Close monitoring is essential. D. Monitor capillary refill every 2-4 hours Decreased cardiac output would affect tissue perfusion, reflected in a capillary refill of greater than 3 seconds. E. Contact physician The physician needs to be notified regarding decreased cardiac output to decide whether to initiate IV fluids if hypovolemia is an issue and to determine other medical interventions.

The patient with diabetes mellitus has had a right-sided stroke. Which nursing intervention should the nurse plan to provide for this patient related to expected manifestations of this stroke? A. Safety measures B. Patience with communication C. Mobility assistance on the right side D. Place food in the left side of patient's mouth

A. Safety measures A patient with a right-sided stroke has spatial-perceptual deficits, tends to minimize problems, has a short attention span, is impulsive, and may have impaired judgment. Safety is the biggest concern for this patient. Hemiplegia occurs on the left side of this patient's body. The patient with a left-sided stroke has hemiplegia on the right, is more likely to have communication problems, and needs mobility assistance on the right side with food placed on the left side if the patient needs to be fed after a swallow evaluation has taken place.

A 40-year-old patient has a ruptured cerebral aneurysm and subarachnoid hemorrhage. Which intervention will be included in the care plan? a. Apply intermittent pneumatic compression stockings. b. Assist to dangle on edge of bed and assess for dizziness. c. Encourage patient to cough and deep breathe every 4 hours. d. Insert an oropharyngeal airway to prevent airway obstruction.

A. The patient with a subarachnoid hemorrhage usually has minimal activity to prevent cerebral vasospasm or further bleeding and is at risk for venous thromboembolism (VTE). Activities such as coughing and sitting up that might increase intracranial pressure (ICP) or decrease cerebral blood flow are avoided. Because there is no indication that the patient is unconscious, an oropharyngeal airway is inappropriate.

For a patient who had a right hemisphere stroke the nurse establishes a nursing diagnosis of a. risk for injury related to denial of deficits and impulsiveness. b. impaired physical mobility related to right-sided hemiplegia. c. impaired verbal communication related to speech-language deficits. d. ineffective coping related to depression and distress about disability.

A. The patient with right-sided brain damage typically denies any deficits and has poor impulse control, leading to risk for injury when the patient attempts activities such as transferring from a bed to a chair. Right-sided brain damage causes left hemiplegia. Left-sided brain damage typically causes language deficits. Left-sided brain damage is associated with depression and distress about the disability.

The nurse observes a student nurse assigned to initiate oral feedings for a 68-year-old woman with an ischemic stroke. The nurse should intervene if she observes the student nurse: A. giving the patient 8 ounces of ice water to swallow. B. telling the patient to perform a chin tuck before swallowing. C. assisting the patient to sit in a chair before feeding the patient. D. assessing cranial nerves IX and X before the patient attempts to eat.

A. giving the patient 8 ounces of ice water to swallow. The majority of patients after a stroke have dysphagia. The gag reflex and swallowing ability (cranial nerves IX and X) should be assessed before the first oral feeding. To assess swallowing ability, the nurse should elevate the head of the bed to an upright position (unless contraindicated) and give the patient a very small amount (not 8 ounces) of crushed ice or ice water to swallow. The patient should remain in a high Fowler's position, preferably in a chair with the head flexed forward, for the feeding and for 30 minutes following.

After receiving change-of-shift report on the following four patients, which patient should the nurse see first? a. A 60-year-old patient with right-sided weakness who has an infusion of tPA prescribed b. A 50-year-old patient who has atrial fibrillation and a new order for warfarin (Coumadin) c. A 40-year-old patient who experienced a transient ischemic attack yesterday who has a dose of aspirin due d. A 30-year-old patient with a subarachnoid hemorrhage 2 days ago who has nimodipine (Nimotop) scheduled

A. tPA needs to be infused within the first few hours after stroke symptoms start in order to be effective in minimizing brain injury. The other medications should also be given as quickly as possible, but timing of the medications is not as critical.

A patient is admitted to the hospital with a left hemiplegia. To determine the size and location and to ascertain whether a stroke is ischemic or hemorrhagic, the nurse anticipates that the health care provider will request a a. CT scan b. lumbar puncture c. cerebral arteriogram d. positron emission tomography (PET)

A: CT scan- A CT scan is the most commonly used diagnostic test to determine the size and location of the lesion and to differentiate a thrombotic stroke from a hemorrhagic stroke. Positron emission tomography (PET) will show the metabolic activity of the brain and provide a depiction of the extent of tissue damage after a stroke. Lumbar punctures are not performed routinely because of the chance of increased intracranial pressure causing herniation. Cerebral arteriograms are invasive and may dislodge an embolism or cause further hemorrhage; they are performed only when no other test can provide the needed information.

During the acute phase of a stroke, the nurse assesses the patient's vital signs and neurologic status every 4 hours. A cardiovascular sign that the nurse would see as the body attempts to increase cerebral blood flow is a. hypertension b. fluid overload c. cardiac dysrhythmias d. S3 and S4 heart sounds

A: Hypertension- The body responds to the vasopasm and a decreased circulation to the brain that occurs with a stroke by increasing the BP, frequently resulting in hypertension. The other options are important cardiovascular factors to assess, but they do not result from impaired cerebral blood flow.

A newly admitted patient who has suffered a right sided brain stroke has a nursing diagnosis of disturbed visual sensory perception related to homonymous hemianopsia. Early in the care of the patient, the nurse should a. place objects on the right side within the patient's field of vision b. approach the patient from the left side to encourage the patient to turn the head c. place objects on the patient's left side to assess the patient's ability to compensate d. patch the affected eye to encourage the patient to turn the head to scan the environment

A: Place objects on the right side within the patient's field of vision- the presence of homonymous hemianopia in a patient with right-hemisphere brain damage causes a loss of vision in the left field. Early in the care of the patient, objects should be placed on the right side of the patient in the field of vision, and the nurse should approach the patient from the right side. Later in treatment, patients should be taught to turn the head and scan the environment and should be approached from the affected side to encourage head turning. Eye patches are used if patients have diplopia (double vision).

Four days following a stroke, a patient is to start oral fluids and feedings. Before feeding the patient, the nurse should first a. check the patient's gag reflex b. order a soft diet for the patient c. raise the head of the bed to sitting position d. evaluate the patient's ability to swallow small sips of ice water

A: check the patient's gag reflex- the first step in providing oral feedings for a patient with a stroke is ensuring that the patient has an intact gag reflex because oral feedings will not be provided if gag reflex is impaired. The nurse should then evaluate the patient's ability to swallow ice chips or ice water after placing the patient in an upright position

1. A 63-year-old patient who began experiencing right arm and leg weakness is admitted to the emergency department. In which order will the nurse implement these actions included in the stroke protocol? (Put a comma and a space between each answer choice [A, B, C, D].) a. Obtain computed tomography (CT) scan without contrast. b. Infuse tissue plasminogen activator (tPA). c. Administer oxygen to keep O2 saturation >95%. d. Use National Institute of Health Stroke Scale to assess patient.

ANS: C, D, A, B The initial actions should be those that help with airway, breathing, and circulation. Baseline neurologic assessments should be done next. A CT scan will be needed to rule out hemorrhagic stroke before tPA can be administered.

A 63-year-old patient who began experiencing right arm and leg weakness is admitted to the emergency department. In which order will the nurse implement these actions included in the stroke protocol? (Put a comma and a space between each answer choice [A, B, C, D].) a. Obtain computed tomography (CT) scan without contrast. b. Infuse tissue plasminogen activator (tPA). c. Administer oxygen to keep O2 saturation >95%. d. Use National Institute of Health Stroke Scale to assess patient.

ANS: C, D, A, B The initial actions should be those that help with airway, breathing, and circulation. Baseline neurologic assessments should be done next. A CT scan will be needed to rule out hemorrhagic stroke before tPA can be administered. DIF: Cognitive Level: Apply (application) REF: 1401 | 1404 | 1398 OBJ: Special Questions: Prioritization TOP: Nursing Process: Implementation MSC: NCLEX: Physiological Integrity

24. Which posthospital option should the nurse encourage a patient to do when recovering from a CVA to provide the most comprehensive assistance? a. Transfer to a rehabilitation center. b. Discharge to home with scheduled visits from home health care nurses. c. Discharge to home with scheduled visits from a physical therapist. d. Discharge to home with scheduled visits from an occupational therapist.

ANS: A A rehabilitation center with all modalities of support (e.g., physical therapy, occupational therapy, speech therapy, simulated home environments) is obviously the best option.

9. A patient in the acute phase of a CVA who has been speaking distinctly begins to speak indistinctly and only with great effort but still coherent. What should this nurse determine when assessing this patient? a. Stroke in evolution with dysarthria b. Lacunar stroke with fluent aphasia c. Complete stroke with global aphasia d. Stroke in evolution with dyspraxia

ANS: A As symptoms worsen, the CVA is still evolving. Speech that is coherent but difficult is dysarthria rather than any type of aphasia. Dyspraxia is a motor impairment, not a speech impairment.

6. The nurse is assessing a client with a history of absence seizures. Which clinical manifestation does the nurse assess for? a. Automatisms b. Intermittent rigidity c. Sudden loss of muscle tone d. Brief jerking of the extremities

ANS: A Automatisms are characteristic of absence seizures. These behaviors consist of lip smacking, patting, and picking at clothing. The other manifestations do not correlate with absence seizures.

21. A nurse delegates care for a client with Parkinson disease to an unlicensed assistive personnel (UAP). Which statement should the nurse include when delegating this client's care? a. "Allow the client to be as independent as possible with activities." b. "Assist the client with frequent and meticulous oral care." c. "Assess the client's ability to eat and swallow before each meal." d. "Schedule appointments early in the morning to ensure rest in the afternoon."

ANS: A Clients with Parkinson disease do not move as quickly and can have functional problems. The client should be encouraged to be as independent as possible and provided time to perform activities without rushing. Although oral care is important for all clients, instructing the UAP to provide frequent and meticulous oral is not a priority for this client. This statement would be a priority if the client was immune-compromised or NPO. The nurse should assess the client's ability to eat and swallow; this should not be delegated. Appointments and activities should not be scheduled early in the morning because this may cause the client to be rushed and discourage the client from wanting to participate in activities of daily living.

4. The nurse is assessing a client with a cluster headache. Which clinical manifestation does the nurse expect to find? a. Ipsilateral tearing of the eye b. Exophthalmos c. Abrupt loss of consciousness d. Neck and shoulder tenderness

ANS: A Cluster headache is usually accompanied by ipsilateral tearing, rhinorrhea or nasal congestion, ptosis, eyelid edema, facial sweating, and miosis. The other manifestations are not associated with cluster headaches.

9. A female patient who had a stroke 24 hours ago has expressive aphasia. The nurse identifies the nursing diagnosis of impaired verbal communication. An appropriate nursing intervention to help the patient communicate is to a. ask questions that the patient can answer with "yes" or "no." b. develop a list of words that the patient can read and practice reciting. c. have the patient practice her facial and tongue exercises with a mirror. d. prevent embarrassing the patient by answering for her if she does not respond.

ANS: A Communication will be facilitated and less frustrating to the patient when questions that require a "yes" or "no" response are used. When the language areas of the brain are injured, the patient might not be able to read or recite words, which will frustrate the patient without improving communication. Expressive aphasia is caused by damage to the language areas of the brain, not by the areas that control the motor aspects of speech. The nurse should allow time for the patient to respond.

A female patient who had a stroke 24 hours ago has expressive aphasia. The nurse identifies the nursing diagnosis of impaired verbal communication. An appropriate nursing intervention to help the patient communicate is to a. ask questions that the patient can answer with "yes" or "no." b. develop a list of words that the patient can read and practice reciting. c. have the patient practice her facial and tongue exercises with a mirror. d. prevent embarrassing the patient by answering for her if she does not respond.

ANS: A Communication will be facilitated and less frustrating to the patient when questions that require a "yes" or "no" response are used. When the language areas of the brain are injured, the patient might not be able to read or recite words, which will frustrate the patient without improving communication. Expressive aphasia is caused by damage to the language areas of the brain, not by the areas that control the motor aspects of speech. The nurse should allow time for the patient to respond. DIF: Cognitive Level: Apply (application) REF: 1407 TOP: Nursing Process: Implementation MSC: NCLEX: Physiological Integrity

4. A nurse is updating a teaching plan for a patient who sustained a TIA. What should the nurse be sure to include? a. Daily aspirin dose b. Long rest periods daily c. Reduction of fluid intake to 800 mL/day d. High-carbohydrate diet

ANS: A Daily aspirin reduces platelet aggregation and may prevent another attack. Reductions of fluid and long rest periods encourage clot formation.

17. The nurse is caring for a hospitalized client with Alzheimer's disease who has a history of agitation. Which intervention does the nurse implement to help prevent agitation and aggressive behavior in this client? a. Provide undisturbed sleep. b. Orient the client to reality. c. Leave the television turned on. d. Administer hypnotic drugs as needed.

ANS: A Fatigue from disturbed sleep increases confusion and behavioral manifestations, such as aggression and agitation. Reality orientation is inappropriate for clients in a later stage of the disease. Constant noise from the TV most likely would agitate the client. Sedation should be used as a last resort.

23. Which instruction is most helpful in teaching the family and patient who is in the rehabilitation phase after a CVA about altered sensation? a. Make frequent assessments for signs of pressure or injury. b. Use the affected side in supporting the patient in ambulation and transfer to stimulate better sensation. c. Apply ice packs to the affected limbs to encourage a return of sensation. d. Apply a heating pad to the affected limbs to increase circulation.

ANS: A Frequent assessment using the National Institutes of Health Stroke Scale will allow early detection. The use of hot or cold applications and using the affected limbs in transfer or ambulation may cause injury.

1. A patient has weakness on the right side and impaired reasoning after having a cerebrovascular accident (CVA). What part of the brain is affected? a. Left hemisphere of the cerebrum b. Right hemisphere of the cerebrum c. Left cerebellum d. Right cerebellum

ANS: A Impaired motor strength on the right side in conjunction with impaired reasoning indicates a lesion in the left hemisphere of the cerebrum. The cerebellum controls balance and is not contralateral.

6. A patient with carotid atherosclerosis asks the nurse to describe a carotid endarterectomy. Which response by the nurse is accurate? a. "The obstructing plaque is surgically removed from an artery in the neck." b. "The diseased portion of the artery in the brain is replaced with a synthetic graft." c. "A wire is threaded through an artery in the leg to the clots in the carotid artery and the clots are removed." d. "A catheter with a deflated balloon is positioned at the narrow area, and the balloon is inflated to flatten the plaque."

ANS: A In a carotid endarterectomy, the carotid artery is incised and the plaque is removed. The response beginning, "The diseased portion of the artery in the brain is replaced" describes an arterial graft procedure. The answer beginning, "A catheter with a deflated balloon is positioned at the narrow area" describes an angioplasty. The final response beginning, "A wire is threaded through the artery" describes the mechanical embolus removal in cerebral ischemia (MERCI) procedure.

A patient with carotid atherosclerosis asks the nurse to describe a carotid endarterectomy. Which response by the nurse is accurate? a. "The obstructing plaque is surgically removed from an artery in the neck." b. "The diseased portion of the artery in the brain is replaced with a synthetic graft." c. "A wire is threaded through an artery in the leg to the clots in the carotid artery and the clots are removed." d. "A catheter with a deflated balloon is positioned at the narrow area, and the balloon is inflated to flatten the plaque."

ANS: A In a carotid endarterectomy, the carotid artery is incised and the plaque is removed. The response beginning, "The diseased portion of the artery in the brain is replaced" describes an arterial graft procedure. The answer beginning, "A catheter with a deflated balloon is positioned at the narrow area" describes an angioplasty. The final response beginning, "A wire is threaded through the artery" describes the mechanical embolus removal in cerebral ischemia (MERCI) procedure. DIF: Cognitive Level: Understand (comprehension) REF: 1397 TOP: Nursing Process: Implementation MSC: NCLEX: Physiological Integrity

9. A nurse obtains a focused health history for a client who is suspected of having bacterial meningitis. Which question should the nurse ask? a. "Do you live in a crowded residence?" b. "When was your last tetanus vaccination?" c. "Have you had any viral infections recently?" d. "Have you traveled out of the country in the last month?"

ANS: A Meningococcal meningitis tends to occur in multiple outbreaks. It is most likely to occur in areas of high-density population, such as college dormitories, prisons, and military barracks. A tetanus vaccination would not place the client at increased risk for meningitis or protect the client from meningitis. A viral infection would not lead to bacterial meningitis but could lead to viral meningitis. Simply knowing if the client traveled out of the country does not provide enough information. The nurse should ask about travel to specific countries in which the disease is common, for example, sub-Saharan Africa.

12. The nurse is taking the health history of a client suspected of having bacterial meningitis. Which question is most important for the nurse to ask? a. "Do you live in a crowded residence?" b. "When was your last tetanus vaccination?" c. "Have you had any viral infections recently?" d. "Have you traveled out of the country in the last month?"

ANS: A Meningococcal meningitis tends to occur in outbreaks. It is most likely to occur in areas of high-density population, such as college dormitories, prisons, and military barracks. The other questions do not identify risk factors for bacterial meningitis.

12. A patient who has sustained a hemorrhagic stroke is placed on a protocol of 60 mg of calcium channel blocker (nimodipine) every 4 hours. The patient's pulse is 82 beats/min before the administration of the prescribed dose. Which action should the nurse implement? a. Give the full dose as prescribed without further assessment. b. Omit the dose, recording the pulse rate as the rationale. c. Delay the dose until the pulse is below 60 beats/min. d. Give half of the prescribed dose (30 mg).

ANS: A The dose should be given; it would be held only if the pulse is below 60 beats/min. Assessments should be made regarding BP, urine output, and edema.

19. A client is prescribed levetiracetam (Keppra). Which laboratory tests does the nurse monitor for potential adverse effects of this medication? a. Serum electrolyte levels b. Kidney function tests c. Complete blood cell count d. Antinuclear antibodies

ANS: B Adverse effects of levetiracetam (Keppra) include coordination problems and renal toxicity. The other laboratory tests are not affected by levetiracetam.

13. A 40-year-old patient has a ruptured cerebral aneurysm and subarachnoid hemorrhage. Which intervention will be included in the care plan? a. Apply intermittent pneumatic compression stockings. b. Assist to dangle on edge of bed and assess for dizziness. c. Encourage patient to cough and deep breathe every 4 hours. d. Insert an oropharyngeal airway to prevent airway obstruction.

ANS: A The patient with a subarachnoid hemorrhage usually has minimal activity to prevent cerebral vasospasm or further bleeding and is at risk for venous thromboembolism (VTE). Activities such as coughing and sitting up that might increase intracranial pressure (ICP) or decrease cerebral blood flow are avoided. Because there is no indication that the patient is unconscious, an oropharyngeal airway is inappropriate.

A 40-year-old patient has a ruptured cerebral aneurysm and subarachnoid hemorrhage. Which intervention will be included in the care plan? a. Apply intermittent pneumatic compression stockings. b. Assist to dangle on edge of bed and assess for dizziness. c. Encourage patient to cough and deep breathe every 4 hours. d. Insert an oropharyngeal airway to prevent airway obstruction.

ANS: A The patient with a subarachnoid hemorrhage usually has minimal activity to prevent cerebral vasospasm or further bleeding and is at risk for venous thromboembolism (VTE). Activities such as coughing and sitting up that might increase intracranial pressure (ICP) or decrease cerebral blood flow are avoided. Because there is no indication that the patient is unconscious, an oropharyngeal airway is inappropriate. DIF: Cognitive Level: Apply (application) REF: 1405 TOP: Nursing Process: Planning MSC: NCLEX: Physiological Integrity

10. For a patient who had a right hemisphere stroke the nurse establishes a nursing diagnosis of a. risk for injury related to denial of deficits and impulsiveness. b. impaired physical mobility related to right-sided hemiplegia. c. impaired verbal communication related to speech-language deficits. d. ineffective coping related to depression and distress about disability.

ANS: A The patient with right-sided brain damage typically denies any deficits and has poor impulse control, leading to risk for injury when the patient attempts activities such as transferring from a bed to a chair. Right-sided brain damage causes left hemiplegia. Left-sided brain damage typically causes language deficits. Left-sided brain damage is associated with depression and distress about the disability.

For a patient who had a right hemisphere stroke the nurse establishes a nursing diagnosis of a. risk for injury related to denial of deficits and impulsiveness. b. impaired physical mobility related to right-sided hemiplegia. c. impaired verbal communication related to speech-language deficits. d. ineffective coping related to depression and distress about disability.

ANS: A The patient with right-sided brain damage typically denies any deficits and has poor impulse control, leading to risk for injury when the patient attempts activities such as transferring from a bed to a chair. Right-sided brain damage causes left hemiplegia. Left-sided brain damage typically causes language deficits. Left-sided brain damage is associated with depression and distress about the disability. DIF: Cognitive Level: Apply (application) REF: 1407 TOP: Nursing Process: Diagnosis MSC: NCLEX: Physiological Integrity

18. A hospitalized client with late-stage Alzheimer's disease says that breakfast has not been served. The nurse witnessed the client eating breakfast earlier. Which statement made to this client is an example of validation therapy? a. "I see you are still hungry. I will get you some toast." b. "You are confused about mealtimes this morning." c. "You ate your breakfast 30 minutes ago." d. "You look tired. Maybe a nap will help."

ANS: A Use of validation therapy involves acknowledgment of the client's feelings and concerns. This technique has proved more effective in later stages of the disease, when using reality orientation only increases agitation. Telling the client that he or she already ate breakfast may agitate the client. The other statements do not validate the client's concerns.

14. A nurse witnesses a client with late-stage Alzheimer's disease eat breakfast. Afterward the client states, "I am hungry and want breakfast." How should the nurse respond? a. "I see you are still hungry. I will get you some toast." b. "You ate your breakfast 30 minutes ago." c. "It appears you are confused this morning." d. "Your family will be here soon. Let's get you dressed."

ANS: A Use of validation therapy with clients who have Alzheimer's disease involves acknowledgment of the client's feelings and concerns. This technique has proved more effective in later stages of the disease, when using reality orientation only increases agitation. Telling the client that he or she already ate breakfast may agitate the client. The other statements do not validate the client's concerns.

10. Several days after a CVA, a patient's family asks a nurse if tissue plasminogen activator (tPA) is a drug therapy option now. The nurse's response is based on the knowledge that this drug must be used within how many hours after the onset of symptoms? a. 3 b. 5 c. 10 d. 24

ANS: A tPA is to be given within 3 hours of the onset of symptoms per the U.S. Food and Drug Administration's guidelines. In some special treatment centers this drug is given intravenously up to 6 hours after the stroke.

26. After receiving change-of-shift report on the following four patients, which patient should the nurse see first? a. A 60-year-old patient with right-sided weakness who has an infusion of tPA prescribed b. A 50-year-old patient who has atrial fibrillation and a new order for warfarin (Coumadin) c. A 40-year-old patient who experienced a transient ischemic attack yesterday who has a dose of aspirin due d. A 30-year-old patient with a subarachnoid hemorrhage 2 days ago who has nimodipine (Nimotop) scheduled

ANS: A tPA needs to be infused within the first few hours after stroke symptoms start in order to be effective in minimizing brain injury. The other medications should also be given as quickly as possible, but timing of the medications is not as critical.

After receiving change-of-shift report on the following four patients, which patient should the nurse see first? a. A 60-year-old patient with right-sided weakness who has an infusion of tPA prescribed b. A 50-year-old patient who has atrial fibrillation and a new order for warfarin (Coumadin) c. A 40-year-old patient who experienced a transient ischemic attack yesterday who has a dose of aspirin due d. A 30-year-old patient with a subarachnoid hemorrhage 2 days ago who has nimodipine (Nimotop) scheduled

ANS: A tPA needs to be infused within the first few hours after stroke symptoms start in order to be effective in minimizing brain injury. The other medications should also be given as quickly as possible, but timing of the medications is not as critical. DIF: Cognitive Level: Apply (application) REF: 1398 OBJ: Special Questions: Prioritization; Multiple Patients TOP: Nursing Process: Implementation MSC: NCLEX: Safe and Effective Care Environment

7. A nurse assesses clients on a medical-surgical unit. Which clients should the nurse identify as at risk for secondary seizures? (Select all that apply.) a. A 26-year-old woman with a left temporal brain tumor b. A 38-year-old male client in an alcohol withdrawal program c. A 42-year-old football player with a traumatic brain injury d. A 66-year-old female client with multiple sclerosis e. A 72-year-old man with chronic obstructive pulmonary disease

ANS: A, B, C Clients at risk for secondary seizures include those with a brain lesion from a tumor or trauma, and those who are experiencing a metabolic disorder, acute alcohol withdrawal, electrolyte disturbances, and high fever. Clients with a history of stroke, heart disease, and substance abuse are also at risk. Clients with multiple sclerosis or chronic obstructive pulmonary disease are not at risk for secondary seizures.

31. Which home modifications will support rehabilitation for a patient who had a stroke? (Select all that apply.) a. Raised commode seat b. Provision of a seat in the shower c. Availability of soft, low chairs d. Bathtub hand rails e. Bright-colored scatter rugs

ANS: A, B, D A raised commode seat, a seat in the shower, and bathtub rails assist the patient who is recovering from a stroke with self-care. Low chairs are difficult to manage, and scatter rugs pose a hazard for falls.

4. A nurse assesses a client who is experiencing a cluster headache. Which clinical manifestations should the nurse expect to find? (Select all that apply.) a. Ipsilateral tearing of the eye b. Miosis c. Abrupt loss of consciousness d. Neck and shoulder tenderness e. Nasal congestion f. Exophthalmos

ANS: A, B, E Cluster headache is usually accompanied by ipsilateral tearing, miosis, rhinorrhea or nasal congestion, ptosis, eyelid edema, and facial sweating. Abrupt loss of consciousness, neck and shoulder tenderness, and exophthalmos are not associated with cluster headaches.

9. A nurse is caring for a client with meningitis. Which laboratory values should the nurse monitor to identify potential complications of this disorder? (Select all that apply.) a. Sodium level b. Liver enzymes c. Clotting factors d. Cardiac enzymes e. Creatinine level

ANS: A, C Inflammation associated with meningitis can stimulate the hypothalamus and result in excessive production of antidiuretic hormone. The nurse should monitor sodium levels for early identification of syndrome of inappropriate antidiuretic hormone. A systemic inflammatory response (SIR) can also occur with meningitis. A SIR can result in a coagulopathy that leads to disseminated intravascular coagulation. The nurse should monitor clotting factors to identify this complication. The other laboratory values are not specific to complications of meningitis.

3. A nurse evaluates the results of diagnostic tests on a client's cerebrospinal fluid (CSF). Which fluid results alerts the nurse to possible viral meningitis? (Select all that apply.) a. Clear b. Cloudy c. Increased protein level d. Normal glucose level e. Bacterial organisms present f. Increased white blood cells

ANS: A, C, D In viral meningitis, CSF fluid is clear, protein levels are slightly increased, and glucose levels are normal. Viral meningitis does not cause cloudiness or increased turbidity of CSF. In bacterial meningitis, the presence of bacteria and white blood cells causes the fluid to be cloudy.

32. What causes the 3% of strokes known to occur in persons younger than 45 years of age? (Select all that apply.) a. Drug abuse b. Alcohol abuse c. Birth control pills d. Sickle cell anemia e. Hemophilia

ANS: A, C, D Strokes in younger people are caused by drug abuse, birth control pills, sickle cell anemia, leukemia, atrial fibrillation, and rheumatic fever. Alcohol abuse and hemophilia do not have a causative role in stroke.

3. The nurse is assessing the results of diagnostic tests on a client's cerebrospinal fluid (CSF). Which values and observations does the nurse correlate as most indicative of viral meningitis? (Select all that apply.) a. Clear b. Cloudy c. Normal protein level d. Increased protein level e. Normal glucose level f. Decreased glucose level

ANS: A, D, E Viral meningitis does not cause cloudiness or increased turbidity of CSF. Protein levels are slightly increased, and glucose levels are normal. In bacterial meningitis, the presence of bacteria and white blood cells causes the fluid to be cloudy.

1. A nurse plans care for a client with epilepsy who is admitted to the hospital. Which interventions should the nurse include in this client's plan of care? (Select all that apply.) a. Have suction equipment at the bedside. b. Place a padded tongue blade at the bedside. c. Permit only clear oral fluids. d. Keep bed rails up at all times. e. Maintain the client on strict bedrest. f. Ensure that the client has IV access.

ANS: A, D, F Oxygen and suctioning equipment with an airway must be readily available. The bed rails should be up at all times while the client is in the bed to prevent injury from a fall if the client has a seizure. If the client does not have an IV access, insert a saline lock, especially for those clients who are at significant risk for generalized tonic- clonic seizures. The saline lock provides ready access if IV drug therapy must be given to stop the seizure. Padded tongue blades may pose a danger to the client during a seizure and should not be used. Dietary restrictions and strict bedrest are not interventions associated with epilepsy. The client should be encouraged to eat a well-balanced diet and ambulate while in the hospital.

1. The nurse is planning care for a client with epilepsy. Which precautions does the nurse implement to ensure the safety of the client while in the hospital? (Select all that apply.) a. Have suction equipment at the bedside. b. Place a padded tongue at the bedside. c. Permit only clear oral fluids. d. Keep bed rails up at all times. e. Maintain the client on strict bedrest. f. Ensure that the client has IV access.

ANS: A, D, F The bed rails should be up at all times while the client is in the bed to prevent injury from a fall if the client has a seizure. Padded tongue blades may pose a danger to the client during a seizure. Be sure that oxygen and suctioning equipment with an airway are readily available. If the client does not have an IV access, insert a saline lock, especially for those clients who are at significant risk for generalized tonicclonic seizures. The saline lock provides ready access if IV drug therapy must be given to stop the seizure.

30. Which patients with CVAs are considered candidates for treatment with tPA? (Select all that apply.) a. A 62-year-old construction worker who had a subdural hematoma 6 months earlier b. A 58-year-old executive with a bleeding ulcer c. A 44-year-old individual who had a seizure at the onset of a stroke d. A 40-year-old individual who is taking warfarin (Coumadin) and has an INR of 2.5 e. A 19-year-old young adult with leukemia with a platelet count of 200,000

ANS: A, E The criteria for exclusion are a head injury within the last 3 months, a platelet count less than 100,000, active gastrointestinal bleeding, current treatment with an anticoagulant, and a seizure noted at the time of the CVA.

15. A nurse assesses a client after administering prescribed levetiracetam (Keppra). Which laboratory tests should the nurse monitor for potential adverse effects of this medication? a. Serum electrolyte levels b. Kidney function tests c. Complete blood cell count d. Antinuclear antibodies

ANS: B Adverse effects of levetiracetam include coordination problems and renal toxicity. The other laboratory tests are not affected by levetiracetam.

22. The nurse is assessing a client with Huntington's disease. Which motor changes does the nurse monitor for in this client? a. Shuffling gait b. Jerky hand movements c. Continuous chewing motions d. Tremors of the hands during fine motor tasks

ANS: B An imbalance between excitatory and inhibitory neurotransmitters leads to uninhibited motor movements, such as brisk, jerky, purposeless movements of the hands, face, tongue, and legs. Shuffling gait, continuous chewing motions, and tremors are associated with Parkinson's disease.

18. A nurse assesses a client with Huntington disease. Which motor changes should the nurse monitor for in this client? a. Shuffling gait b. Jerky hand movements c. Continuous chewing motions d. Tremors of the hands

ANS: B An imbalance between excitatory and inhibitory neurotransmitters leads to uninhibited motor movements, such as brisk, jerky, purposeless movements of the hands, face, tongue, and legs. Shuffling gait, continuous chewing motions, and tremors are associated with Parkinson disease.

1. The nurse is caring for a client experiencing migraine headaches who is receiving a beta blocker to help manage this disorder. When preparing a teaching plan, which instruction does the nurse plan to provide? a. "Take this drug only when you have prodromal symptoms indicating the onset of a migraine headache." b. "Take this drug as ordered, even when feeling well, to prevent vascular changes associated with migraine headaches." c. "This drug will relieve the pain during the aura phase soon after a headache has started." d. "This medication will have no effect on your heart rate or blood pressure because you are taking it for migraines."

ANS: B Beta blockers are prescribed as prophylactic treatment to prevent the vascular changes that initiate migraine headaches. Heart rate and blood pressure will also be affected, and the client should monitor these side effects. The other responses do not discuss an appropriate use of the medication.

1. A nurse is teaching a client who experiences migraine headaches and is prescribed a beta blocker. Which statement should the nurse include in this client's teaching? a. "Take this drug only when you have prodromal symptoms indicating the onset of a migraine headache." b. "Take this drug as ordered, even when feeling well, to prevent vascular changes associated with migraine headaches." c. "This drug will relieve the pain during the aura phase soon after a headache has started." d. "This medication will have no effect on your heart rate or blood pressure because you are taking it for migraines."

ANS: B Beta blockers are prescribed as prophylactic treatment to prevent the vascular changes that initiate migraine headaches. Heart rate and blood pressure will also be affected, and the client should monitor these side effects. The other responses do not discuss appropriate uses of the medication.

11. A nurse explains that a lumbar puncture is most helpful as a diagnostic tool for a new patient who has had a CVA. What would this diagnostic test help determine regarding the stroke? a. It is lacunar. b. It is hemorrhagic or embolic. c. It is complete or in evolution. d. It will result in paralysis.

ANS: B Blood in the spinal fluid indicates a hemorrhagic stroke and will help direct medical protocol in the subsequent treatment.

5. A patient recovering from a CVA asks the purpose of the warfarin (Coumadin). What is the best response by the nurse regarding the purpose of Coumadin? a. Dissolves the clot. b. Prevents the formation of new clots. c. Dilates the vessels to improve blood flow. d. Suppresses the formation of platelets.

ANS: B Coumadin and heparin prevent more clots rather than dissolving them. Coumadin has no effect on vasodilation or blood cell production.

16. Several weeks after a stroke, a 50-year-old male patient has impaired awareness of bladder fullness, resulting in urinary incontinence. Which nursing intervention will be best to include in the initial plan for an effective bladder training program? a. Limit fluid intake to 1200 mL daily to reduce urine volume. b. Assist the patient onto the bedside commode every 2 hours. c. Perform intermittent catheterization after each voiding to check for residual urine. d. Use an external "condom" catheter to protect the skin and prevent embarrassment.

ANS: B Developing a regular voiding schedule will prevent incontinence and may increase patient awareness of a full bladder. A 1200 mL fluid restriction may lead to dehydration. Intermittent catheterization and use of a condom catheter are appropriate in the acute phase of stroke, but should not be considered solutions for long-term management because of the risks for urinary tract infection (UTI) and skin breakdown.

Several weeks after a stroke, a 50-year-old male patient has impaired awareness of bladder fullness, resulting in urinary incontinence. Which nursing intervention will be best to include in the initial plan for an effective bladder training program? a. Limit fluid intake to 1200 mL daily to reduce urine volume. b. Assist the patient onto the bedside commode every 2 hours. c. Perform intermittent catheterization after each voiding to check for residual urine. d. Use an external "condom" catheter to protect the skin and prevent embarrassment.

ANS: B Developing a regular voiding schedule will prevent incontinence and may increase patient awareness of a full bladder. A 1200 mL fluid restriction may lead to dehydration. Intermittent catheterization and use of a condom catheter are appropriate in the acute phase of stroke, but should not be considered solutions for long-term management because of the risks for urinary tract infection (UTI) and skin breakdown. DIF: Cognitive Level: Apply (application) REF: 1406 TOP: Nursing Process: Planning MSC: NCLEX: Physiological Integrity

18. A patient in the clinic reports a recent episode of dysphasia and left-sided weakness at home that resolved after 2 hours. The nurse will anticipate teaching the patient about a. alteplase (tPA). b. aspirin (Ecotrin). c. warfarin (Coumadin). d. nimodipine (Nimotop).

ANS: B Following a transient ischemic attack (TIA), patients typically are started on medications such as aspirin to inhibit platelet function and decrease stroke risk. tPA is used for acute ischemic stroke. Coumadin is usually used for patients with atrial fibrillation. Nimodipine is used to prevent cerebral vasospasm after a subarachnoid hemorrhage.

A patient in the clinic reports a recent episode of dysphasia and left-sided weakness at home that resolved after 2 hours. The nurse will anticipate teaching the patient about a. alteplase (tPA). b. aspirin (Ecotrin). c. warfarin (Coumadin). d. nimodipine (Nimotop).

ANS: B Following a transient ischemic attack (TIA), patients typically are started on medications such as aspirin to inhibit platelet function and decrease stroke risk. tPA is used for acute ischemic stroke. Coumadin is usually used for patients with atrial fibrillation. Nimodipine is used to prevent cerebral vasospasm after a subarachnoid hemorrhage. DIF: Cognitive Level: Apply (application) REF: 1396 TOP: Nursing Process: Planning MSC: NCLEX: Physiological Integrity

6. A nurse cares for a client who is experiencing status epilepticus. Which prescribed medication should the nurse prepare to administer? a. Atenolol (Tenormin) b. Lorazepam (Ativan) c. Phenytoin (Dilantin) d. Lisinopril (Prinivil)

ANS: B Initially, intravenous lorazepam is administered to stop motor movements. This is followed by the administration of phenytoin. Atenolol, a beta blocker, and lisinopril, an angiotensin-converting enzyme inhibitor, are not administered for seizure activity. These medications are typically administered for hypertension and heart failure.

8. A client is actively experiencing status epilepticus. Which prescribed medication does the nurse prepare to administer? a. Atropine b. Lorazepam (Ativan) c. Phenytoin (Dilantin) d. Morphine sulfate

ANS: B Initially, intravenous lorazepam is administered to stop motor movements. This is followed by the administration of phenytoin. Atropine and morphine are not administered for seizure activity.

19. A patient with homonymous hemianopsia is in the rehabilitation phase of a CVA. When arranging this patient's environment where should the nurse assure persons approaching and important items are visible and available? a. Unaffected side b. Affected side c. Direct front d. Either side

ANS: B Making the patient scan the affected side helps stimulate the return of normal function in the rehabilitation phase.

13. During the acute CVA phase, a risk for falls related to paralysis is present. Which intervention best protects the patient from injury? a. Keep the bed in a high position for ease of nursing care. b. Keep the side rails up, according to agency policy. c. Assess vision deficit related to ptosis. d. Monitor the condition every 2 hours.

ANS: B Rails keep patients in bed. The bed should be low, monitoring the patient should be more frequent than every 2 hours, and visual assessment is not directly related to fall prevention.

17. A patient in the acute phase of an embolic CVA has an order for 400 units of heparin per hour IV. The heparin is in a solution of 5000 units/100 mL normal saline (NS). The nurse should set the electronic IV monitor at how many milliliters per hour? a. 6 b. 8 c. 10 d. 16

ANS: B Regardless of the method of calculation, 50 units of heparin are in each milliliter of the solution; 8 mL/hr delivers 400 units (5000 units ÷ 100 mL NS = 50 units/mL. 400 units ÷ 50 units/mL = 8 mL).

4. A nurse assesses a client with a history of epilepsy who experiences stiffening of the muscles of the arms and legs, followed by an immediate loss of consciousness and jerking of all extremities. How should the nurse document this activity? a. Atonic seizure b. Tonic-clonic seizure c. Myoclonic seizure d. Absence seizure

ANS: B Seizure activity that begins with stiffening of the arms and legs, followed by loss of consciousness and jerking of all extremities, is characteristic of a tonic-clonic seizure. An atonic seizure presents as a sudden loss of muscle tone followed by postictal confusion. A myoclonic seizure presents with a brief jerking or stiffening of extremities that may occur singly or in groups. Absence seizures present with automatisms, and the client is unaware of his or her environment.

16. A nurse cares for a client with advanced Alzheimer's disease. The client's caregiver states, "She is always wandering off. What can I do to manage this restless behavior?" How should the nurse respond? a. "This is a sign of fatigue. The client would benefit from a daily nap." b. "Engage the client in scheduled activities throughout the day." c. "It sounds like this is difficult for you. I will consult the social worker." d. "The provider can prescribe a mild sedative for restlessness."

ANS: B Several strategies may be used to cope with restlessness and wandering. One strategy is to engage the client in structured activities. Another is to take the client for frequent walks. Daily naps and a mild sedative will not be as effective in the management of restless behavior. Consulting the social worker does not address the caregiver's concern.

20. The caregiver of a client with advanced Alzheimer's disease states, "She is always wandering off. What can I do to manage this restless behavior?" How does the nurse respond? a. "Allow for a 45-minute daytime nap." b. "Take the client for frequent walks throughout the day." c. "Using a Geri-chair may decrease agitation." d. "Give a mild sedative during periods of restlessness."

ANS: B Several strategies may be used to cope with restlessness and wandering. Taking the client for frequent walks may decrease restless behavior. Another strategy is to engage the client in structured activities. The other options would not be as helpful.

27. The nurse is caring for a patient who has just returned after having left carotid artery angioplasty and stenting. Which assessment information is of most concern to the nurse? a. The pulse rate is 102 beats/min. b. The patient has difficulty speaking. c. The blood pressure is 144/86 mm Hg. d. There are fine crackles at the lung bases.

ANS: B Small emboli can occur during carotid artery angioplasty and stenting, and the aphasia indicates a possible stroke during the procedure. Slightly elevated pulse rate and blood pressure are not unusual because of anxiety associated with the procedure. Fine crackles at the lung bases may indicate atelectasis caused by immobility during the procedure. The nurse should have the patient take some deep breaths.

The nurse is caring for a patient who has just returned after having left carotid artery angioplasty and stenting. Which assessment information is of most concern to the nurse? a. The pulse rate is 102 beats/min. b. The patient has difficulty speaking. c. The blood pressure is 144/86 mm Hg. d. There are fine crackles at the lung bases.

ANS: B Small emboli can occur during carotid artery angioplasty and stenting, and the aphasia indicates a possible stroke during the procedure. Slightly elevated pulse rate and blood pressure are not unusual because of anxiety associated with the procedure. Fine crackles at the lung bases may indicate atelectasis caused by immobility during the procedure. The nurse should have the patient take some deep breaths. DIF: Cognitive Level: Apply (application) REF: 1394 OBJ: Special Questions: Prioritization TOP: Nursing Process: Assessment MSC: NCLEX: Physiological Integrity

3. A nurse obtains a health history on a client prior to administering prescribed sumatriptan succinate (Imitrex) for migraine headaches. Which condition should alert the nurse to hold the medication and contact the health care provider? a. Bronchial asthma b. Prinzmetal's angina c. Diabetes mellitus d. Chronic kidney disease

ANS: B Sumatriptan succinate effectively reduces pain and other associated symptoms of migraine headache by binding to serotonin receptors and triggering cranial vasoconstriction. Vasoconstrictive effects are not confined to the cranium and can cause coronary vasospasm in clients with Prinzmetal's angina. The other conditions would not affect the client's treatment.

7. The nurse is caring for a client with a history of epilepsy who suddenly begins to experience a tonicclonic seizure and loses consciousness. What is the nurse's priority action? a. Restrain the client's extremities. b. Turn the client's head to the side. c. Take the client's blood pressure. d. Place an airway into the client's mouth.

ANS: B The nurse should turn the client's head to the side to prevent aspiration and allow drainage of secretions. The client should not be restrained nor an airway placed in his or her mouth during the seizure because these actions increase seizure activity and can harm the client. Vital signs are measured in the postictal phase of the seizure.

5. A nurse witnesses a client begin to experience a tonic-clonic seizure and loss of consciousness. Which action should the nurse take? a. Start fluids via a large-bore catheter. b. Turn the client's head to the side. c. Administer IV push diazepam. d. Prepare to intubate the client.

ANS: B The nurse should turn the client's head to the side to prevent aspiration and allow drainage of secretions. Anticonvulsants are administered on a routine basis if a seizure is sustained. If the seizure is sustained (status epilepticus), the client must be intubated and should be administered oxygen, 0.9% sodium chloride, and IV push lorazepam or diazepam.

The home health nurse is caring for an 81-year-old who had a stroke 2 months ago. Based on information shown in the accompanying figure from the history, physical assessment, and physical therapy/occupational therapy, which nursing diagnosis is the highest priority for this patient? a. Impaired transfer ability b. Risk for caregiver role strain c. Ineffective health maintenance d. Risk for unstable blood glucose level

ANS: B The spouse's household and patient care responsibilities, in combination with chronic illnesses, indicate a high risk for caregiver role strain. The nurse should further assess the situation and take appropriate actions. The data about the control of the patient's diabetes indicates that ineffective health maintenance and risk for unstable blood glucose are not priority concerns at this time. Because the patient is able to ambulate with a cane, the nursing diagnosis of impaired transfer ability is not supported. DIF: Cognitive Level: Analyze (analysis) REF: 1409 OBJ: Special Questions: Prioritization TOP: Nursing Process: Diagnosis MSC: NCLEX: Psychosocial Integrity

24. Which information about the patient who has had a subarachnoid hemorrhage is mostimportant to communicate to the health care provider? a. The patient complains of having a stiff neck. b. The patient's blood pressure (BP) is 90/50 mm Hg. c. The patient reports a severe and unrelenting headache. d. The cerebrospinal fluid (CSF) report shows red blood cells (RBCs).

ANS: B To prevent cerebral vasospasm and maintain cerebral perfusion, blood pressure needs to be maintained at a level higher than 90 mm Hg systolic after a subarachnoid hemorrhage. A low BP or drop in BP indicates a need to administer fluids and/or vasopressors to increase the BP. An ongoing headache, RBCs in the CSF, and a stiff neck are all typical clinical manifestations of a subarachnoid hemorrhage and do not need to be rapidly communicated to the health care provider.

Which information about the patient who has had a subarachnoid hemorrhage is most important to communicate to the health care provider? a. The patient complains of having a stiff neck. b. The patient's blood pressure (BP) is 90/50 mm Hg. c. The patient reports a severe and unrelenting headache. d. The cerebrospinal fluid (CSF) report shows red blood cells (RBCs).

ANS: B To prevent cerebral vasospasm and maintain cerebral perfusion, blood pressure needs to be maintained at a level higher than 90 mm Hg systolic after a subarachnoid hemorrhage. A low BP or drop in BP indicates a need to administer fluids and/or vasopressors to increase the BP. An ongoing headache, RBCs in the CSF, and a stiff neck are all typical clinical manifestations of a subarachnoid hemorrhage and do not need to be rapidly communicated to the health care provider. DIF: Cognitive Level: Apply (application) REF: 1393 OBJ: Special Questions: Prioritization TOP: Nursing Process: Assessment MSC: NCLEX: Physiological Integrity

26. Which transitory symptoms might occur when a patient is diagnosed with a TIA? (Select all that apply.) a. Incontinence b. Dysphagia c. Ptosis d. Tinnitus e. Dysarthria

ANS: B, C, D, E All, except transitory incontinence, are classic symptoms of a TIA. These deficits usually disappear without permanent disability in approximately 24 hours.

10. A nurse assesses a client who has encephalitis. Which manifestations should the nurse recognize as signs of increased intracranial pressure (ICP), a complication of encephalitis? (Select all that apply.) a. Photophobia b. Dilated pupils c. Headache d. Widened pulse pressure e. Bradycardia

ANS: B, D, E Increased ICP is a complication of encephalitis. The nurse should monitor for signs of increased ICP, including dilated pupils, widened pulse pressure, bradycardia, irregular respirations, and less responsive pupils. Photojans 3 days 47 minutes ago phobia and headache are not related to increased ICP.

2. A nurse is teaching a client who has chronic headaches. Which statements about headache triggers should the nurse include in this client's plan of care? (Select all that apply.) a. "Increase your intake of caffeinated beverages." b. "Incorporate physical exercise into your daily routine." c. "Avoid all alcoholic beverages." d. "Participate in a smoking cessation program." e. "Increase your intake of fruits and vegetables."

ANS: B, D, E Triggers for headaches include caffeine, smoking, and ingestion of pickled foods, so these factors should be avoided. Clients are taught to eat a balanced diet and to get adequate exercise and rest. Alcohol does not trigger chronic headaches but can enhance headaches during the headache period.

21. Which is the most effective intervention for best support of regular bowel elimination and the prevention of constipation? a. Limit fluid intake from 32 to 50 oz daily to compact the stool. b. Administer small soapsuds enema every other day to cleanse the bowel. c. Give stool softeners daily, establishing a consistent time to attempt elimination. d. Administer a strong laxative on a daily basis to encourage evacuation.

ANS: C Daily stool softeners, rather than daily laxatives or frequent enemas, help restore regularity and bowel tone.

2. The nurse is teaching a client with chronic headaches about headache triggers. Which statements does the nurse include in the client's teaching plan? (Select all that apply.) a. "Increase your intake of caffeinated beverages." b. "Increase your intake of fruits and vegetables." c. "Avoid all alcoholic beverages." d. "Avoid drinking red wine." e. "Incorporate physical exercise into your daily routine." f. "Incorporate an occasional fast into your plan."

ANS: B, D, E Triggers for headaches include caffeine, smoking, and ingestion of pickled foods. Clients are taught to eat a balanced diet and to get adequate exercise and rest.

5. A nurse assesses a client who is experiencing an absence seizure. For which clinical manifestations should the nurse assess? (Select all that apply.) a. Intermittent rigidity b. Lip smacking c. Sudden loss of muscle tone d. Brief jerking of the extremities e. Picking at clothing f. Patting of the hand on the leg

ANS: B, E, F Automatisms are characteristic of absence seizures. These behaviors consist of lip smacking, picking at clothing, and patting. Rigidity of muscles is associated with the tonic phase of a seizure, and jerking of the extremities is associated with the clonic phase of a seizure. Loss of muscle tone occurs with atonic seizures.

6. A patient has had a complete stroke as a result of a ruptured vessel in the left hemisphere. How should this patient's CVA be classified? a. Ischemic, embolic b. Hemorrhagic, subarachnoid c. Hemorrhagic, intracerebral d. Ischemic, thrombotic

ANS: C A ruptured vessel in a hemisphere is an intracerebral hemorrhagic CVA. It did not occur in the subarachnoid space. Ischemic CVAs are the result of occluded vessels.

2. A 68-year-old patient is being admitted with a possible stroke. Which information from the assessment indicates that the nurse should consult with the health care provider before giving the prescribed aspirin? a. The patient has dysphasia. b. The patient has atrial fibrillation. c. The patient reports that symptoms began with a severe headache. d. The patient has a history of brief episodes of right-sided hemiplegia.

ANS: C A sudden onset headache is typical of a subarachnoid hemorrhage, and aspirin is contraindicated. Atrial fibrillation, dysphasia, and transient ischemic attack (TIA) are not contraindications to aspirin use, so the nurse can administer the aspirin.

A 68-year-old patient is being admitted with a possible stroke. Which information from the assessment indicates that the nurse should consult with the health care provider before giving the prescribed aspirin? a. The patient has dysphasia. b. The patient has atrial fibrillation. c. The patient reports that symptoms began with a severe headache. d. The patient has a history of brief episodes of right-sided hemiplegia.

ANS: C A sudden onset headache is typical of a subarachnoid hemorrhage, and aspirin is contraindicated. Atrial fibrillation, dysphasia, and transient ischemic attack (TIA) are not contraindications to aspirin use, so the nurse can administer the aspirin. DIF: Cognitive Level: Apply (application) REF: 1392-1393 TOP: Nursing Process: Assessment MSC: NCLEX: Physiological Integrity

25. The nurse is caring for a patient who has been experiencing stroke symptoms for 60 minutes. Which action can the nurse delegate to a licensed practical/vocational nurse (LPN/LVN)? a. Assess the patient's gag and cough reflexes. b. Determine when the stroke symptoms began. c. Administer the prescribed short-acting insulin. d. Infuse the prescribed IV metoprolol (Lopressor).

ANS: C Administration of subcutaneous medications is included in LPN/LVN education and scope of practice. The other actions require more education and scope of practice and should be done by the registered nurse (RN).

The nurse is caring for a patient who has been experiencing stroke symptoms for 60 minutes. Which action can the nurse delegate to a licensed practical/vocational nurse (LPN/LVN)? a. Assess the patient's gag and cough reflexes. b. Determine when the stroke symptoms began. c. Administer the prescribed short-acting insulin. d. Infuse the prescribed IV metoprolol (Lopressor).

ANS: C Administration of subcutaneous medications is included in LPN/LVN education and scope of practice. The other actions require more education and scope of practice and should be done by the registered nurse (RN). DIF: Cognitive Level: Apply (application) REF: 1405 OBJ: Special Questions: Delegation TOP: Nursing Process: Planning MSC: NCLEX: Safe and Effective Care Environment

13. A nurse assesses a client with Alzheimer's disease who is recently admitted to the hospital. Which psychosocial assessment should the nurse complete? a. Assess religious and spiritual needs while in the hospital. b. Identify the client's ability to perform self-care activities. c. Evaluate the client's reaction to a change of environment. d. Ask the client about relationships with family members.

ANS: C As Alzheimer's disease progresses, the client experiences changes in emotional and behavioral affect. The nurse should be alert to the client's reaction to a change in environment, such as being hospitalized, because the client may exhibit an exaggerated response, such as aggression, to the event. The other assessments should be completed but are not as important as assessing the client's reaction to environmental change.

16. A client with Alzheimer's disease is admitted to the hospital. Which psychosocial assessment is most important for the nurse to complete? a. Ability to recall past events b. Ability to perform self-care c. Reaction to a change of environment d. Relationship with close family members

ANS: C As the disease progresses, the client experiences changes in emotional and behavioral affect. The nurse should be alert to the client's reaction to a change in environment, such as being hospitalized, because the client may exhibit an exaggerated response, such as aggression, to the event. The other assessments should be completed but are not as important for a client with Alzheimer's disease.

17. A 72-year-old patient who has a history of a transient ischemic attack (TIA) has an order for aspirin 160 mg daily. When the nurse is administering medications, the patient says, "I don't need the aspirin today. I don't have a fever." Which action should the nurse take? a. Document that the aspirin was refused by the patient. b. Tell the patient that the aspirin is used to prevent a fever. c. Explain that the aspirin is ordered to decrease stroke risk. d. Call the health care provider to clarify the medication order.

ANS: C Aspirin is ordered to prevent stroke in patients who have experienced TIAs. Documentation of the patient's refusal to take the medication is an inadequate response by the nurse. There is no need to clarify the order with the health care provider. The aspirin is not ordered to prevent aches and pains.

A 72-year-old patient who has a history of a transient ischemic attack (TIA) has an order for aspirin 160 mg daily. When the nurse is administering medications, the patient says, "I don't need the aspirin today. I don't have a fever." Which action should the nurse take? a. Document that the aspirin was refused by the patient. b. Tell the patient that the aspirin is used to prevent a fever. c. Explain that the aspirin is ordered to decrease stroke risk. d. Call the health care provider to clarify the medication order.

ANS: C Aspirin is ordered to prevent stroke in patients who have experienced TIAs. Documentation of the patient's refusal to take the medication is an inadequate response by the nurse. There is no need to clarify the order with the health care provider. The aspirin is not ordered to prevent aches and pains. DIF: Cognitive Level: Apply (application) REF: 1396 TOP: Nursing Process: Implementation MSC: NCLEX: Physiological Integrity

25. The nurse is caring for a client who has chronic migraine headaches. Which complementary health therapy does the nurse suggest? a. "Place a hot compress on your forehead at the onset of the headache." b. "Wear dark sunglasses when you are in brightly lit spaces." c. "Lie down in a darkened room when you experience a headache." d. "Do not sleep longer than 6 hours at one time."

ANS: C At the onset of a migraine attack, the client may be able to alleviate pain by lying down and darkening the room. He or she may want both eyes covered and a cool cloth on the forehead. If the client falls asleep, he or she should remain undisturbed until awakening. The other options are not recognized therapies for migraines.

20. A nurse is teaching a client with chronic migraine headaches. Which statement related to complementary therapy should the nurse include in this client's teaching? a. "Place a warm compress on your forehead at the onset of the headache." b. "Wear dark sunglasses when you are in brightly lit spaces." c. "Lie down in a darkened room when you experience a headache." d. "Set your alarm to ensure you do not sleep longer than 6 hours at one time."

ANS: C At the onset of a migraine attack, the client may be able to alleviate pain by lying down and darkening the room. He or she may want both eyes covered and a cool cloth on the forehead. If the client falls asleep, he or she should remain undisturbed until awakening. The other options are not recognized therapies for migraines.

12. The nurse identifies the nursing diagnosis of imbalanced nutrition: less than body requirements related to impaired self-feeding ability for a left-handed patient with left-sided hemiplegia. Which intervention should be included in the plan of care? a. Provide a wide variety of food choices. b. Provide oral care before and after meals. c. Assist the patient to eat with the right hand. d. Teach the patient the "chin-tuck" technique.

ANS: C Because the nursing diagnosis indicates that the patient's imbalanced nutrition is related to the left-sided hemiplegia, the appropriate interventions will focus on teaching the patient to use the right hand for self-feeding. The other interventions are appropriate for patients with other etiologies for the imbalanced nutrition.

The nurse identifies the nursing diagnosis of imbalanced nutrition: less than body requirements related to impaired self-feeding ability for a left-handed patient with left-sided hemiplegia. Which intervention should be included in the plan of care? a. Provide a wide variety of food choices. b. Provide oral care before and after meals. c. Assist the patient to eat with the right hand. d. Teach the patient the "chin-tuck" technique.

ANS: C Because the nursing diagnosis indicates that the patient's imbalanced nutrition is related to the left-sided hemiplegia, the appropriate interventions will focus on teaching the patient to use the right hand for self-feeding. The other interventions are appropriate for patients with other etiologies for the imbalanced nutrition. DIF: Cognitive Level: Apply (application) REF: 1407 TOP: Nursing Process: Planning MSC: NCLEX: Physiological Integrity

22. A nurse delegates care for a client with early-stage Alzheimer's disease to an unlicensed assistive personnel (UAP). Which statement should the nurse include when delegating this client's care? a. "If she is confused, play along and pretend that everything is okay." b. "Remove the clock from her room so that she doesn't get confused." c. "Reorient the client to the day, time, and environment with each contact." d. "Use validation therapy to recognize and acknowledge the client's concerns."

ANS: C Clients who have early-stage Alzheimer's disease should be reoriented frequently to person, place, and time. The UAP should reorient the client and not encourage the client's delusions. The room should have a clock and white board with the current date written on it. Validation therapy is used with late-stage Alzheimer's disease.

21. A client who has Alzheimer's disease is being discharged home. What safety instructions does the nurse include in the teaching plan for the client's caregiver? a. "Keep exercise to a minimum." b. "Place a padded throw rug at the bedside." c. "Install deadbolt locks on all outside doors." d. "Keep the lights off in the bedroom at night."

ANS: C Clients with Alzheimer's disease have a tendency to wander, especially at night. If possible, alarms should be installed on all outside doors to alert family members if the client leaves. At a minimum, all outside doors should have deadbolt locks installed to prevent the client from going outdoors unsupervised. The client should be allowed to exercise within his or her limits. Throw rugs are a slip and fall hazard and should be removed. The client may need or want lights on in the bedroom at night.

17. A nurse prepares to discharge a client with Alzheimer's disease. Which statement should the nurse include in the discharge teaching for this client's caregiver? a. "Allow the client to rest most of the day." b. "Place a padded throw rug at the bedside." c. "Install deadbolt locks on all outside doors." d. "Provide a high-calorie and high-protein diet."

ANS: C Clients with Alzheimer's disease have a tendency to wander, especially at night. If possible, alarms should be installed on all outside doors to alert family members if the client leaves. At a minimum, all outside doors should have deadbolt locks installed to prevent the client from going outdoors unsupervised. The client should be allowed to exercise within his or her limits. Throw rugs are a slip and fall hazard and should be removed. The client should eat a well-balanced diet. There is no need for a high-calorie or high-protein diet.

15. Which assessment indicates a fluid volume excess in a patient in the acute phase of a CVA? a. Decreased BP b. Weak pulse c. Adventitious breath sounds d. High urine-specific gravity

ANS: C Crackles in the lung fields are a major indicator of fluid excess. The pulse and BP are elevated in fluid excess. Urine-specific gravity is low in fluid excess.

12. A nurse is teaching the daughter of a client who has Alzheimer's disease. The daughter asks, "Will the medication my mother is taking improve her dementia?" How should the nurse respond? a. "It will allow your mother to live independently for several more years." b. "It is used to halt the advancement of Alzheimer's disease but will not cure it." c. "It will not improve her dementia but can help control emotional responses." d. "It is used to improve short-term memory but will not improve problem solving."

ANS: C Drug therapy is not effective for treating dementia or halting the advancement of Alzheimer's disease. However, certain drugs may help suppress emotional disturbances and psychiatric manifestations. Medication therapy may not allow the client to safely live independently.

11. When caring for a patient with a new right-sided homonymous hemianopsia resulting from a stroke, which intervention should the nurse include in the plan of care? a. Apply an eye patch to the right eye. b. Approach the patient from the right side. c. Place objects needed on the patient's left side. d. Teach the patient that the left visual deficit will resolve.

ANS: C During the acute period, the nurse should place objects on the patient's unaffected side. Because there is a visual defect in the right half of each eye, an eye patch is not appropriate. The patient should be approached from the left side. The visual deficit may not resolve, although the patient can learn to compensate for the defect.

When caring for a patient with a new right-sided homonymous hemianopsia resulting from a stroke, which intervention should the nurse include in the plan of care? a. Apply an eye patch to the right eye. b. Approach the patient from the right side. c. Place objects needed on the patient's left side. d. Teach the patient that the left visual deficit will resolve.

ANS: C During the acute period, the nurse should place objects on the patient's unaffected side. Because there is a visual defect in the right half of each eye, an eye patch is not appropriate. The patient should be approached from the left side. The visual deficit may not resolve, although the patient can learn to compensate for the defect. DIF: Cognitive Level: Apply (application) REF: 1407 TOP: Nursing Process: Planning MSC: NCLEX: Physiological Integrity

2. The nurse is assessing a client with a history of migraines. Which clinical manifestation is an early sign of a migraine with aura? a. Vertigo b. Lethargy c. Visual disturbances d. Numbness of the tongue

ANS: C Early warning of impending migraine with aura usually consists of visual changes, flashing lights, or diplopia. The other manifestations are not associated with an impending migraine with aura.

2. A nurse assesses a client who has a history of migraines. Which clinical manifestation should the nurse identify as an early sign of a migraine with aura? a. Vertigo b. Lethargy c. Visual disturbances d. Numbness of the tongue

ANS: C Early warning of impending migraine with aura usually consists of visual changes, flashing lights, or diplopia. The other manifestations are not associated with an impending migraine with aura.

20. Which stroke risk factor for a 48-year-old male patient in the clinic is most important for the nurse to address? a. The patient is 25 pounds above the ideal weight. b. The patient drinks a glass of red wine with dinner daily. c. The patient's usual blood pressure (BP) is 170/94 mm Hg. d. The patient works at a desk and relaxes by watching television.

ANS: C Hypertension is the single most important modifiable risk factor. People who drink more than 1 (for women) or 2 (for men) alcoholic beverages a day may increase risk for hypertension. Physical inactivity and obesity contribute to stroke risk but not as much as hypertension.

Which stroke risk factor for a 48-year-old male patient in the clinic is most important for the nurse to address? a. The patient is 25 pounds above the ideal weight. b. The patient drinks a glass of red wine with dinner daily. c. The patient's usual blood pressure (BP) is 170/94 mm Hg. d. The patient works at a desk and relaxes by watching television.

ANS: C Hypertension is the single most important modifiable risk factor. People who drink more than 1 (for women) or 2 (for men) alcoholic beverages a day may increase risk for hypertension. Physical inactivity and obesity contribute to stroke risk but not as much as hypertension. DIF: Cognitive Level: Apply (application) REF: 1390 OBJ: Special Questions: Prioritization TOP: Nursing Process: Assessment MSC: NCLEX: Health Promotion and Maintenance

16. Which intervention should the nurse include in a patient's plan of care to help preserve joint mobility in the acute phase of a CVA? a. Pull the limbs on the affected side into a functional position. b. Perform aggressive full range-of-motion exercises for all extremities. c. Support affected points in good functional alignment. d. Exercise the limbs every 8 hours.

ANS: C Limbs maintained in a functional anatomic position and gently exercised (never pulled) into an acceptable range of motion several times during a shift will maintain optimal mobility.

3. The nurse is reviewing a client's prescription for sumatriptan succinate (Imitrex). Which condition in this client's medical history does the nurse report to the health care provider? a. Bronchial asthma b. Gonorrhea c. Prinzmetal's angina d. Chronic kidney disease

ANS: C Sumatriptan succinate effectively reduces pain and other associated symptoms of migraine by binding to serotonin receptors and triggering cranial vasoconstriction. Vasoconstrictive effects are not confined to the cranium and can cause coronary vasospasm in clients with Prinzmetal's angina. The other conditions would not affect the client's treatment.

15. A male patient who has right-sided weakness after a stroke is making progress in learning to use the left hand for feeding and other activities. The nurse observes that when the patient's wife is visiting, she feeds and dresses him. Which nursing diagnosis is most appropriate for the patient? a. Interrupted family processes related to effects of illness of a family member b. Situational low self-esteem related to increasing dependence on spouse for care c. Disabled family coping related to inadequate understanding by patient's spouse d. Impaired nutrition: less than body requirements related to hemiplegia and aphasia

ANS: C The information supports the diagnosis of disabled family coping because the wife does not understand the rehabilitation program. There are no data supporting low self-esteem, and the patient is attempting independence. The data do not support an interruption in family processes because this may be a typical pattern for the couple. There is no indication that the patient has impaired nutrition.

A male patient who has right-sided weakness after a stroke is making progress in learning to use the left hand for feeding and other activities. The nurse observes that when the patient's wife is visiting, she feeds and dresses him. Which nursing diagnosis is most appropriate for the patient? a. Interrupted family processes related to effects of illness of a family member b. Situational low self-esteem related to increasing dependence on spouse for care c. Disabled family coping related to inadequate understanding by patient's spouse d. Impaired nutrition: less than body requirements related to hemiplegia and aphasia

ANS: C The information supports the diagnosis of disabled family coping because the wife does not understand the rehabilitation program. There are no data supporting low self-esteem, and the patient is attempting independence. The data do not support an interruption in family processes because this may be a typical pattern for the couple. There is no indication that the patient has impaired nutrition. DIF: Cognitive Level: Apply (application) REF: 1409 TOP: Nursing Process: Diagnosis MSC: NCLEX: Psychosocial Integrity

28. A 70-year-old female patient with left-sided hemiparesis arrives by ambulance to the emergency department. Which action should the nurse take first? a. Monitor the blood pressure. b. Send the patient for a computed tomography (CT) scan. c. Check the respiratory rate and effort. d. Assess the Glasgow Coma Scale score.

ANS: C The initial nursing action should be to assess the airway and take any needed actions to ensure a patent airway. The other activities should take place quickly after the ABCs (airway, breathing, and circulation) are completed.

A 70-year-old female patient with left-sided hemiparesis arrives by ambulance to the emergency department. Which action should the nurse take first? a. Monitor the blood pressure. b. Send the patient for a computed tomography (CT) scan. c. Check the respiratory rate and effort. d. Assess the Glasgow Coma Scale score.

ANS: C The initial nursing action should be to assess the airway and take any needed actions to ensure a patent airway. The other activities should take place quickly after the ABCs (airway, breathing, and circulation) are completed. DIF: Cognitive Level: Apply (application) REF: 1397-1398 OBJ: Special Questions: Prioritization TOP: Nursing Process: Implementation MSC: NCLEX: Physiological Integrity

20. Which outcome criterion is the most appropriate for a patient with "Imbalanced nutrition, related to dysphagia, with the goal of adequate nutrition"? a. Offers a variety of food groups b. Eats half of all meals offered c. Maintains body weight of 150 to 155 lb d. Eats all meals independently

ANS: C The maintenance of a desired weight is indicative of adequate nutrition. Eating a portion of a meal or eating independently does not adequately measure the extent to which the goal was met. Offering a variety of foods is a nursing or dietary function, not an outcome.

27. What purposes exist for a stent in the carotid artery of a person with a TIA? (Select all that apply.) a. Capture circulating clots. b. Help with subsequent angioplasties. c. Keep the artery open. d. Prevent hemorrhage. e. Measure the pressure in the artery.

ANS: C The only purpose of a stent is to keep an artery open.

14. A 47-year-old patient will attempt oral feedings for the first time since having a stroke. The nurse should assess the gag reflex and then a. order a varied pureed diet. b. assess the patient's appetite. c. assist the patient into a chair. d. offer the patient a sip of juice.

ANS: C The patient should be as upright as possible before attempting feeding to make swallowing easier and decrease aspiration risk. To assess swallowing ability, the nurse should initially offer water or ice to the patient. Pureed diets are not recommended because the texture is too smooth. The patient may have a poor appetite, but the oral feeding should be attempted regardless.

A 47-year-old patient will attempt oral feedings for the first time since having a stroke. The nurse should assess the gag reflex and then a. order a varied pureed diet. b. assess the patient's appetite. c. assist the patient into a chair. d. offer the patient a sip of juice.

ANS: C The patient should be as upright as possible before attempting feeding to make swallowing easier and decrease aspiration risk. To assess swallowing ability, the nurse should initially offer water or ice to the patient. Pureed diets are not recommended because the texture is too smooth. The patient may have a poor appetite, but the oral feeding should be attempted regardless. DIF: Cognitive Level: Apply (application) REF: 1406 TOP: Nursing Process: Implementation MSC: NCLEX: Physiological Integrity

1. After a patient experienced a brief episode of tinnitus, diplopia, and dysarthria with no residual effects, the nurse anticipates teaching the patient about a. cerebral aneurysm clipping. b. heparin intravenous infusion. c. oral low-dose aspirin therapy. d. tissue plasminogen activator (tPA).

ANS: C The patient's symptoms are consistent with transient ischemic attack (TIA), and drugs that inhibit platelet aggregation are prescribed after a TIA to prevent stroke. Continuous heparin infusion is not routinely used after TIA or with acute ischemic stroke. The patient's symptoms are not consistent with a cerebral aneurysm. tPA is used only for acute ischemic stroke, not for TIA.

After a patient experienced a brief episode of tinnitus, diplopia, and dysarthria with no residual effects, the nurse anticipates teaching the patient about a. cerebral aneurysm clipping. b. heparin intravenous infusion. c. oral low-dose aspirin therapy. d. tissue plasminogen activator (tPA).

ANS: C The patient's symptoms are consistent with transient ischemic attack (TIA), and drugs that inhibit platelet aggregation are prescribed after a TIA to prevent stroke. Continuous heparin infusion is not routinely used after TIA or with acute ischemic stroke. The patient's symptoms are not consistent with a cerebral aneurysm. tPA is used only for acute ischemic stroke, not for TIA. DIF: Cognitive Level: Apply (application) REF: 1391 | 1396 TOP: Nursing Process: Planning MSC: NCLEX: Physiological Integrity

4. During the change of shift report a nurse is told that a patient has an occluded left posterior cerebral artery. The nurse will anticipate that the patient may have a. dysphasia. b. confusion. c. visual deficits. d. poor judgment.

ANS: C Visual disturbances are expected with posterior cerebral artery occlusion. Aphasia occurs with middle cerebral artery involvement. Cognitive deficits and changes in judgment are more typical of anterior cerebral artery occlusion.

2. Which patient is at the greatest risk for a CVA? a. A 20-year-old obese Latin woman who is taking birth control pills b. A 40-year-old athletic white man with a family history of CVA c. A 60-year-old Asian woman who smokes occasionally d. A 65-year-old African American man with hypertension

ANS: D Older African Americans have a higher incidence of CVA than occasional smokers, young persons, or athletes. Hypertension increases the risk.

During the change of shift report a nurse is told that a patient has an occluded left posterior cerebral artery. The nurse will anticipate that the patient may have a. dysphasia. b. confusion. c. visual deficits. d. poor judgment.

ANS: C Visual disturbances are expected with posterior cerebral artery occlusion. Aphasia occurs with middle cerebral artery involvement. Cognitive deficits and changes in judgment are more typical of anterior cerebral artery occlusion. DIF: Cognitive Level: Apply (application) REF: 1393 TOP: Nursing Process: Assessment MSC: NCLEX: Physiological Integrity

18. Which assessment indicates that a patient with a CVA is in transition to the rehabilitation phase? a. BP has been within normal limits for 24 hours. b. Patient makes positive statements about his condition. c. No further neurologic deficits are observed. d. Successful attempts are made at independent function.

ANS: C When no further deficits are noted and all vital signs have stabilized, the patient is considered to be in the rehabilitation phase. Positive statements and attempts at independence are not sufficient.

8. A nurse assesses a client who is recovering from the implantation of a vagal nerve stimulation device. For which clinical manifestations should the nurse assess as common complications of this procedure? (Select all that apply.) a. Bleeding b. Infection c. Hoarseness d. Dysphagia e. Seizures

ANS: C, D Complications of surgery to implant a vagal nerve stimulation device include hoarseness (most common), dyspnea, neck pain, and dysphagia. The device is tunneled under the skin with an electrode connected to the vagus nerve to control simple or complex partial seizures. Bleeding is not a common complication of this procedure, and infection would not occur during the recovery period.

29. How does a lacunar stroke differ from an ischemic CVA? (Select all that apply.) a. Causes a great deal of pain b. Alters the personality c. Affects small arteries d. Nearly always results in blindness e. Produces a small amount of neurologic damage

ANS: C, E The lacunar CVA only affects small arteries and produces a small amount of neurologic damage.

3. A patient experienced a period of momentary confusion, dizziness, and slurred speech but recovered in 2 hours. Which assessment in the diagnosis of this episode would be most helpful? a. Patient's complaint of nausea b. Blood pressure (BP) of 140/90 mm Hg c. Patient's complaint of headache d. Auscultation of a bruit over the carotid artery

ANS: D A carotid bruit is evidence of a narrowing in that vessel, a symptom of a possible CVA or transient ischemic attack (TIA). BP of 140/90 mm Hg, although at the high end, is considered within normal limits. Headache and nausea alone are too common to be definitive.

13. The nurse is talking to the family of a client who has Parkinson's disease. Which statement indicates that the family has a good understanding of the changes in motor movement associated with this disease? a. "I can never tell what she's thinking. She hides behind a frozen face." b. "She drools all the time so I just can't take her out anywhere." c. "I think this disease makes her nervous. She perspires all the time." d. "She has trouble chewing so I will offer bite-sized portions."

ANS: D A masklike face, drooling, and excess perspiration are common in clients with Parkinson's disease. Changes in facial expression or a masklike facies in a Parkinson's disease client can be misinterpreted. Because chewing and swallowing can be problematic, small frequent meals and a supplement are better for meeting the client's nutritional needs. The other statements indicate poor understanding of the disease process.

10. After teaching the wife of a client who has Parkinson disease, the nurse assesses the wife's understanding. Which statement by the client's wife indicates she correctly understands changes associated with this disease? a. "His masklike face makes it difficult to communicate, so I will use a white board." b. "He should not socialize outside of the house due to uncontrollable drooling." c. "This disease is associated with anxiety causing increased perspiration." d. "He may have trouble chewing, so I will offer bite-sized portions."

ANS: D Because chewing and swallowing can be problematic, small frequent meals and a supplement are better for meeting the client's nutritional needs. A masklike face and drooling are common in clients with Parkinson disease. The client should be encouraged to continue to socialize and communicate as normally as possible. The wife should understand that the client's masklike face can be misinterpreted and additional time may be needed for the client to communicate with her or others. Excessive perspiration is also common in clients with Parkinson disease and is associated with the autonomic nervous system's response.

7. A patient admitted with possible stroke has been aphasic for 3 hours and his current blood pressure (BP) is 174/94 mm Hg. Which order by the health care provider should the nurse question? a. Keep head of bed elevated at least 30 degrees. b. Infuse normal saline intravenously at 75 mL/hr. c. Administer tissue plasminogen activator (tPA) per protocol. d. Administer a labetalol (Normodyne) drip to keep BP less than 140/90 mm Hg.

ANS: D Because elevated BP may be a protective response to maintain cerebral perfusion, antihypertensive therapy is recommended only if mean arterial pressure (MAP) is >130 mm Hg or systolic pressure is >220 mm Hg. Fluid intake should be 1500 to 2000 mL daily to maintain cerebral blood flow. The head of the bed should be elevated to at least 30 degrees, unless the patient has symptoms of poor tissue perfusion. tPA may be administered if the patient meets the other criteria for tPA use.

A patient admitted with possible stroke has been aphasic for 3 hours and his current blood pressure (BP) is 174/94 mm Hg. Which order by the health care provider should the nurse question? a. Keep head of bed elevated at least 30 degrees. b. Infuse normal saline intravenously at 75 mL/hr. c. Administer tissue plasminogen activator (tPA) per protocol. d. Administer a labetalol (Normodyne) drip to keep BP less than 140/90 mm Hg.

ANS: D Because elevated BP may be a protective response to maintain cerebral perfusion, antihypertensive therapy is recommended only if mean arterial pressure (MAP) is >130 mm Hg or systolic pressure is >220 mm Hg. Fluid intake should be 1500 to 2000 mL daily to maintain cerebral blood flow. The head of the bed should be elevated to at least 30 degrees, unless the patient has symptoms of poor tissue perfusion. tPA may be administered if the patient meets the other criteria for tPA use. DIF: Cognitive Level: Apply (application) REF: 1397 TOP: Nursing Process: Implementation MSC: NCLEX: Physiological Integrity

5. When teaching about clopidogrel (Plavix), the nurse will tell the patient with cerebral atherosclerosis a. to monitor and record the blood pressure daily. b. that Plavix will dissolve clots in the cerebral arteries. c. that Plavix will reduce cerebral artery plaque formation. d. to call the health care provider if stools are bloody or tarry.

ANS: D Clopidogrel (Plavix) inhibits platelet function and increases the risk for gastrointestinal bleeding, so patients should be advised to notify the health care provider about any signs of bleeding. The medication does not lower blood pressure, decrease plaque formation, or dissolve clots.

When teaching about clopidogrel (Plavix), the nurse will tell the patient with cerebral atherosclerosis a. to monitor and record the blood pressure daily. b. that Plavix will dissolve clots in the cerebral arteries. c. that Plavix will reduce cerebral artery plaque formation. d. to call the health care provider if stools are bloody or tarry.

ANS: D Clopidogrel (Plavix) inhibits platelet function and increases the risk for gastrointestinal bleeding, so patients should be advised to notify the health care provider about any signs of bleeding. The medication does not lower blood pressure, decrease plaque formation, or dissolve clots. DIF: Cognitive Level: Apply (application) REF: 1398 TOP: Nursing Process: Implementation MSC: NCLEX: Physiological Integrity

9. A client with new-onset status epilepticus is prescribed phenytoin (Dilantin). After teaching the client about this treatment regimen, the nurse assesses the client's understanding. Which statement indicates that the client understands the teaching? a. "I must drink at least 2 liters of water daily." b. "This will stop me from getting an aura before a seizure." c. "I will not be able to be employed while taking this medication." d. "Even when my seizures stop, I will take this drug."

ANS: D Discontinuing antiepileptic drugs can lead to the recurrence of seizures or status epilepticus. The client does not need to drink more water and can continue to work while taking this medication. The medication will not stop an aura before a seizure.

7. After teaching a client who is diagnosed with new-onset status epilepticus and prescribed phenytoin (Dilantin), the nurse assesses the client's understanding. Which statement by the client indicates a correct understanding of the teaching? a. "To prevent complications, I will drink at least 2 liters of water daily." b. "This medication will stop me from getting an aura before a seizure." c. "I will not drive a motor vehicle while taking this medication." d. "Even when my seizures stop, I will continue to take this drug."

ANS: D Discontinuing antiepileptic drugs can lead to the recurrence of seizures or status epilepticus. The client does not need to drink more water and can drive while taking this medication. The medication will not stop an aura before a seizure.

15. The daughter of a client with Alzheimer's disease asks, "Will the medication my mother is taking improve her dementia?" How does the nurse respond? a. "It will help your mother live independently once more." b. "It is used to halt the advancement of Alzheimer's disease but will not cure it." c. "It will provide a steady improvement in memory but not in problem solving." d. "It will not improve dementia but can help control emotional responses."

ANS: D Drug therapy is not effective for treating dementia or halting the advancement of Alzheimer's disease. However, certain drugs may help suppress emotional disturbances and psychiatric manifestations.

14. The nurse is caring for a client with Parkinson's disease. Which intervention does the nurse implement to prevent respiratory complications in the client? a. Keep an oral airway at the bedside. b. Ensure fluid intake of at least 3 L/day. c. Teach the client pursed-lip breathing techniques. d. Maintain the head of the bed at 30 degrees or greater.

ANS: D Elevation of the back rest will help prevent aspiration. The other options will not prevent aspiration, which is the greatest respiratory complication of Parkinson's disease.

11. A nurse plans care for a client with Parkinson disease. Which intervention should the nurse include in this client's plan of care? a. Ambulate the client in the hallway twice a day. b. Ensure a fluid intake of at least 3 liters per day. c. Teach the client pursed-lip breathing techniques. d. Keep the head of the bed at 30 degrees or greater.

ANS: D Elevation of the head of the bed will help prevent aspiration. The other options will not prevent aspiration, which is the greatest respiratory complication of Parkinson disease, nor do these interventions address any of the complications of Parkinson disease. Ambulation in the hallway is usually implemented to prevent venous thrombosis. Increased fluid intake flushes out toxins from the client's blood. Pursed-lip breathing increases exhalation of carbon dioxide.

14. Pneumonia is the most frequent cause of death after a stroke. Which intervention would be contraindicated in the acute care of a patient with a hemorrhagic CVA? a. Thicken liquids to ease swallowing and prevent aspiration. b. Change position every 30 to 60 minutes. c. Maintain adequate fluid intake, orally or IV. d. Encourage forceful coughing to stimulate deep breathing.

ANS: D Forceful coughing is contraindicated for the patient with a hemorrhagic CVA because it may cause increased intracranial pressure.

25. The wife of a husband who has had a CVA asks why he is being treated with insulin since he has no history of diabetes. What is the best response by the nurse as to why hyperglycemia occurs after a stroke? a. Brain swelling b. Hypertension c. Immobility d. Stress

ANS: D Hyperglycemia occurs after a CVA as the body's response to stress. If left untreated, the hyperglycemia will cause increased brain damage and worsen the outcome of the stroke.

23. The nurse is planning to bathe a client diagnosed with meningococcal meningitis. In addition to gloves, what personal protective equipment does the nurse use? a. Particulate respirator b. Isolation gown c. Shoe covers d. Surgical mask

ANS: D Meningeal meningitis is spread via saliva and droplets. Caregivers should wear a surgical mask when within 6 feet of the client and should continue to use Standard Precautions. A particulate respirator, an isolation gown, and shoe covers are not necessary for Droplet Precautions.

11. The nurse assesses for which clinical manifestations in the client with suspected encephalitis? a. Fever of 101° F (38.3° C) b. Nausea and vomiting c. Hypoactive deep tendon reflexes d. Pain on flexion of the neck

ANS: D Nuchal rigidity is associated with meningeal irritation and is frequently present in clients with encephalitis. The other manifestations are not associated with encephalitis.

19. A 58-year-old patient with a left-brain stroke suddenly bursts into tears when family members visit. The nurse should a. use a calm voice to ask the patient to stop the crying behavior. b. explain to the family that depression is normal following a stroke. c. have the family members leave the patient alone for a few minutes. d. teach the family that emotional outbursts are common after strokes.

ANS: D Patients who have left-sided brain stroke are prone to emotional outbursts that are not necessarily related to the emotional state of the patient. Depression after a stroke is common, but the suddenness of the patient's outburst suggests that depression is not the major cause of the behavior. The family should stay with the patient. The crying is not within the patient's control and asking the patient to stop will lead to embarrassment.

A 58-year-old patient with a left-brain stroke suddenly bursts into tears when family members visit. The nurse should a. use a calm voice to ask the patient to stop the crying behavior. b. explain to the family that depression is normal following a stroke. c. have the family members leave the patient alone for a few minutes. d. teach the family that emotional outbursts are common after strokes.

ANS: D Patients who have left-sided brain stroke are prone to emotional outbursts that are not necessarily related to the emotional state of the patient. Depression after a stroke is common, but the suddenness of the patient's outburst suggests that depression is not the major cause of the behavior. The family should stay with the patient. The crying is not within the patient's control and asking the patient to stop will lead to embarrassment. DIF: Cognitive Level: Apply (application) REF: 1409 TOP: Nursing Process: Implementation MSC: NCLEX: Psychosocial Integrity

23. Nurses in change-of-shift report are discussing the care of a patient with a stroke who has progressively increasing weakness and decreasing level of consciousness (LOC). Which nursing diagnosis do they determine has thehighest priority for the patient? a. Impaired physical mobility related to weakness b. Disturbed sensory perception related to brain injury c. Risk for impaired skin integrity related to immobility d. Risk for aspiration related to inability to protect airway

ANS: D Protection of the airway is the priority of nursing care for a patient having an acute stroke. The other diagnoses are also appropriate, but interventions to prevent aspiration are the priority at this time.

Nurses in change-of-shift report are discussing the care of a patient with a stroke who has progressively increasing weakness and decreasing level of consciousness (LOC). Which nursing diagnosis do they determine has the highest priority for the patient? a. Impaired physical mobility related to weakness b. Disturbed sensory perception related to brain injury c. Risk for impaired skin integrity related to immobility d. Risk for aspiration related to inability to protect airway

ANS: D Protection of the airway is the priority of nursing care for a patient having an acute stroke. The other diagnoses are also appropriate, but interventions to prevent aspiration are the priority at this time. DIF: Cognitive Level: Apply (application) REF: 1397 | 1401 OBJ: Special Questions: Prioritization TOP: Nursing Process: Analysis MSC: NCLEX: Physiological Integrity

22. A patient with left-sided weakness that started 60 minutes earlier is admitted to the emergency department and diagnostic tests are ordered. Which test should be done first? a. Complete blood count (CBC) b. Chest radiograph (Chest x-ray) c. 12-Lead electrocardiogram (ECG) d. Noncontrast computed tomography (CT) scan

ANS: D Rapid screening with a noncontrast CT scan is needed before administration of tissue plasminogen activator (tPA), which must be given within 4.5 hours of the onset of clinical manifestations of the stroke. The sooner the tPA is given, the less brain injury. The other diagnostic tests give information about possible causes of the stroke and do not need to be completed as urgently as the CT scan.

A patient with left-sided weakness that started 60 minutes earlier is admitted to the emergency department and diagnostic tests are ordered. Which test should be done first? a. Complete blood count (CBC) b. Chest radiograph (Chest x-ray) c. 12-Lead electrocardiogram (ECG) d. Noncontrast computed tomography (CT) scan

ANS: D Rapid screening with a noncontrast CT scan is needed before administration of tissue plasminogen activator (tPA), which must be given within 4.5 hours of the onset of clinical manifestations of the stroke. The sooner the tPA is given, the less brain injury. The other diagnostic tests give information about possible causes of the stroke and do not need to be completed as urgently as the CT scan. DIF: Cognitive Level: Apply (application) REF: 1395 OBJ: Special Questions: Prioritization TOP: Nursing Process: Implementation MSC: NCLEX: Physiological Integrity

22. A patient in the rehabilitation phase after a CVA accidentally knocks the adapted plate from the table and bursts into tears after failing to feed himself. What is the best response by the nurse? a. "Don't cry. You'll be mastering eating in no time." b. "I don't believe crying will help. Let's try drinking from a special cup." c. "Bless your heart! Let me get a new meal and feed you." d. "Learning new skills is hard. Let's see what may have caused the trouble."

ANS: D Recognizing effort and showing support are the best approaches to depression and frustration. Babying the patient and admonitions against crying add to the problem. Redirection to the task at hand is therapeutic.

3. A 73-year-old patient with a stroke experiences facial drooping on the right side and right-sided arm and leg paralysis. When admitting the patient, which clinical manifestation will the nurse expect to find? a. Impulsive behavior b. Right-sided neglect c. Hyperactive left-sided tendon reflexes d. Difficulty comprehending instructions

ANS: D Right-sided paralysis indicates a left-brain stroke, which will lead to difficulty with comprehension and use of language. The left-side reflexes are likely to be intact. Impulsive behavior and neglect are more likely with a right-side stroke.

A 73-year-old patient with a stroke experiences facial drooping on the right side and right-sided arm and leg paralysis. When admitting the patient, which clinical manifestation will the nurse expect to find? a. Impulsive behavior b. Right-sided neglect c. Hyperactive left-sided tendon reflexes d. Difficulty comprehending instructions

ANS: D Right-sided paralysis indicates a left-brain stroke, which will lead to difficulty with comprehension and use of language. The left-side reflexes are likely to be intact. Impulsive behavior and neglect are more likely with a right-side stroke. DIF: Cognitive Level: Apply (application) REF: 1407 TOP: Nursing Process: Assessment MSC: NCLEX: Physiological Integrity

5. A client with epilepsy develops stiffening of the muscles of the arms and legs, followed by an immediate loss of consciousness and jerking of all extremities. How does the nurse document this seizure activity? a. Atonic seizure b. Absence seizure c. Myoclonic seizure d. Tonic-clonic seizure

ANS: D Seizure activity that begins with stiffening of the arms and legs, followed by loss of consciousness and jerking of all extremities, is characteristic of a tonic-clonic seizure. The other seizures do not manifest in this manner.

10. The nurse is teaching a client who is newly diagnosed with epilepsy. Which statement by the client indicates a need for further teaching concerning the drug regimen? a. "I will not drink any alcoholic beverages." b. "I will wear a medical alert bracelet." c. "I will let my doctor know about all of my prescriptions." d. "I can skip a couple of pills if they make me ill."

ANS: D The nurse must emphasize that antiepileptic drugs must be taken even if seizure activity has stopped. Discontinuing the medication can predispose the client to seizure activity and status epilepticus. The client should not drink alcohol while taking seizure medications. The client should wear a medical alert bracelet and should make the doctor aware of all medications to prevent complications of polypharmacy.

8. After teaching a client newly diagnosed with epilepsy, the nurse assesses the client's understanding. Which statement by the client indicates a need for additional teaching? a. "I will wear my medical alert bracelet at all times." b. "While taking my epilepsy medications, I will not drink any alcoholic beverages." c. "I will tell my doctor about my prescription and over-the-counter medications." d. "If I am nauseated, I will not take my epilepsy medication."

ANS: D The nurse must emphasize that antiepileptic drugs must be taken even if the client is nauseous. Discontinuing the medication can predispose the client to seizure activity and status epilepticus. The client should not drink alcohol while taking seizure medications. The client should wear a medical alert bracelet and should make the doctor aware of all medications to prevent complications of polypharmacy.

8. A 56-year-old patient arrives in the emergency department with hemiparesis and dysarthria that started 2 hours previously, and health records show a history of several transient ischemic attacks (TIAs). The nurse anticipates preparing the patient for a. surgical endarterectomy. b. transluminal angioplasty. c. intravenous heparin administration. d. tissue plasminogen activator (tPA) infusion.

ANS: D The patient's history and clinical manifestations suggest an acute ischemic stroke and a patient who is seen within 4.5 hours of stroke onset is likely to receive tPA (after screening with a CT scan). Heparin administration in the emergency phase is not indicated. Emergent carotid transluminal angioplasty or endarterectomy is not indicated for the patient who is having an acute ischemic stroke.

A 56-year-old patient arrives in the emergency department with hemiparesis and dysarthria that started 2 hours previously, and health records show a history of several transient ischemic attacks (TIAs). The nurse anticipates preparing the patient for a. surgical endarterectomy. b. transluminal angioplasty. c. intravenous heparin administration. d. tissue plasminogen activator (tPA) infusion.

ANS: D The patient's history and clinical manifestations suggest an acute ischemic stroke and a patient who is seen within 4.5 hours of stroke onset is likely to receive tPA (after screening with a CT scan). Heparin administration in the emergency phase is not indicated. Emergent carotid transluminal angioplasty or endarterectomy is not indicated for the patient who is having an acute ischemic stroke. DIF: Cognitive Level: Apply (application) REF: 1391 | 1398 TOP: Nursing Process: Planning MSC: NCLEX: Physiological Integrity

24. A client diagnosed with the Huntington gene but who has no symptoms asks for options related to family planning. Which is the nurse's best response? a. "Most clients with the Huntington gene do not pass on Huntington disease to their children." b. "I understand that they can diagnose this disease in embryos. Therefore you could select a healthy embryo from your fertilized eggs for implantation to avoid passing on Huntington disease." c. "The need for family planning is limited because one of the hallmarks of Huntington disease is infertility." d. "Tell me more specifically what information you need about family planning so that I can direct you to the right information or health care provider."

ANS: D The presence of the Huntington gene means that the trait will be passed on to all offspring of the affected person. Understanding options for contraception and conception (e.g., surrogate mother options) and implications for children may require the expertise of a genetic counselor or a reproductive specialist. The other options are not accurate.

19. A nurse cares for a client who has been diagnosed with the Huntington gene but has no symptoms. The client asks for options related to family planning. What is the nurse's best response? a. "Most clients with the Huntington gene do not pass on Huntington disease to their children." b. "I understand that they can diagnose this disease in embryos. Therefore, you could select a healthy embryo from your fertilized eggs for implantation to avoid passing on Huntington disease." c. "The need for family planning is limited because one of the hallmarks of Huntington disease is infertility." d. "Tell me more specifically what information you need about family planning so that I can direct you to the right information or health care provider."

ANS: D The presence of the Huntington gene means that the trait will be passed on to all offspring of the affected person. Understanding options for contraception and conception (e.g., surrogacy options) and implications for children may require the expertise of a genetic counselor or a reproductive specialist. The other statements are not accurate.

21. A patient in the emergency department with sudden-onset right-sided weakness is diagnosed with an intracerebral hemorrhage. Which information about the patient is most important to communicate to the health care provider? a. The patient's speech is difficult to understand. b. The patient's blood pressure is 144/90 mm Hg. c. The patient takes a diuretic because of a history of hypertension. d. The patient has atrial fibrillation and takes warfarin (Coumadin).

ANS: D The use of warfarin probably contributed to the intracerebral bleeding and remains a risk factor for further bleeding. Administration of vitamin K is needed to reverse the effects of the warfarin, especially if the patient is to have surgery to correct the bleeding. The history of hypertension is a risk factor for the patient but has no immediate effect on the patient's care. The BP of 144/90 indicates the need for ongoing monitoring but not for any immediate change in therapy. Slurred speech is consistent with a left-sided stroke, and no change in therapy is indicated.

A patient in the emergency department with sudden-onset right-sided weakness is diagnosed with an intracerebral hemorrhage. Which information about the patient is most important to communicate to the health care provider? a. The patient's speech is difficult to understand. b. The patient's blood pressure is 144/90 mm Hg. c. The patient takes a diuretic because of a history of hypertension. d. The patient has atrial fibrillation and takes warfarin (Coumadin).

ANS: D The use of warfarin probably contributed to the intracerebral bleeding and remains a risk factor for further bleeding. Administration of vitamin K is needed to reverse the effects of the warfarin, especially if the patient is to have surgery to correct the bleeding. The history of hypertension is a risk factor for the patient but has no immediate effect on the patient's care. The BP of 144/90 indicates the need for ongoing monitoring but not for any immediate change in therapy. Slurred speech is consistent with a left-sided stroke, and no change in therapy is indicated. DIF: Cognitive Level: Apply (application) REF: 1398 OBJ: Special Questions: Prioritization TOP: Nursing Process: Assessment MSC: NCLEX: Physiological Integrity

8. When should a nurse recognize that the acute phase of a CVA has ended? a. Forty-eight hours has passed from its onset. b. The patient begins to respond verbally. c. BP drops. d. Vital signs and neurologic signs stabilize.

ANS: D When the vital and neurologic signs stabilize, the acute phase has ended. Verbal response, lower BP, and the passage of time without other signs are not adequate evidence that the acute phase has ended.

6. A nurse prepares to provide perineal care to a client with meningococcal meningitis. Which personal protective equipment should the nurse wear? (Select all that apply.) a. Particulate respirator b. Isolation gown c. Shoe covers d. Surgical mask e. Gloves

ANS: D, E Meningeal meningitis is spread via saliva and droplets, and Droplet Precautions are necessary. Caregivers should wear a surgical mask when within 6 feet of the client and should continue to use Standard Precautions, including gloves. A particulate respirator, an isolation gown, and shoe covers are not necessary for Droplet Precautions.

28. What signs and symptoms characterize expressive aphasia? (Select all that apply.) a. Speech that sounds normal but makes no sense b. Total inability to communicate c. Difficulty understanding the written and spoken word d. Stuttering and spitting e. Difficulty initiating speech

ANS: E Expressive aphasia makes it difficult for the patient to initiate speech.

The nurse is talking to the family of a stroke patient about home care measures. Which topics does the nurse include in this discussion? (select all that apply) a. Need for caregivers to plan for routine respite care and protection of own health b. Evaluation for potential safety risks such as throw rugs or slippery floors c. Awareness of potential patient frustration associated with communication d. Avoidance of independent transfers by the patient because of safety issues e. Access to health resources such as publications from the American Heart Association f. Referral to hospice and encouragement of family discussion of advance directives.

Abce

The nurse is performing a neurologic assessment on a patient with a suspected stroke. In addition to the level of consciousness (LOC), what is assessed to evaluate cognitive changes that may be occurring? (Select all that apply.) a. Denial of illness b. Proprioceptive dysfunction c. Presence of flaccid paralysis d. Impairment of memory e. Decreased ability to concentrate

Abde

The nurse is preparing to discharge a patient with transient ischemic attacks. What treatment areas does the nurse include in discharge teaching? (Select all that apply.) a. Reduction of high blood pressure b. Drug teaching for aspirin or another antiplatelet drug c. Lifestyle changes such as increased sleep and rest d. Controlling diabetes e. Increased risk for stroke

Abde

A patient with increased ICP is to receive IV mannitol (Osmitrol). Which nursing actions are taken concerning this drug? (Select all that apply.) a. Draw up the drug through a filtered needle. b. Insert a Foley catheter for strict measurement of urine output. c. Monitor serum and urine osmolality on a weekly basis. d. Assess for acute renal failure, weakness, or edema. e. Administer mannitol through a filter in the IV tubing. f. Administer furosemide (Lasix) as an adjunctive therapy.

Abdef

Which patients are at increased risk for stroke? (Select all that apply.) a. 66-year-old man with diabetes mellitus b. 35-year-old healthy woman who uses oral contraceptives c. 47-year -old woman who exercises regularly d. 35-year-old man with history of multiple transient ischemic attacks e. 25-year-old woman with Bell's palsy f. 53-year-old man with chronic alcoholism

Abdf

A patient with a stroke is having some trouble swallowing. Which interventions does the nurse anticipate the speech-language pathologist to suggest after the swallowing evaluation is completed? (Select all that apply.) a. Position the patient upright while eating. b. Administer orange juice using a straw. c. Give small spoonfuls of soft foods such as custard. d. Add powdered thickeners to liquids. e. Provide liquid nutritional supplements between meals for added calories.

Acde

The nurse is performing discharge teaching for the family and patient who has had prolonged hospitalization and rehabilitation therapy for severe craniocerebral trauma after a motor-cycle accident. What elements of instruction does the nurse include? (select all that apply) a. Review seizure precautions b. Stimulate the patient with frequent changes in the environment c. Develop a routine of activities with consistency and structure. d. Attend follow-up appointments with therapists e. Encourage the family to seek respite care if needed. f. Encourage the patient to wear a helmet when riding

Acde

Which symptoms indicate that a patient's stroke has affected the right hemisphere? (select all that apply.) a. Loss of depth perception b. Aphasia c. Denies illness d. Cannot recognize faces e. Loss of hearing f. Depression

Acde

Which groups of people have the highest prevalence of stroke?

American Indian/Alaskan native groups

A patient has been diagnosed with a large lesion of the parietal lobe and demonstrates loss of sensory function. Which nursing intervention is applicable for this patient? a. Play music for the patient for at least 30 minutes each day. b. Teach the patient to test the water temperature used for bathing c. Position the patient reclining in bed or in a chair for meals d. Use a picture of the patient's spouse and ask the patient to state the spouse's name.

B

A patient has sustained a traumatic brain injury. Which nursing intervention is best for this patient? a. Assess vital signs every 8 hours b. Position to avoid extreme flexion c. Increase fluid intake for the first 48 hours d. Administer glucocorticoids

B

A patient received rtPA for the treatment of ischemic stroke and the physician ordered an IV sodium heparin infusion. In relation to the drug therapy, what does the nurse monitor for? a. Elevated prothrombin level b. Bleeding gums or bruising c. Nausea and vomiting d. Elevated hematocrit or hemoglobin

B

A patient with an ischemic stroke is placed on a cardiac monitor. Which cardiac dysrhythmia places the patient at risk for emboli? a. Sinus bradycardia b. Atrial fibrillation c. Sinus tachycardia d. First-degree heart block

B

A priority problem for a patient who was admitted for a brain attack is the potential for aspiration. Which intervention is best to delegate to the UAP? a. Monitor the patient for and notify the charge nurse of any occurrence of coughing, choking, or difficulty breathing. b. Elevate the head of the bed as appropriate and slowly feed small spoonfuls of pudding, pausing between each spoonful. c. Assess the swallow reflex by placing the index finger and thumb on either side of the Adam's apple. d. Give the patient a class of water before feeding solid foods and have oral suction ready at the bedside.

B

A stroke patient is at risk for increased intracranial pressure (ICP) and is receiving oxygen 2 L via nasal cannula. The nurse is reviewing arterial blood gas (ABG) results. Which ABG value is of greatest concern for this patient? a. pH 7.32 b. Paco₂ of 60 mm Hg c. Pao₂ of 95 mm Hg d. HCO₃ of 28 mEq/L

B

In planning care for a patient with increased ICP, what does the nurse do to minimize ICP? a. Gives the bath, changes the linens, and does passive ROM exercises to hands/fingers then allows the patient to rest b. Gives the bath, allows the patient to rest, changes the linens, allows the patient to rest, and then performs passive ROM exercises to hands/fingers c. Defers the bath, changes the linens, and does passive ROM exercises to extremities until the danger of increased ICP as passed. d. Contacts the physician for specific orders about all activities related to the care of the patient that might cause increased ICP.

B

The nurse is giving a discharge instructions to the mother of a child who bumped her head on a table. Which statement by the mother indicates an understanding of instructions? a. "I should not let her fall asleep" b. She may have nausea or headache for the first 24 hours" c. "She should gently blow her nose and I'll observe for bleeding" d. "She can run and play as she usually does"

B

The nurse notices that a patient seems to be having trouble swallowing. Which intervention does the nurse employ for this patient? a. Limit the diet to clear liquids given through a straw. b. Keep the patient on NPO status until swallowing is assessed. c. Monitor the patient's weight and compare to baseline. d. Sit with the patient while the patient eats and observe for swallowing difficulties.

B

The patient with a traumatic brain injury is receiving mechanical ventilation. Why does the health care provider adjust ventilator settings to maintain a partial pressure of arterial carbon dioxide (Paco₂) at 35 to 28 mm Hg? a. Lower levels of arterial carbon dioxide are essential for gas exchange b. Carbon dioxide is a potent vasodilator that can cause increased ICP c. Carbon dioxide is a waste product that must be eliminated from the body d. Lower levels of arterial carbon dioxide facilitate brain oxygenation

B

The preferred administration time for recombinant tissue plasminogen activator (rtPA [Retavase]) is within how long of stroke symptom onset? a. 30 to 60 minutes b. 3 to 4.5 hours c. 6 to 8 hours d. 24 to 30 hours

B

The provider has prescribed barbiturate coma therapy for a patient with increased ICP. Which complication does the nurse monitor for? a. Decreased LOC b. Reduced gastric motility c. Decreased respiratory rate d. Reduced Glasgow coma scale score

B

Which statement about transient ischemic at- tack (TM) is accurate? a. TIAs do not cause permanent brain damage. b. TIA increases the risk of stroke. c. Symptoms of a TIA usually resolve in 10- 15 minutes. d. After a TIA, a patient is prescribed a beta blocker.

B

Which statement is true about motor changes in a patient who has had a stroke? a. Motor deficit is ipsilateral to the hemisphere affected. b. Motor deficit is contralateral to the hemisphere affected. c. Bowel and bladder function remain intact. d. Flaccid paralysis is not an expected finding and should be reported promptly.

B

Gail, the daughter states, "I don't understand what a brain attack is. The healthcare provider told me my mother is in serious condition and they are going to run several tests. I just don't know what is going on. What happened to my mother?" What is the best response by the nurse?

B) "Your mother has had a stroke, and the blood supply to the brain has been blocked." Rationale: The nurse can discuss what a diagnosis means. Nancy is unable to make decisions, so the next of kin, her daughter, Gail, needs sufficient information to make informed decisions. The nurse has the knowledge, and the responsibility, to explain Nancy's condition to Gail. The nurse should give facts first, and then address her feelings after the information is provided.

Which intervention should the RN implement when preparing Mrs. Jackson and her daughter for this procedure?

B) Explain to the daughter that her mother will have to remain still throughout the CT scan. Rationale: Because head motion will distort the images, Nancy will have to remain still throughout the procedure. Allergies to iodine is important if contrast dye is being used for the CT scan. Premedicating the client to decrease pain prior to the procedure is unnecessary because CT scanning is a noninvasive and painless procedure. Providing an explanation of relaxation exercises prior to the procedure is a worthwhile intervention to decrease anxiety but is not of highest priority.

Gail tells the nurse she is going to go outside to smoke a cigarette and will only be gone for a few minutes. Which statement is warranted in this situation? A. "Make sure you smoke in the smoking area only. The hospital has strict rules." B. "I should let you know that smoking is a strong risk factor for a brain attack." C. "That is just fine. I will be here taking care of your mother." D. "How long have you been smoking?"

B. "I should let you know that smoking is a strong risk factor for a brain attack." - The nurse should teach Gail that smoking is a modifiable risk factor that could prevent her from having a stroke. Smoking increases the risk for hypertension, which is a risk factor for a stroke.

As the nurse assesses Nancy, Gail asks, "Why isn't my mother a candidate for thrombolytic therapy?"

B. "She is not a candidate because of therapeutic time constraints related to this medication." Thrombolytic therapy is contraindicated in clients with symptom onset longer than 3 hours prior to admission. Nancy had symptoms for 24 hours before being brought to the medical center.

Gail tells the nurse, "One of the people in the waiting room was telling me about an operation that her mother had to prevent a stroke. Do you know anything about that?" How should the RN respond?

B. "That procedure is only done with small strokes,not like the one your Mom had." This surgery is indicated for clients with symptoms of transient ischemic attack (TIA), or mild stroke, found to be due to severe carotid artery stenosis or moderate stenosis with other significant risk factors.

Nancy is transferred to the Intermediate Care Unit after the MRI is completed. She has a 20 gauge saline lock in her right forearm and an 18 French indwelling (Foley) catheter. Gail is sitting by her mother's bed. The nurse asks Gail if there is anyone that can be called so she won't be alone. She informs the nurse that she is an only child and her father died years ago. Gail states, "I don't understand what a brain attack is. The healthcare provider told me my mother is in serious condition and they are going to run several tests. I just don't know what's going on. What happened to my mother?" Which response is best by the nurse? A. "How do you feel about what the healthcare provider said?" B. "Your mother has had a stroke, and the blood supply to the brain has been compromised." C. "I will call the healthcare provider so he/she can talk to you about your mother's serious condition." D. "I am sorry, but what happened to your mother is confidential and I cannot give you any information."

B. "Your mother has had a stroke, and the blood supply to the brain has been compromised." -The nurse has the knowledge, and the responsibility, to explain Nancy's condition to Gail.

At what rate would the RN set the infusion pump?

B. 60 ml/hr

A female patient has left-sided hemiplegia following an ischemic stroke that she experienced 4 days earlier. How should the nurse best promote the health of the patient's integumentary system? A. Position the patient on her weak side the majority of the time. B. Alternate the patient's positioning between supine and side-lying. C. Avoid the use of pillows in order to promote independence in positioning. D. Establish a schedule for the message of areas where skin breakdown emerges.

B. Alternate the patient's positioning between supine and side-lying. A position change schedule should be established for stroke patients. An example is side-back-side, with a maximum duration of 2 hours for any position. The patient should be positioned on the weak or paralyzed side for only 30 minutes. Pillows may be used to facilitate positioning. Areas of skin breakdown should never be massaged.

The RN assesses Mrs. Jackson's apical pulse but cannot hear anything. Which intervention should the RN implement first?

B. Continue to stay at Mrs. Jackson's bedside and hold Gail's hand.

Several weeks after a stroke, a 50-year-old male patient has impaired awareness of bladder fullness, resulting in urinary incontinence. Which nursing intervention will be best to include in the initial plan for an effective bladder training program? a. Limit fluid intake to 1200 mL daily to reduce urine volume. b. Assist the patient onto the bedside commode every 2 hours. c. Perform intermittent catheterization after each voiding to check for residual urine. d. Use an external "condom" catheter to protect the skin and prevent embarrassment.

B. Developing a regular voiding schedule will prevent incontinence and may increase patient awareness of a full bladder. A 1200 mL fluid restriction may lead to dehydration. Intermittent catheterization and use of a condom catheter are appropriate in the acute phase of stroke, but should not be considered solutions for long-term management because of the risks for urinary tract infection (UTI) and skin breakdown.

The RN remains with Gail at Mrs. Jackson bedside. The HCP is called an pronounces Mrs. Jackson's death. Gail tells the RN that Mrs. Jackson wanted to be in organ donor. Which action should the RN implement?

B. Explain that Mrs. Jackson can only be a tissue donor, not an organ donor.

A patient in the clinic reports a recent episode of dysphasia and left-sided weakness at home that resolved after 2 hours. The nurse will anticipate teaching the patient about a. alteplase (tPA). b. aspirin (Ecotrin). c. warfarin (Coumadin). d. nimodipine (Nimotop).

B. Following a transient ischemic attack (TIA), patients typically are started on medications such as aspirin to inhibit platelet function and decrease stroke risk. tPA is used for acute ischemic stroke. Coumadin is usually used for patients with atrial fibrillation. Nimodipine is used to prevent cerebral vasospasm after a subarachnoid hemorrhage.

Mrs. Jackson was a Roman Catholic so Gail ask for the RN if Mrs. Jackson can receive, the sacrament for the sick. Which action would be most important for the RN to take in the situation?

B. Have a priest perform Mrs. Jackson's anointing of the sick

Gail begins to cry and states, "Mom was just fine last week when we went out to eat and to a show. I love my mom so much, and I am so scared. She is all I have." How should the RN respond?

B. I know this is scary for you. Would you like to sit and talk?

The nurse would expect to find what clinical manifestation in a patient admitted with a left-sided stroke? A. Impulsivity B. Impaired speech C. Left-side neglect D. Short attention span

B. Impaired speech Clinical manifestations of left-sided brain damage include right hemiplegia, impaired speech/language, impaired right/left discrimination, and slow and cautious performance. Impulsivity, left-sided neglect, and short attention span are all manifestations of right-sided brain damage.

Which nursing diagnosis has the highest priority? A. Impaired physical mobility B. Impaired swallowing C. Self-care deficit D. Impaired social interaction

B. Impaired swallowing - According to Maslow's Hierarchy of Needs, physiological needs should be addressed first. Therefore, Nancy's dysphagia is the highest priority nursing diagnosis since she is at risk for aspiration.

The physician orders alteplase (Activase) for a 58-year-old man diagnosed with an acute ischemic stroke. Which nursing action is most appropriate? A. Administer the medication by an IV route at 15 mL/hr for 24 hours. B. Insert two or three large-bore IV catheters before administering the medication. C. If gingival bleeding occurs, discontinue the medication and notify the physician. D. Reduce the medication infusion rate for a systolic blood pressure above 180 mm Hg.

B. Insert two or three large-bore IV catheters before administering the medication. Before giving alteplase, the nurse should start two or three large bore IVs. Bleeding is a major complication with fibrinolytic therapy, and venipunctures should not be attempted after alteplase is administered. Altepase is administered IV with an initial bolus dose followed by an infusion of the remaining medication within the next 60 minutes. Gingival bleeding is a minor complication and may be controlled with pressure or ice packs. Control of blood pressure is critical prior to altepase administration and for the following 24 hours. Before administering altepase, a systolic pressure above 180 mm Hg or diastolic pressure above 110 mm Hg requires aggressive blood pressure treatment to reduce the risk of cerebral hemorrhage.

The ED physician has completed an assessment. Gail is sitting at the bedside while the ED nurse continues to assess Nancy every 15 minutes. Which assessment finding warrants immediate intervention by the nurse? A. Nancy has a negative Babinski's reflex bilaterally B. Nancy only responds to a painful stimuli C. Nancy's Glasgow Coma Scale (GCS) score increases D. Nancy's bilateral grip strength is unequal

B. Nancy only responds to painful stimuli - This decrease in responsiveness warrants immediate intervention by the nurse, indicating a worsening condition (increased intracranial pressure).

Which written documentation should the nurse put in the client's record?

B. PT reported that client became dizzy and was lowered back to the bed with the assistance of a gait belt - This documentation provides the factual data of the events that occurred.

Nancy is experiencing homonymous hemianopsia as a result of her brain attack. Which nursing intervention would the nurse implement address this condition?

B. Place the objects Nancy needs for activities of daily living on the left side of the table. Homonymous hemianopsia is loss of the visual field on the same side as the paralyzed side. This results in the client neglecting that side of the body, so it is beneficial to place objects on that side. Nancy had a left-hemisphere brain attack so her right side is the weak side.

The nurse is caring for a patient who has just returned after having left carotid artery angioplasty and stenting. Which assessment information is of most concern to the nurse? a. The pulse rate is 102 beats/min. b. The patient has difficulty speaking. c. The blood pressure is 144/86 mm Hg. d. There are fine crackles at the lung bases.

B. Small emboli can occur during carotid artery angioplasty and stenting, and the aphasia indicates a possible stroke during the procedure. Slightly elevated pulse rate and blood pressure are not unusual because of anxiety associated with the procedure. Fine crackles at the lung bases may indicate atelectasis caused by immobility during the procedure. The nurse should have the patient take some deep breaths.

Which rehabilitation team member is responsible for evaluating Mrs. Jackson's dysphagia?

B. The speech therapist The speech therapist evaluates the e client's gag reflex and ability to swallow, then makes recommendations regarding feeding techniques and diet.

The home health nurse is caring for an 81-year-old who had a stroke 2 months ago. Based on information shown in the accompanying figure from the history, physical assessment, and physical therapy/occupational therapy, which nursing diagnosis is the highest priority for this patient? a. Impaired transfer ability b. Risk for caregiver role strain c. Ineffective health maintenance d. Risk for unstable blood glucose level

B. The spouse's household and patient care responsibilities, in combination with chronic illnesses, indicate a high risk for caregiver role strain. The nurse should further assess the situation and take appropriate actions. The data about the control of the patient's diabetes indicates that ineffective health maintenance and risk for unstable blood glucose are not priority concerns at this time. Because the patient is able to ambulate with a cane, the nursing diagnosis of impaired transfer ability is not supported.

Which information about the patient who has had a subarachnoid hemorrhage is most important to communicate to the health care provider? a. The patient complains of having a stiff neck. b. The patient's blood pressure (BP) is 90/50 mm Hg. c. The patient reports a severe and unrelenting headache. d. The cerebrospinal fluid (CSF) report shows red blood cells (RBCs).

B. To prevent cerebral vasospasm and maintain cerebral perfusion, blood pressure needs to be maintained at a level higher than 90 mm Hg systolic after a subarachnoid hemorrhage. A low BP or drop in BP indicates a need to administer fluids and/or vasopressors to increase the BP. An ongoing headache, RBCs in the CSF, and a stiff neck are all typical clinical manifestations of a subarachnoid hemorrhage and do not need to be rapidly communicated to the health care provider.

Because Nancy is right-handed and is having difficulty performing activities of daily living with the left arm, the nurse also includes the nursing diagnosis "self-care deficit" in the care plan. Which intervention would the nurse implement to address this nursing diagnosis?

B. Utilize plate guards when Nancy is eating Plate guards prevent food from being pushed off the plate. Using plate guards and other assistive devices will encourage independence in a client with a self-care deficit.

A diagnosis of a ruptured cerebral aneurysm has been made in a patient with manifestations of a stroke. The nurse anticipates that treatment options that would be evaluated for the patient include a. hyperventilation therapy b. surgical clipping of the aneurysm c. administration of hyperosmotic agents d. administration of thrombolytic therapy

B: Surgical clipping of they aneurysm- Surgical management with clipping of an aneurysm to decrease re bleeding and vasospasm is an option for a stroke cause by rupture of a cerebral aneurysm. Placement of coils into the lumens of the aneurysm by intercentional radiologists is increasing in popularity. Hyperventilation therapy would increase vasodilation and the potential for hemorrhage. Thrombolytic therapy would be absolutely contraindicated, and if a vessel is patent, osmotic diuretics may leak into tissue, pulling fluid out of the vessel and increasing edema.

A male patient has sustained a stroke and the nurse is planning interventions to help him reestablish urinary continence. What action does the nurse take? a. Obtain an order for a Foley catheter. b. Offer the urinal to the patient every 6 hours. c. Check postvoid residual urine with a bladder ultrasound. d. Restrict fluid to 1500 mL/day.

C

What is a neurotransmitter?

Chemical that transmits signals from a neuron to a target cell

To promote communication during rehabilitation of the patient with aphasia, an appropriate nursing intervention is to a. use gestures, pictures, and music to stimulate patient responses b. talk about activities of daily living (ADLs) that are familiar to the patient c. structure statements so that patient does not have to respond verbally d. use flashcards with simple words and pictures to promote language recall

B: Talk about ADLs that are familiar to the patient- during rehabilitation, the patient with aphasia needs frequent, meaningful verbal stimulation that has relevance for him. Conversation by the nurse and family should address ADLs that are familiar to the patient. Gestures, pictures, and simple statements are more appropriate in the acute phase, when patients may be overwhelmed with verbal stimuli. Flashcards are often perceived by the patient as childish and meaningless.

Which interventions does the nurse use for a patient with a left hemisphere stroke? (Select all that apply.) a. Teach the patient to wash both sides of the face. b. Place pictures and familiar objects around the patient. c. Reorient the patient frequently. d. Repeat names of commonly used objects. e. Approach the patient from the unaffected side. f. Establish a structured routine for the patient

Bcdf

Intracerbral hemorrhage is described as?

Bleeding into the brain tissue generally resulting form severe or sustained hypertension.

A stroke by the national stroke association is also called what?

Brain attack

A patient is admitted for a closed head injury from a fall down the stairs. The patient has no history of respiratory disease and no apparent respiratory distress. However, the physician orders oxygen 2 L via nasal cannula. What is the nurse's best action? a. Check the pulse oximetry and apply the oxygen if the saturation level drops below 90% b. Call the physician to discontinue the order because it is unnecessary c. Deliver the oxygen as ordered because hypoxemia may precipitate increased ICP d. Apply the nasal cannula as ordered and gradually wean the patient off the oxygen when the LOC improves

C

A patient is diagnosed with an ischemic stroke. The UAP reports that the patient's Vital signs are blood pressure 150/ 100 mm Hg, pulse 78 beats/ min, respiratory rate of 20/ min, and temperature of 98.7° F. The patient's blood pressure is normally around 120/80. What action does the nurse take first? a. Report the blood pressure immediately to the physician because there is a danger of rebleeding. b. Ask the nursing assistant to repeat the blood pressure measurement in the other extremity with a manual cuff. c. Check the physician's orders to see if the blood pressure is within the acceptable parameters. d. Nothing; an elevated blood pressure is necessary for cerebral perfusion.

C

A patient is scheduled for a craniotomy. What does the nurse tell the patient and family about the procedure? a. The head will not need to be shaved at the surgical site. b. There is a coma state for up to several days after surgery. c. Drainage of a small to moderate amount of cerebrospinal fluid after surgery is normal. d. The family will need to remind the patient of their names and relationships.

C

The nurse is assessing a patient who was struck in the head several times with a bat. There is clear fluid that appears to be leaking from the nose. What action does the nurse take? a. Hand the patient a tissue and ask him to gently blow the nose; observe the nasal discharge for blood clots. b. Immediately report the finding to the physician and document the observation in the nursing notes c. Place a drop of the fluid on a white absorbent background and look for a yellow halo d. Allow the patient to wipe his nose, but no other action is needed; he has most likely been crying

C

The nurse is caring for a patient at risk for an increased ICP related to ischemic stroke. For what purpose does the nurse place the patient's head in a midline neutral position? a. Provide comfort for the patient b. Protect the cervical spine c. Facilitate venous drainage from brain d. Decrease pressure from cerebrospinal fluid

C

The nurse is caring for a patient with a relatively minor head injury after a bump to the head. The nurse has the greatest concern about which symptom? a. Headache b. Nausea and vomiting c. Unequal pupils d. Dizziness

C

The nurse is caring for a patient with right hemisphere damage. The patient demonstrates disorientation to time and place, he has poor depth perception, and demonstrates neglect of the left visual field. Which task is best delegated to the unlicensed assistive personnel (UAP)? a. Move the patient's bed so that his affected side faces the door b. Teach the patient to wash both sides of his face c. Ensure a safe environment by removing clutter. d. Suggest to the family that they bring familiar family pictures

C

The nurse is conducting a presentation to a group of students on the prevention of head injuries. Which statement by a student indicates a need for additional teaching? a. "Drinking, driving, and speeding contribute to the risk for injury" b. "Males are more likely to sustain head injury compared to females." c. "Young people are less likely to get injured because of faster reflexes" d. "Following game rules and not goofing around can prevent injuries"

C

The nurse is taking a history on a teenager who was involved in a motor vehicle accident with friends. The patient has an obvious contusion of the forehead, seems confused, and is laughing loudly and yelling "Ruby! Ruby!" What is the best question for the nurse to ask the patient's friends? a. "Where and why did the accident occur?" b. How can we notify the family for consent for treatment?" c. "Was the patient using drugs or alcohol prior to the accident?" d. "Who is Ruby and why is the patient calling for her?"

C

Which determination must be made first in assessing a patient with traumatic brain injury? a. Presence of spinal injury b. Whether the patient is hypotensive c. Presence of a patent airway d. Level of consciousness using the Glasgow coma scale

C

Which statement is true about a patient at risk for increased ICP? a. The appearance of abnormal posturing occurs only when the patient is not positioned for comfort. b. Cushing's reflex, an early sign of increased ICP, consists of severe hypertension, wideing pulse pressure, and bradycardia. c. Dilated or pinpoint pupils that are slow to react to light or nonreactive to light are signs of increased ICP. d. Areas of tenderness over the scalp indicate the presence of contrecoup injuries.

C

A neurologist prescribes a magnetic resonance imaging (MRI) of the head STAT for a patient. Which data warrants immediate intervention by the nurse concerning this diagnostic test?

C) Right hip replacement. The magnetic field generated by the MRI is so strong that metal-containing items are strongly attracted to the magnet. Because the hip joint is made of metal, a lead shield must be used during the procedure. Elevated blood pressure, an allergy to shell fish, and a history of atrial fibrillation would not affect the MRI.

A 63-year-old patient who began experiencing right arm and leg weakness is admitted to the emergency department. In which order will the nurse implement these actions included in the stroke protocol? a. Obtain computed tomography (CT) scan without contrast. b. Infuse tissue plasminogen activator (tPA). c. Administer oxygen to keep O2 saturation >95%. d. Use National Institute of Health Stroke Scale to assess patient.

C, D, A, B The initial actions should be those that help with airway, breathing, and circulation. Baseline neurologic assessments should be done next. A CT scan will be needed to rule out hemorrhagic stroke before tPA can be administered.

Which intervention should the nurse implement to prevent joint deformities? A. Place the elbow lower than the shoulder and the wrist lower than the elbow on the affected side. B. Position the fingers so that they are totally flexed in a slight pronation position. C. Place Nancy in a pone position for 15 minutes at least 4 times a day. D. Apply splints to the arms and legs during the day but remove at night

C. Place Nancy in a prone position for 15 minutes at least 4 times a day - This helps to promote hyperextension of the hip joints, which helps prevent knee and hip flexion contractures.

Gail starts to cry and states, "Mom was just fine last week when we went out to eat and to a show. I love my mom so much, and I am so scared. She is all I have." How should the nurse respond? A. "I am sure everything will be all right." B. "I will notify the chaplain to come and sit with you so you won't be alone." C. "I know this is scary for you. Would you like to sit and talk?" C. "I am sure your mother knows you are here. Just keep talking to her."

C. "I know this is scary for you. Would you like to sit and talk? -This therapeutic response provides acknowledgment of Gail's fears, and the nurse offers to take time to discuss the situation.

Gail tells the nurse, "One of the people in the waiting room was telling me about an operation that her mother had to prevent a stroke. Do you know anything about that?" A. "There is currently no surgery that can help prevent a stroke." B. "I am sure your healthcare provider will discuss that with you at a later date." C. "That procedure is only done with small strokes, not like the one your Mom had." D. "Yes, it is a carotid endarterectomy, and your mother may be able to have one."

C. "That procedure is only done with small strokes,not like the one your Mom had." - This surgery is indicated for clients with symptoms of transient ischemic attack (TIA), or mild stroke, found to be due to severe carotid artery stenosis or moderate stenosis with other significant risk factors.

A 68-year-old patient is being admitted with a possible stroke. Which information from the assessment indicates that the nurse should consult with the health care provider before giving the prescribed aspirin? a. The patient has dysphasia. b. The patient has atrial fibrillation. c. The patient reports that symptoms began with a severe headache. d. The patient has a history of brief episodes of right-sided hemiplegia.

C. A sudden onset headache is typical of a subarachnoid hemorrhage, and aspirin is contraindicated. Atrial fibrillation, dysphasia, and transient ischemic attack (TIA) are not contraindications to aspirin use, so the nurse can administer the aspirin.

The nurse is caring for a patient who has been experiencing stroke symptoms for 60 minutes. Which action can the nurse delegate to a licensed practical/vocational nurse (LPN/LVN)? a. Assess the patient's gag and cough reflexes. b. Determine when the stroke symptoms began. c. Administer the prescribed short-acting insulin. d. Infuse the prescribed IV metoprolol (Lopressor).

C. Administration of subcutaneous medications is included in LPN/LVN education and scope of practice. The other actions require more education and scope of practice and should be done by the registered nurse (RN).

A 72-year-old patient who has a history of a transient ischemic attack (TIA) has an order for aspirin 160 mg daily. When the nurse is administering medications, the patient says, "I don't need the aspirin today. I don't have a fever." Which action should the nurse take? a. Document that the aspirin was refused by the patient. b. Tell the patient that the aspirin is used to prevent a fever. c. Explain that the aspirin is ordered to decrease stroke risk. d. Call the health care provider to clarify the medication order.

C. Aspirin is ordered to prevent stroke in patients who have experienced TIAs. Documentation of the patient's refusal to take the medication is an inadequate response by the nurse. There is no need to clarify the order with the health care provider. The aspirin is not ordered to prevent aches and pains.

The nurse identifies the nursing diagnosis of imbalanced nutrition: less than body requirements related to impaired self-feeding ability for a left-handed patient with left-sided hemiplegia. Which intervention should be included in the plan of care? a. Provide a wide variety of food choices. b. Provide oral care before and after meals. c. Assist the patient to eat with the right hand. d. Teach the patient the "chin-tuck" technique.

C. Because the nursing diagnosis indicates that the patient's imbalanced nutrition is related to the left-sided hemiplegia, the appropriate interventions will focus on teaching the patient to use the right hand for self-feeding. The other interventions are appropriate for patients with other etiologies for the imbalanced nutrition.

Which action should the RN implement to address this situation?

C. Discuss how to use a communication board w/ both Mrs. Jackson and her daughter.

When caring for a patient with a new right-sided homonymous hemianopsia resulting from a stroke, which intervention should the nurse include in the plan of care? a. Apply an eye patch to the right eye. b. Approach the patient from the right side. c. Place objects needed on the patient's left side. d. Teach the patient that the left visual deficit will resolve.

C. During the acute period, the nurse should place objects on the patient's unaffected side. Because there is a visual defect in the right half of each eye, an eye patch is not appropriate. The patient should be approached from the left side. The visual deficit may not resolve, although the patient can learn to compensate for the defect.

Which nursing care task should the nurse delegate to the UAP? A. Assist Nancy to eat her breakfast B. Use a walker to help Nancy ambulate down the hall C. Give Nancy a bed bath and change the bed linens D. Flush Nancy's saline lock with 2 ml of normal saline

C. Give Nancy a bed bath and change the bed linens - The UAP can assist Nancy with bathing and then change the bed linens. This task does not require professional judgment or expertise

Which stroke risk factor for a 48-year-old male patient in the clinic is most important for the nurse to address? a. The patient is 25 pounds above the ideal weight. b. The patient drinks a glass of red wine with dinner daily. c. The patient's usual blood pressure (BP) is 170/94 mm Hg. d. The patient works at a desk and relaxes by watching television.

C. Hypertension is the single most important modifiable risk factor. People who drink more than 1 (for women) or 2 (for men) alcoholic beverages a day may increase risk for hypertension. Physical inactivity and obesity contribute to stroke risk but not as much as hypertension.

The telephone at Mrs. Jackson's bedside starts ringing. The RN answers the phone. The caller is one of Mrs. Jackson's neighbors, wanting to know how Mrs. Jackson is doing. How should the nurse respond?

C. I am sorry, but I am unable to give you any information

Nancy is experiencing pain in her right shoulder. The nurse is aware that up to 70% of clients with a brain attack experience severe pain in the shoulder that prevents them from learning new skills. Shoulder function helps clients achieve balance, perform transfer skills, and participate in self-care activities.Which intervention should the nurse implement when addressing this condition? A. Move Nancy by lifting with the affected shoulder B. Assist Nancy to keep the affected arm in a dependent position as much as possible C. Instruct Nancy to clasp the right hand with the left hand and raise both hands above the head D. Remind Nancy to perform active range of motion exercises daily

C. Instruct Nancy to clasp the right hand with the left hand and raise both hands above the head - This exercise helps prevent "frozen shoulder" and will aid the nurse when moving or positioning the client.

With a diagnosis of a brain attack (stroke), which priority intervention should the nurse include in Nancy's plan of care? A. Monitor INR daily B. Assess neurological status every shift C. Keep the head of the bed elevated D. Evaluate platelet levels daily

C. Keep the head of the bed elevated - Maintaining a patent airway is essential to support oxygenation and cerebral perfusion. Elevating the head of the bed 30 degrees aids in preventing the tongue from falling backward and obstructing the airway.

With a diagnosis of a brain attack, stroke, which priority intervention should the RN include and Mrs. Jackson's plan of care?

C. Keep the head of the bed elevated Maintaining a patent airway is essential to support oxygenation and cerebral perfusion. Elevating the head of the bed 30 degrees aids in preventing the tongue from falling backward and obstructing the airway.

Which assessment finding warrants immediate intervention by the RN? (Select all that apply)

C. Mrs. Jackson only response to painful stimuli D. Positive Babinski's reflex bilaterally E. Pupils are reacting unequally, and she is experiencing sensitivity to light

A 70-year-old female patient with left-sided hemiparesis arrives by ambulance to the emergency department. Which action should the nurse take first? a. Monitor the blood pressure. b. Send the patient for a computed tomography (CT) scan. c. Check the respiratory rate and effort. d. Assess the Glasgow Coma Scale score.

C. The initial nursing action should be to assess the airway and take any needed actions to ensure a patent airway. The other activities should take place quickly after the ABCs (airway, breathing, and circulation) are completed.

The nurse is planning psychosocial support for the patient and family of the patient who suffered a stroke. What factor will most likely have the greatest impact on positive family coping with the situation? A. Specific patient neurologic deficits B. The patient's ability to communicate C. Rehabilitation potential of the patient D. Presence of complications of a stroke

C. Rehabilitation potential of the patient Although a patient's neurologic deficit might initially be severe after a stroke, the ability of the patient to recover is most likely to positively impact the family's coping with the situation. Providing explanations and emotional support beginning in the acute phase through the rehabilitation phase will facilitate coping. Emphasizing successes will offer the most realistic hope for the patient's rehabilitation and helps maintain hope for the patient's future abilities.

The neurologist also prescribes a magnetic resonance imaging (MRI) of the head STAT. Which data warrants immediate intervention by the nurse concerning this diagnostic test? A. Allergy to shellfish B. History of atrial fibrillation C. Right hip replacement D. Elevated blood pressure

C. Right hip replacement -The magnetic field generated by the MRI is so strong that metal-containing items are strongly attracted to the magnet. Because the hip joint is made of metal, a lead shield must be used during the procedure.

Which sensory-perceptual deficit is associated with left-sided stroke (right hemiplegia)? A. Overestimation of physical abilities. B. Difficulty judging position and distance. C. Slow and possibly fearful performance of tasks. D. Impulsivity and impatience at performing tasks.

C. Slow and possibly fearful performance of tasks. Patients with a left-sided stroke (right hemiplegia) commonly are slower in organization and performance of tasks and may have a fearful, anxious response to a stroke. Overconfidence, spatial disorientation, and impulsivity are more commonly associated with a right-sided stroke.

A male patient who has right-sided weakness after a stroke is making progress in learning to use the left hand for feeding and other activities. The nurse observes that when the patient's wife is visiting, she feeds and dresses him. Which nursing diagnosis is most appropriate for the patient? a. Interrupted family processes related to effects of illness of a family member b. Situational low self-esteem related to increasing dependence on spouse for care c. Disabled family coping related to inadequate understanding by patient's spouse d. Impaired nutrition: less than body requirements related to hemiplegia and aphasia

C. The information supports the diagnosis of disabled family coping because the wife does not understand the rehabilitation program. There are no data supporting low self-esteem, and the patient is attempting independence. The data do not support an interruption in family processes because this may be a typical pattern for the couple. There is no indication that the patient has impaired nutrition.

A 47-year-old patient will attempt oral feedings for the first time since having a stroke. The nurse should assess the gag reflex and then a. order a varied pureed diet. b. assess the patient's appetite. c. assist the patient into a chair. d. offer the patient a sip of juice.

C. The patient should be as upright as possible before attempting feeding to make swallowing easier and decrease aspiration risk. To assess swallowing ability, the nurse should initially offer water or ice to the patient. Pureed diets are not recommended because the texture is too smooth. The patient may have a poor appetite, but the oral feeding should be attempted regardless.

After a patient experienced a brief episode of tinnitus, diplopia, and dysarthria with no residual effects, the nurse anticipates teaching the patient about a. cerebral aneurysm clipping. b. heparin intravenous infusion. c. oral low-dose aspirin therapy. d. tissue plasminogen activator (tPA).

C. The patient's symptoms are consistent with transient ischemic attack (TIA), and drugs that inhibit platelet aggregation are prescribed after a TIA to prevent stroke. Continuous heparin infusion is not routinely used after TIA or with acute ischemic stroke. The patient's symptoms are not consistent with a cerebral aneurysm. tPA is used only for acute ischemic stroke, not for TIA.

Because Nancy is right-handed and is having difficulty performing activities of daily living with the left arm, the nurse also includes the nursing diagnosis "self-care deficit" in the care plan. Which intervention would the nurse implement to address this nursing diagnosis? A. Use narrow grip utensils to accommodate a weak grasp B. Recommend a regular type toilet seat with grab hand bars C. Utilize plate guards when Nancy is eating D. Discourage Nancy from using assistive devices

C. Utilize plate guards when Nancy is eating - Plate guards prevent food from being pushed off the plate. Using plate guards and other assistive devices will encourage independence in a client with a self-care deficit.

During the change of shift report a nurse is told that a patient has an occluded left posterior cerebral artery. The nurse will anticipate that the patient may have a. dysphasia. b. confusion. c. visual deficits. d. poor judgment.

C. Visual disturbances are expected with posterior cerebral artery occlusion. Aphasia occurs with middle cerebral artery involvement. Cognitive deficits and changes in judgment are more typical of anterior cerebral artery occlusion.

Gail wants to bury her mother beside her father in the local cemetery and tells the RN, I just don't know what I should do. How should the RN respond?

C. You seem really confused about what to do. Would you like to talk about it?

Which intervention should the nurse delegate to the LPN when caring for a patient following an acute stroke? a. assess the patient's neurologic status b. assess the patient's gag reflex before beginning feeding c. administer ordered antihypertensives and platelet inhibitors d. teach the patient's caregivers strategies to minimize unilateral neglect

C: Administer ordered antihypertensives and platelet inhibitors- medication administration is within the scope of practice for an LPN. Assessment and teaching are within the scope of practice for the RN.

Which of the following is the best treatment for acute ischemic stroke? a. heparin b. LMWH c. Alteplase d. Eptifibatie e. Warfarin Which of the following is the best treatment for acute ischemic stroke? a. heparin b. LMWH c. Alteplase d. Eptifibatie e. Warfarin

C: Alteplase

A thrombus that develops in a cerebral artery does not always cause a loss of neurologic function because a. the body can dissolve the atherosclerotic plaques as they form b. some tissues of the brain do not require constant blood supply to prevent damage c. circulation through the circle of Willis may provide blood supply to the affected area of the brain d. neurologic deficits occur only when major arteries are occluded by thrombus formation around an atherosclerotic plaque

C: Circulation through the circle of Willis may provide blood supply to the affected area of the brain. The communication between cerebral arteries in the circle of Willing provides a collateral circulation, which may maintain circulation to an area of the brain if its original blood supply is obstructed. ALl areas of the brain require constant blood supply, and atherosclerotic plaques are not readily reversed. Neurologic deficits can result from ischemia cause by many factors.

The incidence of ischemic stroke in patients with TIAs and other risk factors is reduced with administration of a. furosemide (Lasix) b. lovastatin (Mevacor) c. daily low dose aspirin d. nimodipine (Nimotop)

C: Daily low dose aspirin- the administration of antiplatelet agents, such as aspirin, dipyridamole (Persantine), and ticlopdipine (Ticlid), reduces the incidence of stroke in those at risk. Anticoagulants are also used for prevention of embolic strokes but increase the risk for hemorrhage. Diuretics are not indicated for stroke prevention other than for their role in controlling BP, and antilipemic agents have bot been found to have a significant effect on stroke prevention. The calcium channel blocker nimodipine is used in patients with subarachnoid hemorrhage to decrease the effects of vasospasm and minimize tissue damage. P.S. I freaking love you and good luck on the final!!

A patient with a stroke has a right sided hemiplegia. The nurse prepares family members to help control behavior changes seen with this type of stroke by teaching them to a. ignore undesirable behaviors manifested by the patient b. provide directions to the patient verbally in small steps c. distract the patient from inappropriate emotional responses d. supervise all activities before allowing the patient to pursue them independently

C: Distract the patient from inappropriate emotional responses- patients with left-sided brain damage from stroke often experience emotional lability, inappropriate emotional responses, mood swings, and uncontrolled tears or laughter disproportionate or out of context with the situation. The behavior is upsetting and embarrassing to both the patient and the family, and the patient should be distracted to minimize its presence. Patients with right-brain damage often have impulsive, rapid behavior that supervision and direction.

In promoting health maintenance for prevention of strokes, the nurse understands that the highest risk for the most common type of stroke is present in a. African Americans b. women who smoke c.individuals with hypertension and diabetes d. those who are obese with high dietary fat intake

C: Individuals with hypertension and diabetes- The highest risk factors for thrombotic stroke are hypertension and diabetes. African Americans have a higher risk for stroke than do white persons but probably because they have a greater incidence of hypertension. Factors such as obesity, diet high in saturated fats and cholesterol, cigarette smoking, and excessive alcohol use are also risk factors but carry less risk than hypertension.

A carotid endarterectomy is being considered as a treatment for a patient who has had several TIAs. The nurse explains to the patient that this surgery a. is used to restore blood to the brain following an obstruction of a cerebral artery b. involves intracranial surgery to join a superficial extracranial artery to an intracranial artery c. involves removing an atherosclerotic plaque in the carotid artery to prevent an impending stroke d. is sued to open a stenosis in a carotid artery with a balloon and stent to restore cerebral circulation

C: Involves removing an atherosclerotic plaque in the carotid artery to prevent an impending stroke- An endarterectomy is a removal of an atherosclerotic plaque, and plaque in the carotid artery may impair circulation enough to cause a stroke. A carotid endarterectomy is performed to prevent a cerebrovascular accident (CVA), as are most other surgical procedures. An extacranial-intracranial bypass involves cranial surgery to bypass a sclerotic intacranial artery. Percutaneous transluminal angioplasty uses a balloon to compress stenotic areas in the carotid and vertebrobasilar arteries and often includes inserting a stent to hold the artery open.

A patient with right hemisphere stroke has a nursing diagnosis of unilateral neglect related to sensory perceptual deficits. During the patient's rehabilitation, it is important for the nurse to a. avoid positioning the patient on the affected side b. place all objects for care on the patient's unaffected side c. teach the patient to care consciously for the affected side d. protect the affected side from injury with pillows and supports

C: Teach the patient to care consciously for the affected side- unilateral neglect, or neglect syndrome, occurs when the patient with a stroke is unaware of the affected side of the body, which puts the patient at risk for injury. During the acute phase, the affected side is cared for by the nurse with positioning and support, during rehabilitation the patient is taught to care consciously for and attend to the affected side of the body to protect it from injury. Patients may be positioned on the affected side for up to 30 minutes.

The neurologic functions that are affected by a stroke are primarily related to a. the amount of tissue area involved b. the rapidity of onset of symptoms c. the brain area perfused by the affected artery d. the presence or absence of collateral circulation

C: The brain area perfused by the affected artery- clinical manifestation of altered neurologic function differ, depending primarily on the specific cerebral artery involved and the area of the brain that is perfused by the artery. The degree of impairment depends on rapidity of onset, the size of the lesion, and the presence of collateral circulation.

An appropriate food for a patient with a stroke who has mild dysphagia is a. fruit juices b. pureed meat c. scrambled eggs d. fortified milkshakes

C: scrambled eggs- soft foods that provide enough texture, flavor, and bulk to stimulate swallowing should be used for the patient with dysphasia. Thin liquids are difficult to swallow, and patients may not be able to control them in the mouth. Pureed foods are often too bland and to smooth, and milk products should be avoided because they tend to increase the viscosity of mucus and increase salivation.

Which drug can cause a sudden hemorrhage stroke?

Cocaine use is one example of a trigger for sudden, dramatic BP elevation leading to hemorrhagic stroke.

Question 1 Type: MCSA The nurse is discussing stroke etiology with a community group. The nurse would describe which mechanism as causing the most common kind of stroke? 1. Ischemia 2. Hemorrhage 3. Headache 4. Spasm

Correct Answer: 1 Rationale 1: Eighty percent of all strokes are caused by ischemia. Rationale 2: Hemorrhagic strokes are less common than another type of stroke. Rationale 3: Headache is a symptom related to stroke but is not a causative mechanism. Rationale 4: Some strokes are caused by vasospasm, but they are not the predominant type.

Question 31 Type: MCSA Results of diagnostic procedures reveal that a patient's cerebral arteriovenous malformation (AVM) is Spetzler- Martin grade 1. The nurse anticipates which most common treatment for this patient? 1. Surgical resection 2. Endovascular coiling 3. Radiosurgery 4. Watchful waiting

Correct Answer: 1 Rationale 1: If there is no condition that would preclude surgery, patients with Spetzler-Martin grade 1 or 2 AVM are generally treated with surgical resection. Rationale 2: Endovascular coiling may be indicated, but this is not the most common treatment approach to this AVM. Rationale 3: Gamma knife radiosurgery is an option, but this treatment is generally reserved for surgically inaccessible lesions. Rationale 4: Conservative treatment or "watchful waiting" is reserved for higher-grade AVM because treatment is felt to be unsafe.

Question 18 Type: MCSA The nurse is caring for a patient who is at risk for developing cerebral vasospasm. The nurse recognizes which order as inappropriate for this patient? 1. Discontinue IV fluids when tolerating fluids. 2. Monitor serum electrolytes daily. 3. Hold antihypertensive medications. 4. Monitor blood pressure hourly.

Correct Answer: 1 Rationale 1: Patients with subarachnoid hemorrhage are at significant risk for vasospasm. IV fluids should not be discontinued even if the patient has adequate oral intake. Rationale 2: Monitoring serum electrolytes, especially sodium, is appropriate. Rationale 3: Antihypertensive medications should not be administered without parameters for administration. Rationale 4: Blood pressure should be monitored at least hourly.

Question 28 Type: MCSA After a stroke, a patient was left with painful feet and hands. The physician diagnoses central pain syndrome. Which home management technique should the nurse teach this patient? 1. "Keep your feet and hands warm." 2. "Walking will improve your comfort." 3. "Taking an ACE inhibitor will decrease the pain." 4. "Ice massage may help your pain."

Correct Answer: 1 Rationale 1: Temperature change, especially cold temperature, makes the pain worse. Rationale 2: Movement often makes pain worse. Rationale 3: ACE inhibitors are prescribed for blood pressure control. Rationale 4: Cold temperatures often make the pain more severe.

Question 29 Type: MCSA A patient is admitted to the emergency department after complaining of a headache and having a seizure. On CT examination the patient is assigned a Fisher score of 1. How does the nurse evaluate this information? 1. There is no blood detected on the CT scan. 2. The CT scan shows no ischemia. 3. The CT scan must be repeated with contrast. 4. The aneurysm can be treated with coiling.

Correct Answer: 1 Rationale 1: The Fisher grading scale is based on the amount of blood on the CT scan. Rationale 2: The Fisher scale is not associated with ischemia. Rationale 3: The Fisher scan does not determine the quality of the CT scan. Rationale 4: The Fisher grading scale does not address the choice of treatment.

Question 15 Type: MCMA A diagnosis of cerebral salt wasting is made for a patient who recently experienced a subarachnoid hemorrhage (SAH) and is hyponatremic. The nurse recognizes the importance of which interventions? Note: Credit will be given only if all correct choices and no incorrect choices are selected. Standard Text: Select all that apply. 1. Strictly monitoring intake and output 2. Monitoring hypertonic IV fluid therapy 3. Securing a serum sodium level every 6 hours 4. Restricting the patient's fluids to 200 mL daily 5. Administering oral salt supplements

Correct Answer: 1,2,3,5 Rationale 1: It is important for the patient's fluid intake and output to be carefully monitored. Rationale 2: Poor management of hypertonic fluid intake can result in cardiac overload problems. Rationale 3: It is very important for the patient's serum sodium levels to be monitored frequently, every 6 hours if the patient is receiving hypertonic saline. Rationale 4: Fluid restriction in this setting is absolutely contraindicated. Rationale 5: Generally, the treatment for hyponatremia in SAH is hypertonic saline with oral salt supplements, if possible.

Question 17 Type: MCMA The nurse is caring for a patient who is being treated for cerebral vasospasm with a medical treatment known as triple-H therapy. Which assessment data confirm that the treatment is currently effective? Note: Credit will be given only if all correct choices and no incorrect choices are selected. Standard Text: Select all that apply. 1. Mean arterial pressure (MAP) 110 mmHg 2. Hematocrit 34% 3. Sodium of 125 mg/dL 4. Urine output 40 ml/hr 5. BP 170/96

Correct Answer: 1,2,3,5 Rationale 1: Mean arterial pressure (MAP) should be between 60 and 150 mmHg. Rationale 2: Hematocrit should be greater than 30% to reflect good cerebral perfusion. Rationale 3: Sodium level is not an assessment value used to monitor triple-H therapy. Rationale 4: Urine output is not considered an assessment value for monitoring triple-H therapy. Rationale 5: Triple-H therapy consists of hypertension, hemodilution, and hypervolemia. The goal of triple-H therapy is to increase cerebral perfusion pressure and CBF, therefore reducing the risk for further neurological deficits. Systolic blood pressure should be kept at no less than 160 mmHg.

Question 6 Type: MCMA A patient has been hospitalized for scheduled repair of an intracranial aneurysm. The nurse caring for the patient prior to surgery would recognize which manifestations as indicating the aneurysm may be leaking or may have ruptured? Note: Credit will be given only if all correct choices and no incorrect choices are selected. Standard Text: Select all that apply. 1. Visual deficits 2. Headache 3. Mild nausea 4. Dilated pupil 5. Stiff neck

Correct Answer: 1,2,4 Rationale 1: The most common cranial nerve signs are dilated pupil, decreased mobility of the eye, and ptosis. Rationale 2: The most common complaint is "the worst headache of my life." Rationale 3: Nausea is not a prodromal sign, but it may be present once rupture has occurred. Rationale 4: Pupil dilation is a common prodromal finding. Rationale 5: Stiff neck is not a prodromal sign of intracranial aneurysm.

Question 11 Type: MCMA The nurse is reinforcing education regarding the risks of bypass procedures for a patient who has had an ischemic stroke. What risks should the nurse discuss? Note: Credit will be given only if all correct choices and no incorrect choices are selected. Standard Text: Select all that apply. 1. Infection 2. Unstable blood pressure 3. Renal failure 4. Thrombotic stroke 5. Hemorrhage

Correct Answer: 1,2,4,5 Rationale 1: All surgical procedures, including bypass procedures, carry risks of infection and alteration in wound healing. Rationale 2: Blood pressure instability can occur as a result of hemorrhage. Rationale 3: Renal failure is not a general postprocedure risk after bypass surgery. Rationale 4: Bypass procedures carry significant risk to the patient, such as stroke due to a clot in the graft or to a variety of other factors. Rationale 5: Bypass procedures carry significant risk to the patient, including hemorrhage of the vessel.

Question 19 Type: MCMA A patient has infarct of the right anterior cerebral artery. The nurse would attribute which assessment findings to this condition? Note: Credit will be given only if all correct choices and no incorrect choices are selected. Standard Text: Select all that apply. 1. The patient is incontinent of urine. 2. The patient cannot move the right leg. 3. The patient does not respond to light touch on the feet. 4. The patient cannot speak. 5. The patient has paralysis of the right arm.

Correct Answer: 1,3 Rationale 1: Incontinence is associated with infarct of the anterior cerebral artery. Rationale 2: Anterior cerebral artery infarct would result in paresis of the left leg. Rationale 3: Sensory deficits in the lower extremities are a finding associated with anterior cerebral artery infarct. Rationale 4: Aphasia is not associated with infarction of the anterior cerebral artery. Rationale 5: Paralysis of the upper extremities is not associated with infarct of the anterior cerebral artery.

Question 16 Type: MCMA A patient is being treated for an aneurysmal subarachnoid hemorrhage (SAH) that occurred 10 days ago. The nurse recognizes that the patient is at risk for decreased cerebral blood flow and is especially concerned when which assessments are made? Standard Text: Select all that apply. 1. The patient seems unable to verbalize needs. 2. The patient has difficulty starting the flow of urine. 3. The patient reports a stiff neck. 4. The patient has a temperature of 101°F. 5. The patient has unequal but reactive pupils.

Correct Answer: 1,3,4 Rationale 1: The peak incidence of vasospasm is 3 to 14 days from the incidence of SAH, although it may occur up to 21 days after the bleed occurs. Signs of vasospasm include aphasia. Rationale 2: Signs of vasospasm do not include urinary hesitation. Rationale 3: The peak incidence of vasospasm is 3 to 14 days from the incidence of SAH, although it may occur up to 21 days after the bleed occurs. Signs of vasospasm include neck stiffness. Rationale 4: The peak incidence of vasospasm is 3 to 14 days from the incidence of SAH, although it may occur up to 21 days after the bleed occurs. Signs of vasospasm include fever. Rationale 5: Signs of vasospasm do not include abnormal pupil size or reactivity.

Question 9 Type: MCSA After being informed that their father has experienced a stroke that has affected a portion of his cerebrum, the patient's family asks the nurse, "What effect will aphasia have on his life?" How should the nurse respond? 1. "He will likely become depressed until he adjusts to the dysfunction." 2. "You'll have to speak very loudly when you talk." 3. "The way he communicates with you will change." 4. "Perhaps you should learn about the different options for speech therapy."

Correct Answer: 3 Rationale 1: Depression may occur, but it is not related to the patient's problems with verbal communication. Rationale 2: Speaking loudly is more related to a hearing problem and would have little impact on the patient's ability to communicate verbally. Rationale 3: Aphasia is the term used to denote problems with verbal communication. Broca's area in the cerebrum regulates verbal expression, the ability to say words. Rationale 4: Speech therapy may be an option for rehabilitation, but this response does not address the family's question.

Question 10 Type: MCMA The nurse is preparing an educational program focusing on various types of strokes. What types of strokes would the nurse include in a discussion of ischemic strokes? Note: Credit will be given only if all correct choices and no incorrect choices are selected. Standard Text: Select all that apply. 1. Cardioembolic 2. Intracerebral hemorrhagic 3. Lacunar 4. Subarachnoid hemorrhagic 5. Atherosclerotic thrombotic

Correct Answer: 1,3,5 Rationale 1: Cardioembolic strokes are considered ischemic because they restrict or obstruct blood flow to a portion of the brain, resulting in cellular death. Rationale 2: Intracerebral hemorrhagic strokes are a result of a severe blood loss from the vascular system of the brain. Rationale 3: Lacunar strokes are considered ischemic because they restrict or obstruct blood flow to a portion of the brain, resulting in cellular death. Rationale 4: Subarachnoid hemorrhagic strokes are a result of a severe blood loss from the vascular system of the brain. Rationale 5: Atherosclerotic thrombotic strokes are considered ischemic because they restrict or obstruct blood flow to a portion of the brain, resulting in cellular death.

Question 13 Type: MCSA An emergency department nurse is caring for a patient who has been diagnosed with an evolving ischemic stroke. The nurse anticipates preparing the patient and family for which initial medical intervention? 1. Craniotomy 2. Administration of t-PA (tissue plasminogen activator) 3. Full-body X-ray series 4. Watchful waiting over the next 24 hours

Correct Answer: 2 Rationale 1: A craniotomy evacuates a hematoma to relieve mass effect resulting from a hemorrhagic stroke. Rationale 2: The administration of t-PA does not affect the infarcted, necrotic core, but may revitalize the penumbra and limit the extent of damage caused by ischemic stroke, thus minimizing the effects of the stroke. Rationale 3: There is no indication that full-body X-rays are required. Rationale 4: If an ischemic stroke is evolving, watchful waiting will allow it to progress.

Question 4 Type: MCSA A post-stroke patient is going home on oral Coumadin (warfarin). During discharge teaching, which statement by the patient reflects an understanding of the effects of this medication? 1. "I will stop taking this medicine if I notice any bruising." 2. "I will not eat spinach while I'm taking this medicine." 3. "It will be OK for me to eat anything." 4. "I'll check my blood pressure frequently while taking this medication."

Correct Answer: 2 Rationale 1: Bruising is a common side effect, and the drug should not be stopped unless by prescriber order. Rationale 2: Warfarin is a vitamin K antagonist. Green, leafy vegetables contain vitamin K and will therefore interfere with the therapeutic effects of the drug. Rationale 3: Fatty foods interfere with warfarin therapy. Rationale 4: This medication does not affect the blood pressure.

Question 12 Type: MCSA A 27-year-old male arrives at the emergency department reporting a sudden severe-onset headache. Diagnostic studies confirm he is experiencing an intracerebral hemorrhage. The nurse recognizes which risk factor for this condition in the patient's history? 1. The patient reports having a sprained ankle last week, for which he took ibuprofen. 2. The patient reports smoking a pack of cigarettes a day since he was 16 years old. 3. The patient is 27 years old. 4. The patient is male.

Correct Answer: 2 Rationale 1: Ibuprofen use is not a specific risk for intracerebral hemorrhage. Rationale 2: Smoking is a risk factor for the development of aneurysms. Rationale 3: Aneurysms are more common in those over 40. Rationale 4: Aneurysms are more common in women.

Question 22 Type: MCSA A patient is diagnosed with a lacunar stroke. The patient's daughter says, "If this is a stroke in a small artery, why is my dad's movement so affected?" Which nursing response is indicated? 1. "Lacunar strokes occur in large arteries, not small ones." 2. "Lacunar strokes look small on the scans, but they often happen in areas where lots of motor and sensory tracts are located." 3. "Most of the time with lacunar strokes, we see more sensory effects than those your dad has experienced." 4. "Lacunar strokes occur in small arteries, but the damage seen on scans is massive."

Correct Answer: 2 Rationale 1: Lacunar strokes are also known as small artery occlusive disease. Rationale 2: Lacunar strokes often occur in areas that are rich in motor and sensory tracts. This results in a small area of damage causing a large effect. Rationale 3: Lacunar syndromes can be subdivided into pure motor, pure sensory, sensorimotor, and ataxic hemiparesis. Rationale 4: Lacunar strokes look small on imaging.

Question 25 Type: FIB A patient experienced onset of weakness, left-sided facial drooping, and difficulty talking 2 hours before presenting in the emergency department. The patient has been in the ED for 30 minutes, and the nurse is aware that t-PA treatment for stroke must be administered within the next _______ hours to be effective. Standard Text:

Correct Answer: 2 Rationale : The t-PA must be administered within 4.5 hours of the onset of symptoms. This patient waited 2 hours before coming to the ED and has been in the ED for 30 minutes, for a total of 2.5 hours. 4.5-2.5 = 2 hours

Question 7 Type: MCMA A patient has undergone repair of a subarachnoid hemorrhage. The nurse should monitor this patient for development of which complications? Note: Credit will be given only if all correct choices and no incorrect choices are selected. Standard Text: Select all that apply. 1. Hypernatremia 2. Hydrocephalus 3. Rebleeding 4. Vasospasm 5. Hemodilution

Correct Answer: 2,3,4 Rationale 1: Hyponatremia is a more common complication. Rationale 2: Hydrocephalus is a major complication that a nurse must anticipate following a ruptured intracranial aneurysm. Rationale 3: Rebleeding is a major complication that a nurse must anticipate following a ruptured intracranial aneurysm. Rationale 4: Vasospasm is a major complication that a nurse must anticipate following a ruptured intracranial aneurysm. Rationale 5: Hemodilution is part of the treatment of cerebral vasospasm.

Question 32 Type: MCMA Diagnostic testing has revealed that the patient has an arteriovenous malformation (AVM). Surgical treatment is scheduled for tomorrow. Which critical assessment findings should the nurse report to the surgeon? Note: Credit will be given only if all correct choices and no incorrect choices are selected. Standard Text: Select all that apply. 1. Headache unchanged since admission 2. Seizure 3. A period of sudden clarity after previously disturbed thought processing 4. Increased difficulty in talking 5. Sudden decrease in level of consciousness

Correct Answer: 2,4,5 Rationale 1: Headache is a common finding in AVM and may be the reason the patient presented for treatment. The headache will likely continue until the AVM is treated. Rationale 2: Because AVMs often affect cortical brain, seizures are common. Rationale 3: There is no indication that a period of lucidity will change the patient's treatment plan. The AVM poses a lifelong risk of rupture and should be treated. Rationale 4: Increased difficulty talking may indicate that the AVM is leaking. Rationale 5: A sudden decrease in LOC indicates possible rupture of the AVM, which is an emergency.

Question 5 Type: MCSA The nurse is teaching a wellness class and is discussing the warning signs of stroke. A patient asks, "What is the most important thing for me to remember?" Which is an appropriate response by the nurse? 1. "Know your family history." 2. "Keep a list of your medications." 3. "Be alert for sudden weakness or numbness." 4. "Call 911 if you notice a gradual onset of paralysis or confusion."

Correct Answer: 3 Rationale 1: Family history and past medical history can be indicators for risk, but they are not warning signs of stroke. Rationale 2: Keeping a list of medications will not assist in identifying a stroke. Rationale 3: Warning signs of stroke include sudden weakness, paralysis, loss of speech, confusion, dizziness, unsteadiness, and loss of balance. The key word is sudden. Rationale 4: Gradual onset of symptoms is not indicative of a stroke.

Question 3 Type: MCSA A hospitalized patient has become unresponsive. The left side of the body is flaccid. The attending physician believes the patient may have had a hemorrhagic stroke. What is the nurse's priority intervention? 1. Move the patient to the critical care unit. 2. Assess blood pressure. 3. Assess the airway and breathing. 4. Observe urinary output.

Correct Answer: 3 Rationale 1: Moving the patient to the critical care unit is not a priority intervention. Rationale 2: Blood pressure assessment is an important intervention but not the most vital. Rationale 3: In any unconscious patient, the airway must be protected. Assessment of the current airway and breathing status is of highest priority and will continue to be. Rationale 4: Urinary output assessment is an important intervention but not the most vital.

Question 20 Type: MCSA A patient's initial assessment revealed blood pressure 128/70, pulse 68, respirations 20, and pupils equal and reactive. The patient is awake and responding to verbal stimuli but demonstrating weakness on the left side. A medical diagnosis is presumptive stroke. Two hours later the patient is not awake but is easily aroused, has a blood pressure of 140/70, pulse 52, respirations 18, and the left pupil now reacts more slowly to light than the right pupil. Which complication is likely to have occurred? 1. Reperfusion injury 2. Normal stabilization poststroke 3. Increased intracranial pressure 4. Impending brain death

Correct Answer: 3 Rationale 1: Reperfusion injury results in further injury to already damaged tissue that is compromised and may be seen most often with thrombolytic therapy. There is no indication that this patient has had thrombolytic therapy. Rationale 2: The most current assessment findings indicate that the patient's condition is becoming more unstable. Rationale 3: Rising systolic blood pressure, falling pulse, and a pupil that has become sluggish suggest increasing intracranial pressure (IICP) and requires notification of the health care provider. Rationale 4: Brain death is diagnosed by a lack of brain waves and inability to maintain vital function.

Question 34 Type: MCSA The nurse is planning care for a patient undergoing diagnostic evaluation for presence of an intracranial bleed. Which nursing diagnosis (NDX) would the nurse set as the first priority? 1. Impaired Verbal Communication 2. Potential for Aspiration 3. Risk for Altered Cerebral Tissue Perfusion 4. Altered Coping

Correct Answer: 3 Rationale 1: The patient may have impairment of verbal communication, so this is an appropriate NDX but is not the priority.

Question 24 Type: MCSA The patient with assessment findings of stroke has a normal CT scan upon admission to the emergency department. How should the nurse evaluate this information? 1. Intracranial aneurysm is a more likely diagnosis. 2. The patient must have a psychosomatic illness. 3. A second CT done later may show damage. 4. The patient should have an electroencephalogram.

Correct Answer: 3 Rationale 1: There is no evidence that this is true. Rationale 2: The patient likely has a physiological illness. Rationale 3: Initial CT scans often do not show infarcted tissue even though an occluded artery is present. Rationale 4: Electroencephalograms are useful in detecting abnormal brain activity such as seizure.

Question 30 Type: FIB The nurse is monitoring a patient who has had a subarachnoid bleed. The nurse plans care based on the knowledge that the risk for cerebral vasospasm is highest in Fisher grade _______ aneurysms. Standard Text:

Correct Answer: 3 Rationale : Fisher grade 3 aneurysms have a localized clot or a layer of blood greater than 1 mm thick. This configuration makes cerebral vasospasm more likely due to irritation of tissues.

Question 2 Type: MCSA A patient is admitted with signs of a stroke (CVA). On admission, vital signs were blood pressure 128/70, pulse 68, and respirations 20. Two hours later the patient is not awake, has a blood pressure of 170/70, pulse 52, and the left pupil is now slower than the right pupil in reacting to light. These findings suggest which condition? 1. Impending brain death 2. Decreasing intracranial pressure 3. Stabilization of the patient's condition 4. Increasing intracranial pressure

Correct Answer: 4 Rationale 1: Brain death is diagnosed by a lack of brain waves and inability to maintain vital function. Rationale 2: Rising systolic blood pressure, falling pulse, and a pupil that has become sluggish suggest increasing another condition. Rationale 3: This is an emergency situation that requires intervention, as the patient's condition is becoming more unstable. Rationale 4: Rising systolic blood pressure, falling pulse, and a pupil that has become sluggish suggest increasing intracranial pressure (IICP). This is an emergency situation that requires notification of the physician.

Question 33 Type: MCSA A patient with a family history of cavernous malformation presents to the emergency department following a seizure. The nurse anticipates which diagnostic testing? 1. Angiography 2. CT scan 3. Skull series X-ray 4. MRI scan

Correct Answer: 4 Rationale 1: Cavernous malformations are low-flow lesions, and angiography is not diagnostic. Rationale 2: A CT scan is not the most diagnostic tool available. Rationale 3: Skull series X-ray is not the most diagnostic tool available. Rationale 4: MRI scanning is the diagnostic tool of choice when cavernous malformation is suspected.

Question 8 Type: MCSA Which assessment data alerts the nurse to the fact that the patient is at risk for an embolic stroke? 1. Blood sugar of 110 mg 2. Right partial lung lobectomy 6 months ago 3. BP 108/68 4. History of atrial fibrillation

Correct Answer: 4 Rationale 1: Normal blood sugar is not a risk factor for embolic stroke. Rationale 2: Partial lobectomy is not a risk factor for embolic stroke. Rationale 3: Hypotension is not a risk factor for embolic stroke. Rationale 4: Embolisms from cardiac sources are referred to as cardiogenic embolisms. The most common cause is atrial fibrillation, which accounts for almost 10% of all ischemic (embolic) strokes.

Question 26 Type: MCSA A patient is scheduled for a lumbar puncture to assess for subarachnoid hemorrhage. The nurse should check to see if which test has been performed prior to the procedure? 1. Serum glucose 2. BUN 3. Chest X-ray 4. CT scan of the head

Correct Answer: 4 Rationale 1: While many laboratory tests will probably be done on this patient, serum glucose is not the most critical. Rationale 2: While many laboratory tests will probably be done on this patient, BUN is not the most critical. Rationale 3: While a chest X-ray may be done on this patient, it is not the most critical test. Rationale 4: Lumbar puncture should not be performed until the presence of a space-occupying lesion is ruled out.

Question 21 Type: MCMA A patient will have either endovascular coiling or neurosurgical clipping as treatment for an intracranial aneurysm. How would the nurse describe the benefits and risks of each form of treatment? Note: Credit will be given only if all correct choices and no incorrect choices are selected. Standard Text: Select all that apply. 1. Surgery is the most reliable method of treating these aneurysms. 2. Coiling is only successful in very small aneurysms. 3. The decision of which approach to use is based on the patient's preference. 4. Coiling is generally a less invasive option. 5. Coiling may require placement of a stent.

Correct Answer: 4,5 Rationale 1: Both surgery and coiling are acceptable methods of treating intracerebral aneurysms, and each has specific indications for use. Rationale 2: The size of the aneurysm is not as important as its configuration and the configuration of vessels in the immediate vicinity. Rationale 3: The decision of which approach to use is based largely on the angiographic features of the aneurysm. Rationale 4: Coiling is a nonsurgical approach and is less invasive than craniotomy. Rationale 5: If the aneurysm is broad based, a stent may be placed in the parent artery to act as a scaffold for the coils.

Question 14 Type: MCMA The nurse is caring for a patient who experienced an ischemic stroke 8 hours ago. The nurse would expect an order to administer which medications designed to prevent further obstruction of vascular cerebral blood flow? Standard Text: Select all that apply. 1. Intravenous dopamine 2. Intravenous mannitol 3. Subcutaneous insulin 4. Intravenous heparin 5. Subcutaneous low-molecular-weight heparin

Correct Answer: 4,5 Rationale 1: Intravenous dopamine is a vasopressor directed at stabilizing blood pressure. Rationale 2: Intravenous mannitol is used to reduce intracranial pressure and brain mass. Rationale 3: Subcutaneous insulin may be used to manage hyperglycemia resulting from the ineffective utilization of glucose. Rationale 4: Anticoagulation with IV infusion of heparin for several days during the acute care management of a patient with ischemic stroke is common. Rationale 5: Subcutaneous low-molecular-weight heparin (LMWH) can be used to manage a patient after ischemic stroke.

_____ a classic but late sign of increased ICP

Cushing's triad

A patient presents to the emergency department with signs and symptoms of an ischemic stroke. What is the priority factor when considering fibrinolytic therapy? a. Age older than 80 years b. History of stroke c. Recent surgery d. Time since onset of symptoms

D

Blood flow to the brain remains fairly constant as a result of which process? a. Autostasis b. Automobilization c. Hemodynamic stasis d. Autoregulation

D

Following a left hemisphere stroke, the patient has expressive (Broca's) aphasia. Which intervention is best to use when communicating with this patient? a. Repeat the names of objects on a routine basis. b. Face the patient and speak slowly and clearly. c. Obtain a whiteboard with an erasable marker. d. Develop a picture board that has objects and activities.

D

The nurse is caring for a patient with an ischemic stroke. Which position is the patient placed in according to current nursing practice? a. Head of the bed is elevated 25 to 30 degrees b. Head of the bed is elevated to 45 degrees c. Supine with hips in flexed position d. The best head of bed position has not been determined

D

The nurse who is providing postoperative care for a patient who had a craniotomy immediately notifies the surgeon of which assessment finding? a. Drainage in the Jackson-Pratt container of 45 mL/8 hours b. Intracranial pressure of 15 mm Hg c. Pco₂ level of 35 mm Hg d. Serum sodium of 117 mEq/L

D

What clinical manifestation further supports this assessment?

D) Global aphasia. Rationale: Global aphasia refers to difficulty speaking, listening, and understanding, as well as difficulty reading and writing. Symptoms vary from person to person. Aphasia may occur secondary to any brain injury involving the left hemisphere. Visual field deficits, spatial-perceptual deficits, and paresthsia of the left side usually occur with right-sided brain attack.

The nurse continues to monitor Nancy's condition closely. Which finding would require immediate intervention by the nurse? A. Nancy's pulse oximeter reading is greater than 95% B. Nancy's serum potassium level is 3.9 mEq/L C. Nancy's telemetry shows normal sinus rhythm with occasional premature ventricular contractions D. Nancy's cardiac output is less than 4 L/min

D. Nancy's cardiac output is less than 4 L/min - The normal range for cardiac output to ensure cerebral blood flow and oxygen delivery is 4 to 8 L/min.

cerebrovascular accident (CVA)

term used to describe a stroke

The nurse is teaching a senior citizen's group about signs and symptoms of a stroke. Which statement by the nurse would provide accurate information? A. "Take the person to the hospital if a headache lasts for more than 24 hours." B. "Stroke symptoms usually start when the person is awake and physically active." C. "A person with a transient ischemic attack has mild symptoms that will go away." D. "Call 911 immediately if a person develops slurred speech or difficulty speaking."

D. "Call 911 immediately if a person develops slurred speech or difficulty speaking." Medical assistance should be obtained immediately for someone with signs and symptoms of a stroke such as sudden numbness; weakness; paralysis of the face, arm, or leg (especially on one side of the body); sudden confusion; trouble speaking or understanding; slurred speech; sudden trouble seeing in one or both eyes; sudden trouble walking; dizziness; loss of balance or coordination; or a sudden, severe headache with no known cause. A person with signs and symptoms of a transient ischemic attack should seek medical attention immediately because it is unknown if the symptoms will resolve or persist and progress to a stroke. Onset of signs and symptoms of a stroke vary depending on the type. Onset of an ischemic thrombotic stroke usually occurs at rest. Onset of an ischemic embolic stroke is not related to rest or activity, and onset of a hemorrhagic stroke usually occurs with activity.

brain attack

term used to describe a stroke; communicates the urgency of recognizing the clinical manifestations of a stroke and treating a medical emergency

As the nurse assesses Nancy, Gail asks, "Why isn't my mother a candidate for thrombolytic therapy?" A. "Since your mother was alert on admission, she is not a candidate to receive this medication. B. "I think that is something you should discuss with your mother's healthcare provider." C. "tPA is usually not administered to anyone older than 65 years." D. "She is not a candidate because of therapeutic time constraints related to this medication."

D. "She is not a candidate because of therapeutic time constraints related to this medication." - Thrombolytic therapy is contraindicated in clients with symptom onset longer than 3 hours prior to admission. Nancy had symptoms for 24 hours before being brought to the medical center.

Of the following patients, the nurse recognizes that the one with the highest risk for stroke is a(n): A. obese 45-year old Native American. B. 35-year-old Asian American woman who smokes. C. 32-year-old white woman taking oral contraceptives. D. 65-year-old African American man with hypertension.

D. 65-year-old African American man with hypertension. Nonmodifiable risk factors for stroke include age (older than 65 years), male gender, ethnicity or race (incidence is highest in African Americans; next highest in Hispanics, Native Americans/Alaska Natives, and Asian Americans; and next highest in white people), and family history of stroke or personal history of a transient ischemic attack or stroke. Modifiable risk factors for stroke include hypertension (most important), heart disease (especially atrial fibrillation), smoking, excessive alcohol consumption (causes hypertension), abdominal obesity, sleep apnea, metabolic syndrome, lack of physical exercise, poor diet (high in saturated fat and low in fruits and vegetables), and drug abuse (especially cocaine). Other risk factors for stroke include a diagnosis of diabetes mellitus, increased serum levels of cholesterol, birth control pills (high levels of progestin and estrogen), history of migraine headaches, inflammatory conditions, hyperhomocystinemia, and sickle cell disease.

A 58-year-old patient with a left-brain stroke suddenly bursts into tears when family members visit. The nurse should a. use a calm voice to ask the patient to stop the crying behavior. b. explain to the family that depression is normal following a stroke. c. have the family members leave the patient alone for a few minutes. d. teach the family that emotional outbursts are common after strokes.

D. Patients who have left-sided brain stroke are prone to emotional outbursts that are not necessarily related to the emotional state of the patient. Depression after a stroke is common, but the suddenness of the patient's outburst suggests that depression is not the major cause of the behavior. The family should stay with the patient. The crying is not within the patient's control and asking the patient to stop will lead to embarrassment.

The nurse in a primary care provider's office is assessing several patients today. Which patient is most at risk for a stroke? A. A 92-year-old female who takes warfarin (Coumadin) for atrial fibrillation. B. A 28-year-old male who uses marijuana after chemotherapy to control nausea. C. A 42-year-old female who takes oral contraceptives and has migraine headaches. D. A 72-year-old male who has hypertension and diabetes mellitus and smokes tobacco.

D. A 72-year-old male who has hypertension and diabetes mellitus and smokes tobacco. Stroke risk increases after 65 years of age. Strokes are more common in men. Hypertension is the single most important modifiable risk factor for stroke. Diabetes mellitus is a significant stroke risk factor; and smoking nearly doubles the risk of a stroke. Other risk factors include drug abuse (especially cocaine), high-dose oral contraception use, migraine headaches, and untreated heart disease such as atrial fibrillation.

Which condition is considered a non-modifiable risk factor for a brain attack? A. High cholesterol levels B. Obesity C. History of atrial fibrillation D. Advanced age

D. Advanced age - People over age 55 are a high-risk group for a brain attack because the incidence of stroke more than doubles in each successive decade of life. Non-modifiable risk factor means the client cannot do anything to change the risk factor.

A 74-year-old man who has right-sided extremity paralysis related to a thrombotic stroke develops constipation. Which action should the nurse take first? A. Assist the patient to the bathroom every 2 hours. B. Provide incontinence briefs to wear during the day. C. Administer a bisocodyl (Dulcolax) rectal suppository every day. D. Arrange for several servings per day of cooked fruits and vegetables.

D. Arrange for several servings per day of cooked fruits and vegetables. Patients after a stroke frequently have constipation. Dietary management includes the following: fluid intake of 2500 to 3000 mL daily, prune juice (120 mL) or stewed prunes daily, cooked fruit three times daily, cooked vegetables three times daily, and whole-grain cereal or bread three to five times daily. Patients with urinary incontinence should be assisted to the bathroom every 2 hours when appropriate. Suppositories may be ordered for short-term management if the patient does not respond to increased fluid and fiber. Incontinence briefs are indicated as a short-term intervention for urinary incontinence.

A patient admitted with possible stroke has been aphasic for 3 hours and his current blood pressure (BP) is 174/94 mm Hg. Which order by the health care provider should the nurse question? a. Keep head of bed elevated at least 30 degrees. b. Infuse normal saline intravenously at 75 mL/hr. c. Administer tissue plasminogen activator (tPA) per protocol. d. Administer a labetalol (Normodyne) drip to keep BP less than 140/90 mm Hg.

D. Because elevated BP may be a protective response to maintain cerebral perfusion, antihypertensive therapy is recommended only if mean arterial pressure (MAP) is >130 mm Hg or systolic pressure is >220 mm Hg. Fluid intake should be 1500 to 2000 mL daily to maintain cerebral blood flow. The head of the bed should be elevated to at least 30 degrees, unless the patient has symptoms of poor tissue perfusion. tPA may be administered if the patient meets the other criteria for tPA use.

When teaching about clopidogrel (Plavix), the nurse will tell the patient with cerebral atherosclerosis a. to monitor and record the blood pressure daily. b. that Plavix will dissolve clots in the cerebral arteries. c. that Plavix will reduce cerebral artery plaque formation. d. to call the health care provider if stools are bloody or tarry.

D. Clopidogrel (Plavix) inhibits platelet function and increases the risk for gastrointestinal bleeding, so patients should be advised to notify the health care provider about any signs of bleeding. The medication does not lower blood pressure, decrease plaque formation, or dissolve clots.

Which nursing diagnosis has the highest priority?

D. Drooling According to Maslow's Hierarchy of Needs, physiological needs should be addressed first. Therefore, Nancy's dysphagia is the highest priority nursing diagnosis since she is at risk for aspiration.

Which nursing care task should the nurse delegate to the UAP?

D. Give Mrs. Jackson a bed bath and change the bed linens The UAP can assist Nancy with bathing and then change the bed linens. This task does not require professional judgment or expertise

Due to her deteriorating condition, Nancy is immediately referred to the neurologist. The ED nurse realizes that Nancy has probably suffered a left-sided brain attack. Which clinical manifestation further supports this assessment? A. Spatial-perceptual deficits. B. Visual field deficit on the left side C. Paresthesia of the left side D. Global aphasia

D. Global aphasia -Global aphasia refers to difficulty speaking, listening, and writing. Symptoms vary from person to person. Aphasia may occur secondary to any brain injury involving the left hemisphere.

Mrs. Jackson is experiencing pain in her right shoulder. The nurse is aware that up to 70% of clients with a brain attack experience severe pain in the shoulder that prevents them from learning new skills. Shoulder function helps clients achieve balance, perform transfer skills, and participate in self-care activities.Which intervention should the nurse implement when addressing this condition?

D. Instruct Mrs. Jackson to clasp the right hand with the left hand and raise both hands above the head This exercise helps prevent "frozen shoulder" and will aid the nurse when moving or positioning the client.

Nancy is experiencing homonymous hemianopsia as a result of her brain attack. Which nursing intervention would the nurse implement address this condition? A. Request that the dietary department thicken all liquids on Nancy's meal and snack trays. B. Turn Nancy every 2 hours and perform active range of motion exercises. C. Speak slowly and clearly to assist Nancy in forming sounds to words D. Place the objects Nancy needs for activities of daily living on the left side of the table.

D. Place the objects Nancy needs for activities of daily living on the left side of the table. - Homonymous hemianopsia is loss of the visual field on the same side as the paralyzed side. This results in the client neglecting that side of the body, so it is beneficial to place objects on that side. Nancy had a left-hemisphere brain attack so her right side is the weak side.

Nurses in change-of-shift report are discussing the care of a patient with a stroke who has progressively increasing weakness and decreasing level of consciousness (LOC). Which nursing diagnosis do they determine has the highest priority for the patient? a. Impaired physical mobility related to weakness b. Disturbed sensory perception related to brain injury c. Risk for impaired skin integrity related to immobility d. Risk for aspiration related to inability to protect airway

D. Protection of the airway is the priority of nursing care for a patient having an acute stroke. The other diagnoses are also appropriate, but interventions to prevent aspiration are the priority at this time.

A patient with left-sided weakness that started 60 minutes earlier is admitted to the emergency department and diagnostic tests are ordered. Which test should be done first? a. Complete blood count (CBC) b. Chest radiograph (Chest x-ray) c. 12-Lead electrocardiogram (ECG) d. Noncontrast computed tomography (CT) scan

D. Rapid screening with a noncontrast CT scan is needed before administration of tissue plasminogen activator (tPA), which must be given within 4.5 hours of the onset of clinical manifestations of the stroke. The sooner the tPA is given, the less brain injury. The other diagnostic tests give information about possible causes of the stroke and do not need to be completed as urgently as the CT scan.

A 73-year-old patient with a stroke experiences facial drooping on the right side and right-sided arm and leg paralysis. When admitting the patient, which clinical manifestation will the nurse expect to find? a. Impulsive behavior b. Right-sided neglect c. Hyperactive left-sided tendon reflexes d. Difficulty comprehending instructions

D. Right-sided paralysis indicates a left-brain stroke, which will lead to difficulty with comprehension and use of language. The left-side reflexes are likely to be intact. Impulsive behavior and neglect are more likely with a right-side stroke.

The female patient has been brought to the ED with a sudden onset of a severe headache that is different from any other headache she has had previously. When considering the possibility of a stroke, which type of stroke should the nurse know is most likely occurring? A. TIA B. Embolic stroke C. Thrombotic stroke D. Subarachnoid hemorrhage

D. Subarachnoid hemorrhage Headache is common in a patient who has a subarachnoid hemorrhage or an intracerebral hemorrhage. A TIA is a transient loss of neurologic function usually without a headache. A headache may occur with an ischemic embolic stroke, but severe neurologic deficits are the initial symptoms. The ischemic thrombotic stroke manifestations progress in the first 72 hours as infarction and cerebral edema increase.

A 56-year-old patient arrives in the emergency department with hemiparesis and dysarthria that started 2 hours previously, and health records show a history of several transient ischemic attacks (TIAs). The nurse anticipates preparing the patient for a. surgical endarterectomy. b. transluminal angioplasty. c. intravenous heparin administration. d. tissue plasminogen activator (tPA) infusion.

D. The patient's history and clinical manifestations suggest an acute ischemic stroke and a patient who is seen within 4.5 hours of stroke onset is likely to receive tPA (after screening with a CT scan). Heparin administration in the emergency phase is not indicated. Emergent carotid transluminal angioplasty or endarterectomy is not indicated for the patient who is having an acute ischemic stroke.

Which rehabilitation team member is responsible for evaluating Nancy's dysphagia? A. The occupational therapist B. The rehabilitation physician C. The case manager D. The speech therapist

D. The speech therapist - The speech therapist evaluates the e client's gag reflex and ability to swallow, then makes recommendations regarding feeding techniques and diet.

A patient in the emergency department with sudden-onset right-sided weakness is diagnosed with an intracerebral hemorrhage. Which information about the patient is most important to communicate to the health care provider? a. The patient's speech is difficult to understand. b. The patient's blood pressure is 144/90 mm Hg. c. The patient takes a diuretic because of a history of hypertension. d. The patient has atrial fibrillation and takes warfarin (Coumadin).

D. The use of warfarin probably contributed to the intracerebral bleeding and remains a risk factor for further bleeding. Administration of vitamin K is needed to reverse the effects of the warfarin, especially if the patient is to have surgery to correct the bleeding. The history of hypertension is a risk factor for the patient but has no immediate effect on the patient's care. The BP of 144/90 indicates the need for ongoing monitoring but not for any immediate change in therapy. Slurred speech is consistent with a left-sided stroke, and no change in therapy is indicated.

Which intervention is most appropriate when communicating with a patient suffering from aphasia following a stroke? A. Present several thoughts at once so that the patient can connect the ideas. B. Ask open-ended questions to provide the patient the opportunity to speak. C. Finish the patient's sentences to minimize frustration associated with slow speech. D. Use simple, short sentences accompanied by visual cues to enhance comprehension.

D. Use simple, short sentences accompanied by visual cues to enhance comprehension. When communicating with a patient with aphasia, the nurse should present one thought or idea at a time. Ask questions that can be answered with a "yes," "no," or simple word. Use visual cues and allow time for the individual to comprehend and respond to conversation.

A nursing intervention is indicated for the patient with hemiplegia is a. the use of a footboard to prevent plantar flexion b. immobilization of the affected arm against the chest with a sling c. positioning the patient in bed with each joint lower than the joint proximal to it d. having the patient perform passive ROM of the affected limb with the unaffected limb

D: Having the patient perform passive ROM of the affected limb with the unaffected limb- active ROM should be initiated on the unaffected side as soon as possible, and passive ROM of the affected side should be started on the first day. Having the patient actively exercise the unaffected side provides the patient with active and passive ROM as needed. Use of footboards is controversial because they stimulate plantar flexion. The unaffected arm should be supported, but immobilization may precipitate a painful shoulder-hand syndrome. The patient should be positioned with each joint higher than the joint proximal to it to prevent dependent edema.

The nurse can assist the patient and the family in coping with the long term effects of a stroke by a. informing family members that the patient will need assistance with almost all ADLs b. explaining that the patient's prestroke behavior will return as improvement progresses c. encouraging the patient and family members to seek assistance from family therapy or stroke support groups d. helping the patient and family understand the significance of residual stroke damage to promote problem solving and planning

D: Helping the patient and family understand the significance of residual stroke damage to promote problem solving and planning- the patient and family need accurate and complete information about the effects of the stroke to problem solve and make plans for chronic care of the patient. It is uncommon for patients with major strokes to return completely to pre stroke function, behaviors, and role, and both the patient and family will mourn these losses. The patient's specific needs for care must be identified, and rehabilitation efforts should be continued at home. Family therapy and support groups may be helpful for some patients and families.

The priority intervention in the emergency department for the patient with a stroke is a. intravenous fluid replacement b. administration of osmotic diuretics to reduce cerebral edema c. initiation of hypothermia to decrease the oxygen needs of the brain d. maintenance of respiratory function with a patent airway and oxygen administration

D: Maintenance of respiratory function with a patent airway and oxygen administration- the first priority in acute management of the patient with a stroke is preservation of life. Because the patient with a stroke may be unconscious or have a reduced gag reflex, it is most important to maintain a patent airway for the patient and provide oxygen if respiratory effort is impaired. IV fluid replacement, treatment with osmotic diuretics, and perhaps hypothermia may be used for further treatment.

A patient's wife asks the nurse why her husband did not receive the clot busting medication (tPA) she has been reading about. Her husband is diagnosed with a hemorrhagic stroke. What should the nurse respond? a. He didn't arrive within the time frame for that therapy b. Not every is eligible for this drug. Has he had surgery lately? c. You should discuss the treatment of your husband with your doctor d. The medication you are talking about dissolves clots and could cause more bleeding in your husband's head

D: The medication you are talking about dissolves clots and could cause more bleeding in your husband's head- tPA dissolves clots and increases the risk for bleeding. It is not used with hemorrhagic strokes. If the patient had a thrombotic/embolic stroke the time frame would be important as well as a history of surgery. The nurse should answer the question as accurately as possible and then encourage the individual to talk with the primary care physician if he or she has further questions.

A patient comes to the emergency department immediately after experiencing numbness of the face and an inability to speak, but while the patient awaits examination, the symptoms disappear and the patient request discharge. The nurse stresses that it is important for the patient to be evaluated primarily because a. the patient has probably experienced an asymptomatic lacunar stroke b. the symptoms are likely to return and progress to worsening neurologic deficit in the next 24 hours c. neurologic deficits that are transient occur most often as a result of small hemorrhages that clot off d. the patient has probably experienced a transient ischemic attack (TIA), which is a sign of progressive cerebral vascular disease

D: The patient has probably experienced a transient ischemic attack (TIA), which is a sign of progressive cerebral vascular disease- A TIA is a temporary focal loss of neurologic function caused by ischemia of an area of the brain, usually lasting only about 3 hours. TIAs may be due to microemboli from heart disease or carotid or cerebral thrombi and are a warning of progressive disease. Evaluation is necessary to determine the cause of the neurologic deficit and provide prophylactic treatment if possible.

What are the key features of ICP?

Decreased LOC, behavior changes (restlessness, irritability, and confusion), headache, nausea and vomiting, and change in speech pattern

Describe severe brain injury

Defined by a GCS score of 3 to 8 and LOC of longer than 6 hours. Critical care required. Neurologic assessment should be every 10 to 15 minutes. **monitoring for intracranial pressure regularly**

Primary prevention of ICP

Diet, exercise, and avoidance of tobacco products

What is a coup-contrecoup injury?

Dual impacting of the brain into the skull; coup injury occurs at the point of impact; contrecoup injury occurs on the opposite side of impact, as the brain rebounds.

What is the main reason why an 18 yr old would have a stroke?

Due to illicit drug use such as meth and cocaine. Causes impaired clotting and is a hemorrhagic type.

What are the early symptoms of ICP: adults

Headache or change in consciousness

Vessel integrity is interrupted and bleeding occurs into the brain tissue or into the subarachnoid space—? What type of stroke?

Hemorrhagic stroke

Characteristics of subarachnoid hemorrhage

High initial mortality, symptoms of meningeal irritation, caused by rupture of intracranial aneurysm, associated with sudden, severe headache

Carotid artery angioplasty with stenting..

High risk of stroke following procedure, and assessment is key post-op. Aspirin to be given within 24-48 hours.

What is hydrocephalus?

Hydrocephalus is an abnormal increase in CSF volume. It may be caused by impaired reabsorption of CSF at the arachnoid villi (from subarachnoid hemorrhage or meningitis), called a communicating hydrocephalus.

What are the two main causative agents of a stroke?

Hypertension, and anteriovenous malformation

Assessment of the patient's ability to swallow reflects the function of CNs

IX and X

Receptive (wernicke's or sensory) aphasia is due to ?

Injury involving wernicke's area in the temporoparietal area. The patient cannot understand the spoken and often written word.

Most common artery involved

Internal Carotid Artery

What are the early symptoms of ICP: infants

Irritability, lethargy, poor feeding, may have bulge of fontanal

What is the national institutes of health stroke scale?

Is a commonly used valid and reliable assessment tool that nurses complete as soon as possible after the patient arrives in the ED.

The first sign of increased ICP is what?

Is a declining level of consciousness

Describe moderate brain injury

Is characterized by a period of loss of consciousness for 30 minutes to 6 hours and GCS score of 9 to 12

Which stroke is caused by an occlusion of a cerebral artery by either a thrombus or an embolus?

Ischemic

What are the two major classifications of stroke?

Ischemic and hemorrhagic

What is AVM (arteriovenous malformation)?

It is genetic. Usually occurs between the ages of 30-40. Capillaries get congested and eventually burst.

Right hemiplegia or hemiparesis indicates stroke involving which cerebral hemisphere?

LEFT

Which cerebral hemisphere stroke results in aphasia, alexia or dyslexia, agraphia and acalulia?

LEFT

Manifestations of right brain damage

Left homonymous hemianopsia, agnosia, quick impulsive behavior, neglect of the left side of the body

Treat for hemorrhagic stroke

Lower BP

The best practice for all suspected and diagnosed stroke patients is to?

Maintain a NPO status until their swallowing ability is assessed.

What is intracranial regulation?

Maintenance of balance to promote an environment conducive to optimal brain function.

Subarachnoid hemorrhage is described as?

Much more common. Results from bleeding into the subarachnoid space— **usually caused by a ruptured aneurysm or anteriovenous malformation. Can also be caused by trauma**

Secondary prevention for ICP

No true screening measures related to intracranial regulation

1st thing you do when you suspect a stroke

Non-contrast CT scan (determines hemorrhagic vs ischemic)

Characteristics of embolic stroke

Onset unrelated to activity, quick onset and resolution, associated with endocardial disorders

What is ICP

Pathological condition or trauma causes pressure within the cranial vault to increase

What is neurotransmission?

Process of sending signals from nerve to nerve across a synapse

What are lucid intervals??

Pts with epidural hematomas have lucid intervals that last for minutes during which time the patient is awake and walking. This then follows a momentary unconsciousness that occurs within minutes of the injury.

Which cerebral hemisphere is more involved with visual and spatial awareness and proprioception?

RIGHT

Characteristics of intracerebral hemorrhage

Rupture of atherosclerotic vessels, carries the poorest prognosis, creates mass that compresses the brain

What is the 5th leading cause of death in older adults?

TBI **more specifically stroke, hematoma**

Typically symptoms of a ______ RESOLVE within 30-60 minutes

TIA's **TIAs indicate high stroke risk, recurrent and multiple TIAs increase the risk for permanent brain damage.**

Characteristics of a thrombotic stroke

Type most often signaled by TIAs, commonly occurs during or after sleep, strong association with hypertension

Ischemic strokes often follow warning signs such as what?

Transient ischemic attack. **temporary neurologic dysfunction resulting from a brief interruption in cerebral blood flow is easy to ignore or miss**

Ischemic strokes are usually caused by WHAT?

Usually caused by heart ailments like AFIB

dysarthria

a disturbance in the muscular control of speech, resulting from interference in the control and execution over the muscles of speech, usually caused by damage to a central or peripheral motor nerve

subarachnoid hemorrhage

a stroke resulting from intracranial bleeding into the cerebrospinal fluid-filled space between the arachnoid and pia mater membranes on the surface of the brain

thrombotic stroke

a stroke resulting from thrombosis or narrowing of the blood vessel

embolic stroke

a stroke that occurs when an embolus lodges in and occludes a cerebral artery, resulting in infarction and edema of the area supplied by the involved vessel

hemorrhagic stroke

a stroke that results from bleeding into the brain tissue itself (intracerebral or intraparenchymal hemorrhage) or into the subarachnoid space or ventricles (subarachnoid hemorrhage or intraventricular hemorrhage)

transient ischemic attack (TIA)

a transient episode of neurologic dysfunction caused by focal brain, spinal cord, or retinal ischemia, but without acute infarction of the brain. Clinical symptoms typically last less than 1 hour

intracerebral hemorrhage

a type of hemorrhagic stroke in which bleeding within the brain caused by a rupture of a blood vessel occurs; often caused by hypertension and is associated with increased intracranial pressure

Common psychosocial reactions of the stroke patient to the stroke include (select all that apply) a. depression. b. disassociation. c. intellectualization. d. sleep disturbances. e. denial of severity of stroke.

a, d, e Rationale: The patient with a stroke may experience many losses, including sensory, intellectual, communicative, functional, role behavior, emotional, social, and vocational losses. Some patients experience long-term depression, manifesting symptoms such as anxiety, weight loss, fatigue, poor appetite, and sleep disturbances. The time and energy required to perform previously simple tasks can result in anger and frustration. Frustration and depression are common in the first year after a stroke. A stroke is usually a sudden, extremely stressful event for the patient, caregiver, family, and significant others. The family is often affected emotionally, socially, and financially as their roles and responsibilities change. Reactions vary considerably but may involve fear, apprehension, denial of the severity of stroke, depression, anger, and sorrow.

aphasia

an abnormal neurologic condition in which language function is disordered or absent because of an injury to certain areas of the cerebral cortex

For a patient who is suspected of having a stroke, one of the most important pieces of information that the nurse can obtain is a. time of the patient's last meal. b. time at which stroke symptoms first appeared. c. patient's hypertension history and management. d. family history of stroke and other cardiovascular diseases.

b Rationale: During initial evaluation, the most important point in the patient's history is the time since onset of stroke symptoms. If the stroke is ischemic, recombinant tissue plasminogen activator (tPA) must be administered within 3 to 4.5 hours of the onset of clinical signs of ischemic stroke; tPA reestablishes blood flow through a blocked artery and prevents brain cell death in patients with acute onset of ischemic stroke.

The nurse explains to the patient with a stroke who is scheduled for angiography that this test is used to determine the a. presence of increased ICP. b. site and size of the infarction. c. patency of the cerebral blood vessels. d. presence of blood in the cerebrospinal fluid.

c Rationale: Angiography provides visualization of cerebral blood vessels and can help estimate perfusion and detect filling defects in the cerebral arteries.

A patient with right-sided hemiplegia and aphasia resulting from a stroke most likely has involvement of the a. brainstem. b. vertebral artery. c. left middle cerebral artery. d. right middle cerebral artery.

c Rationale: If the middle cerebral artery is involved in a stroke, the expected clinical manifestations include aphasia, motor and sensory deficit, and hemianopsia on the dominant side and include neglect, motor and sensory deficit, and hemianopsia on the nondominant side.

A patient experiencing TIAs is scheduled for a carotid endarterectomy. The nurse explains that this procedure is done to a. decrease cerebral edema. b. reduce the brain damage that occurs during a stroke in evolution. c. prevent a stroke by removing atherosclerotic plaques blocking cerebral blood flow. d. provide a circulatory bypass around thrombotic plaques obstructing cranial circulation.

c Rationale: In a carotid endarterectomy, the atheromatous lesion is removed from the carotid artery to improve blood flow.

Bladder training in a male patient who has urinary incontinence after a stroke includes a. limiting fluid intake. b. keeping a urinal in place at all times. c. assisting the patient to stand to void. d. catheterizing the patient every 4 hours.

c Rationale: In the acute stage of stroke, the primary urinary problem is poor bladder control and incontinence. Nurses should promote normal bladder function and avoid the use of indwelling catheters. A bladder retraining program consists of (1) adequate fluid intake, with most fluids administered between 7:00 am and 7:00 pm; (2) scheduled toileting every 2 hours with the use of a bedpan, commode, or bathroom; and (3) noting signs of restlessness, which may indicate the need for urination. Intermittent catheterization may be used for urinary retention (not urinary incontinence). During the rehabilitation phase after a stroke, nursing interventions focused on urinary continence include (1) assessment for bladder distention by palpation; (2) offering the bedpan, urinal, commode, or toilet every 2 hours during waking hours and every 3 to 4 hours at night; (3) using a direct command to help the patient focus on the need to urinate; (4) assistance with clothing and mobility; (5) scheduling most fluid intake between 7:00 am and 7:00 pm; and (6) encouraging the usual position for urinating (i.e., standing for men and sitting for women).

The factor related to cerebral blood flow that most often determines the extent of cerebral damage from a stroke is the a. amount of cardiac output. b. oxygen content of the blood. c. degree of collateral circulation. d. level of carbon dioxide in the blood.

c Rationale: The extent of the stroke depends on the rapidity of onset, size of the lesion, and presence of collateral circulation.

What is a subdural hematoma?

results from venous bleeding into the space beneath the dura and above the arachnoid ** It occurs most often from a tearing of the bridging veins within the cerebral hemispheres or from a laceration of brain tissue.**

dysphasia

difficulty related to the comprehension or use of language

aneurysms

congenital or acquired weakness of the arterial wall resulting in dilation and ballooning of the vessel

Information provided by the patient that would help differentiate a hemorrhagic stroke from a thrombotic stroke includes a. sensory disturbance. b. a history of hypertension. c. presence of motor weakness. d. sudden onset of severe headache.

d Rationale: A hemorrhagic stroke usually causes a sudden onset of symptoms, which include neurologic deficits, headache, nausea, vomiting, decreased level of consciousness, and hypertension. Ischemic stroke symptoms may progress in the first 72 hours as infarction and cerebral edema increase.

Of the following patients, the nurse recognizes that the one with the highest risk for a stroke is a(n) a. obese 45-year-old Native American. b. 35-year-old Asian American woman who smokes. c. 32-year-old white woman taking oral contraceptives. d. 65-year-old African American man with hypertension.

d Rationale: Nonmodifiable risk factors for stroke include age (older than 65 years), male gender, ethnicity or race (incidence is highest in African Americans; next highest in Hispanics, Native Americans/Alaska Natives, and Asian Americans; and next highest in white people), and family history of stroke or personal history of a transient ischemic attack or stroke. Modifiable risk factors for stroke include hypertension (most important), heart disease (especially atrial fibrillation), smoking, excessive alcohol consumption (causes hypertension), abdominal obesity, sleep apnea, metabolic syndrome, lack of physical exercise, poor diet (high in saturated fat and low in fruits and vegetables), and drug abuse (especially cocaine). Other risk factors for stroke include a diagnosis of diabetes mellitus, increased serum levels of cholesterol, birth control pills (high levels of progestin and estrogen), history of migraine headaches, inflammatory conditions, hyperhomocystinemia, and sickle cell disease.

Describe a mild brain injury

is characterized by a blow to the head, transient confusion or feeling dazed or disoriented, and one or more to these conditions— 1) LOC for up to 30 min 2) loss of memory for events immediately before or after accident 3) focal neurologic deficits that may or may not be transient **mild traumatic brain injury and concussion are used synonymously**

What is a intracerebral hemorrhage?

is the accumulation of blood within the brain tissue caused by the tearing of small arteries and veins in the subcortical white matter. **ICH may also produce significant brain edema and ICP elevations**

Problem with IV-TPA

only 1-3% of patients arrive in time to receive it

What is a epidural hematoma?

results from arterial bleeding into the space between the dura and the inner skull ** It is often caused by a fracture of the temporal bone, which houses the middle meningeal artery**

ischemic stroke

stroke that results from inadequate blood flow to the brain caused by partial or complete occlusion of an artery

As a nurse what cognitive changes are you going to look for (4 of them specifically)

• Denial of the illness • Spatial and proprioceptive (awareness of body position in space) dysfunction • Impairment of memory, judgment, or problem-solving and decision-making abilities • Decreased ability to concentrate and attend to tasks Dysfunction in one or more of these areas may be severe depending on the hemisphere involved

What are the 5 MOST common symptoms of stroke

• Sudden confusion or trouble speaking or understanding others • Sudden numbness or weakness of the face, arm, or leg • Sudden trouble seeing in one or both eyes • Sudden dizziness, trouble walking, or loss of balance or coordination • Sudden severe headache with no known cause


Related study sets

Chapter 53: Drug Therapy for Seizure Disorders and Spasticity

View Set

EAQ Maternity, Women's Health Disorders Childerbearing Health Promotion

View Set

ACSM's Guidelines for Exercise Testing and Prescription Chapters 1-12

View Set

Physician Assistant Interview-Example 3

View Set

Sample Professional Scrum Master Exam

View Set